NCLEX Endocrine Review

Réussis tes devoirs et examens dès maintenant avec Quizwiz!

fluids

Replacing ________ is the first treatment of hyperglycemic hyperosmolar nonketotic coma

"A client is admitted for treatment of the syndrome of inappropriate antidiuretic hormone (SIADH). Which nursing intervention is appropriate?

Restricting fluids

Liver biopsy:

Right side position post procedure to prevent patient from bleeding.

"A client is transferred to a rehabilitation center after being treated in the hospital for a stroke. Because the client has a history of Cushing's syndrome (hypercortisolism) and chronic obstructive pulmonary disease (COPD), the nurse helps formulate a nursing diagnosis of:

Risk for impaired skin integrity related to tissue catabolism secondary to cortisol hypersecretion.

"Which nursing diagnosis is most appropriate for a client with Addison's disease?

Risk for infection

"A client with type 1 diabetes must learn how to self-administer insulin. The physician has prescribed 10 units of U-100 regular insulin and 35 units of U-100 isophane insulin suspension (NPH) to be taken before breakfast. When teaching the client how to select and rotate insulin injection sites, the nurse should provide which instruction?

Rotate injection sites within the same anatomic region, not among different regions.

symptoms of CAD are:

SOB, chest pain and increased pulse.

Thyroidectomy:

Semi Fowler and avoid hyperflexion and hyperextension of the neck

"A client with a tentative diagnosis of hyperosmolar hyperglycemic nonketotic syndrome (HHNS) has a history of type 2 diabetes, which is controlled by tolazamide (Tolinase). What is the most important laboratory test for confirming HHNS?

Serum osmolarity

Hypoparathyroidism

Signs of __________________ include paresthesia, muscle creamps and tetany, Chvostek's and Trousseau's signs, alopecia, dry skin, and painful menstruation

What is the pathophysiology for DKA?

Since there is no insulin, glucose builds up in the vascular space > the blood becomes hypertonic and pulls fluid into the vascular space > the kidneys filter excess glucose and fluids (polyuria, polydipsia) and the cells are astarving so they break down protein and fat for energy (polyphagia) > when you breakdown fat you get ketones > now the client becomes metabolically acidotic.

Risk factors for type 2 diabetes include all of the following except: "a. Advanced age b. Obesity c. Smoking d. Physical inactivity"

Smoking "Additional risk factors for type 2 diabetes are a family history of diabetes, impaired glucose metabolism, history of gestational diabetes, and race/ethnicity. African-Americans, Hispanics/Latinos, Asian Americans, Native Hawaiians, Pacific Islanders, and Native Americans are at greater risk of developing diabetes than whites."

Increase what in diet for addisons??

Sodium - remember low aldosterone - not retain sodium and remember if they eat more sodium than they will retain more water.

Mineralocorticoids/Aldosterone: makes you retain ______, but inversely makes you lose _________. Too much aldosterone ______ vascular space and _____ serum potassium.

Sodium, Potassium. Increases. Decreases.

Gluco-corticosteroids

Solumedrol-used for decrease adrenal ftn, modifying the body's immune response to various conditions & decrease inflammation. Decadron (dexamethezone)-treat certain conditions associated with decrease adrenal ftn, works by decreasing or preventing tissue from responding to inflammation. Prednizone

"A client who has had type 2 diabetes for 20 years tells the nurse that sometimes she has diarrhea and other times constipation. In addition, she sometimes feels ""full"" after eating small amounts. Which of the following would be an appropriate response for the nurse to make?

Sometimes people with diabetes have problems with their digestion. Did you tell your physician about this?

catecholamines

Stimulate the "fight or flight" response to danger; sympathetic nervous system response (epinephrine, norepinephrine)

Insulin

Stimulates active transport of glucose into muscle and adipose tissue Stimulates protein synthesis Promotes conversion of glucose to glycogen for storage Promotes conversion of fatty acids into fat

Insulin

Stimulates protein synthesis

Insulin

Stimulating active transport of glucose into muscle and adipose tissue

"Parathyroid hormone (PTH) has which effect on the kidneys?

Stimulation of calcium reabsorption and phosphate excretion

If someone with hypothyroid is getting Synthroid as a treatment and experience signs of Coranary Artery Disease, what do you do?

Stop the infusion!

glucocorticoids

Substances that affect carb, fat, and protein metabolism; affect stress reactions and the inhibition of the inflammatory process (cortisol, cortisone, corticosterone)

Addisonian Crisis

Sudden extreme weakness, severe abdominal, back, and leg pain; hyperpyrexia; coma; and death secondary to physical stress or trauma

"A client with type 2 diabetes comes to the clinic with a diabetic foot ulcer on his left heel that hasn't responded to treatment. Which action should a nurse take after assessing the ulcer?

Suggest a consult with a wound care specialist.

Addison's disease

Symptoms include fatigue, anorexia, weight loss, hyperpigmentation, hypotension, hypoglycemia, hyponatremia, hyperkalemia

What 3 hormones do the thyroid produce? What product is needed to make these hormones?

T3, T4, & Calcitonin. Iodide.

"The nurse explains to a client with thyroid disease that the thyroid gland normally produces:

T3, T4, and calcitonin.

"A client with hypothyroidism (myxedema) is receiving levothyroxine (Synthroid), 25 mcg P.O. daily. Which finding should the nurse recognize as an adverse drug effect?

Tachycardia

A client with type 1 diabetes calls the nurse because he is nauseous and does not feel well. The client asks if he should take his medication. What would be important for the nurse to tell the client?

Take his regular dose of insulin, replace food with fruit juices, and monitor the blood glucose level.

"Which instruction concerning the administration of levothyroxine (Synthroid) should the nurse teach a client?

Take the drug on an empty stomach.

"A client with type 2 diabetes was diagnosed with retinopathy. While a nurse reviews the client's medication dosage, the client states, ""I can't read the names on the medicine bottles, so I hope I'm taking the right pills at the right time."" What should the nurse do with this information?

Teach the client how to tell the difference between the medicine bottles.

Androgens

Testosterone-used in men & boys to treat conditions caused by lack of hormone Virilon-stimulate growth in many tissues especally bone and muscle.

"A nurse is prioritizing care for her four-client assignment. Which client should she attend to first?

The client who requires an insulin injection before eating breakfast

"A client with type 2 diabetes tells the nurse in the clinic, ""I keep gaining weight even though I'm not eating all that much. I can't exercise anymore because of these ulcers on my feet. I don't know what to do."" What would be an appropriate response for the nurse to make to this client?

There are other types of exercise that you can do even though you have ulcers on your feet.

Tx for Hypothyroid include which meds? Do they take these forever?

These meds all sound like they have thyroid in them in a way... levothyroxine (synthroid) thyroglobulin (proloid) liothyronine (cytomel) / Yes

"A diabetic client develops sinusitis and otitis media accompanied by a temperature of 100.8° F (38.2° C). What effect may these findings have on his need for insulin?

They will increase the need for insulin.

List the three symptoms of hyperglycemia?

Think three P's: polyuria (urine loss due to hypertonic vascular fluid and accompanying weight loss), polydipsia , polyphagia (breakdown of fat/protein).

Myxedema is a disease which causes low ________ levels.

Thyroid (hypothyroidism).

"Early this morning, a client had a subtotal thyroidectomy. During evening rounds, the nurse assesses the client, who now has nausea, a temperature of 105° F (40.5° C), tachycardia, and extreme restlessness. What is the most likely cause of these signs?

Thyroid crisis

A client experinces a thyroid storm after removal of the throid gland. The nurse understands the cause of this complication is?

Thyroid hormones moving into the bloodstream during thyroid surgery

"A client has type 1 diabetes. Her husband finds her unconscious at home and administers glucagon, 0.5 mg subQ. She awakens in 5 minutes. Why should her husband offer a complex carbohydrate snack to her as soon as possible?

To restore liver glycogen and prevent secondary hypoglycemia

"A client is being returned to his room after a subtotal thyroidectomy. Which piece of equipment is most important for the nurse to keep at the client's bedside?

Tracheostomy set

graves disease treatment

Treated by surgical removal of the thyroid or partial destruction of thyroid with radioactive iodine

"A client has a serum calcium level of 7.2 mg/dl. During the physical examination, the nurse expects to assess:

Trousseau's sign.

Client with hyperthyroidism may experince a problem with bulging eyes. What nursing intervention is important in caring for this problem?

Use saline eye drops frequently and apply ointment to the eyes at night

"For a client with hyperthyroidism, treatment is most likely to include:

a thyroid hormone antagonist.

diabetes insipidus

is a result of ADH defeicency may result from a brain tumor or infections pituitary surgery, CVA or renal or organ failure

"A client with Addison's disease comes to the clinic for a follow-up visit. When collecting data on this client, the nurse should stay alert for signs and symptoms of:

sodium and potassium abnormalities.

Aldosterone makes you retain: and makes you lose:

sodium and water / potassium

"A client with type 2 diabetes tells a nurse that he stopped walking at the mall because of his ""bad leg pain."" How should the nurse respond to this client?

Did you notify your physician when you started to have the leg pains?

long-lasting

Do not mix ___________- insulins with any other insulin or solution

Since the client has too much aldosterone in Cushing's, the serum K+ will go ______.

Down.

"When teaching a client about insulin administration, the nurse should include which instruction?

Draw up clear insulin first when mixing two types of insulin in one syringe.

An appropriate goal of nursing care for a pt with end-stage renal disease is the pt will be able to do which of the following?

state the advantages and disadvantages of types of renal replacement therapies

what disease is adrenocortical insufficiency or not enough _______.

steriods / addison's disease

"The nurse is caring for a patient whose blood glucose level is 55mg/dL. What is the likely nursing response? "A. Administer a glucagon injection B. Give a small meal C. Administer 10-15 g of a carbohydrate D. Give a small snack of high protein food"

"C The client has low hypoglycemia. This is generally treated with a small snack."

Aclient is admitted with a diagnosis of Cushing's syndrome. What is an important consideration for the nurse to make in caring for this client?

Due to decreased inflammatory response the client will be at increased risk for infections.

24 hr

Duration of long-lasting insulins

What will the nurse teach the diabetic client regarding exercise in their treatment program?

During exercise the body will use carbohydrates for energy production, which will decreased the need for insulin

"(SELECT ALL THAT APPLY) A client is seen in the clinic with suspected parathormone (PTH) deficiency. Part of the diagnosis of this condition includes the analysis of serum electrolyte levels. Which electrolyte levels would the nurse expect to be abnormal in a client with PTH deficiency?

(1) Calcium (6) Phosphorous

ACTH are ___________ and are made where? The stimulate what to be made?

steriods and are made in the pituitary and they stimulate cortisol to be made.

thyroid scan

stop ingestion of iodine (iodized salt, seafood), do not take prescribed thyroid medication because it may interfere with the scan

"(SELECT ALL THAT APPLY) After falling off a ladder and suffering a brain injury, a client develops syndrome of inappropriate antidiuretic hormone (SIADH). Which findings indicate that the treatment he's receiving is effective?

(1) Decrease in body weight (4) Increased urine output (5) Decreased urine osmolarity

"(SELECT ALL THAT APPLY) A client is admitted to the hospital with signs and symptoms of diabetes mellitus. Which of the following findings is the nurse most likely to observe in this client?

(1) Excessive thirst (4) Excessive hunger (6) Frequent, high-volume urination

Analyze the following diagnostic findings for your patient with type 2 diabetes. Which result will need further assessment? A) BP 126/80 B) A1C 9% C)FBG 130mg/dL D) LDL cholesterol 100mg/dL

"B) A1C 9% Rationale: Lowering hemoglobin A1C (to average of 7%) reduces microvascular and neuropathic complications. Tighter glycemic control(normal A1C < 6%) may further reduce complications but increases hypoglycemia risk."

The nurse is caring for a client who has normal glucose levels at bedtime, hypoglycemia at 2am and hyperglycemia in the morning. What is this client likely experiencing? "A. Dawn phenomenon B. Somogyi effect C. An insulin spike D. Excessive corticosteroids"

"1. B The Somogyi effect is when blood sugar drops too low in the morning causing rebound hyperglycemia in the morning. The hypoglycemia at 2am is highly indicative. The Dawn phenomenon is similar but would not have the hypoglycemia at 2am."

"The nurse is teaching a community class to peole with Type 2 diabetes mellitus. Which explanation would explain the development of Type 2 diabetes? 1. The islet cells in the pancreas stop producing insulin. 2. The client eats too many foods that are high in sugar. 3 The pituitary gland does not produce vasopression. 4. The cells become resistant to the circulating insulin.

"1. This is the cause of Type 1 diabetes mellitus. 2. This may be a reason for obesity, which may lead to Type 2 diabetes, but eating too much sugar does not cause diabetes. 3. This is the explanation for diabetes insipidus, which should not be confused with diabetes mellitus. 4. (CORRECT) Normally insulin binds to special receptor sites on the cells and initiates a series of reactions involved in metabolism. In Type 2 diabetes these reactions are diminished primarily as a result of obesity and aging."

"The nurse is caring for a woman at 37 weeks gestation. The client was diagnosed with insulin-dependent diabetes mellitis (IDDM) at age 7. The client states, ""I am so thrilled that I will be breastfeeding my baby."" Which of the following responses by the nurse is best? "1. You will probably need less insulin while you are breastfeeding. 2. You will need to initially increase your insulin after the baby is born. 3. You will be able to take an oral hypoglycemic instead of insulin after the baby is born. 4. You will probably require the same dose of insulin that you are now taking."

"1. breastfeeding has an antidiabetogenic effect, less insulin is needed. (correct) 2. insulin needs will decrease due to antidiabetogenic effect of breastfeeding and physiological changes during immediate postpartum period. 3. client has IDDM, insulin required. 4. during third trimester insulin requirements increase due to increased insulin resistance"

"The client diagnosed with Type 1 diabetes has a glycosylated hemoglobin (A1 c) of 8.1%. Which interpretation should the nurse make based on this result? 1.This result is below normal levels.2.This result is within acceptable levels. 3.This result is above recommended levels 4.This result is dangerously high.

"1.The acceptable level for an A1c for a client with diabetes is between 6% and 7%, which corresponds to a 120-140 mg/dL average blood glucose level. 2.This result is not within acceptable levelsfor the client with diabetes, which is 6% to7%. 3.(CORRECT) This result parallels a serum blood glucoselevel of approximately 180 to 200 mg/dL. An A1 c is a blood test that reflects average blood glucose levels over a period of 2-3months; clients with elevated blood glucose levels are at risk for developing long-term complications. 4.An A1c of 13% is dangerously high; it reflects a 300-mg/dL average blood glucose level overthe past 3 months."

"An 18-year-old female client, 5'4'' tall, weighing 113 kg, comes to the clinic for a non-healing wound on her lower leg, which she has had for two weeks. Which disease process should the nurse suspect the client is developing? "A. Type 1 diabetes B. Type 2 diabetes C. Gestational diabetes D. Acanthosis nigricans"

"A: Type 1 diabetes usually occurs in young clients who are underweight. In this disease, there is no production of insulin from the beta cells in the pancreas. People with type 1 diabetes are insulin dependent with a rapid onset of symptoms, including polyuria, polydipsia, and polyphagia. CORRECT -->B. Type 2 diabetes is a disorder usually occurring around the age of 40, but it is now being detected in children and young adults as a result of obesity and sedentary lifestyles. Non-healing wounds are a hallmark sign of type 2 diabetes. This client weights 248.6 lbs and is short. C. Gestational diabetes occurs during pregnancy. There is no mention of this. D. Acanthosis nigricans (AN), dark pigmentation and skin creases in the neck, is a sign of hyperinsulinemia. The pancreas is secreting excess amounts of insulin as a result of excessive caloric intake. It is identified in young children and is a precursor to the development of type 2 diabetes."

The nurse is educating a pregnant client who has gestational diabetes. Which of the following statements should the nurse make to the client? Select all that apply. "a. Cakes, candies, cookies, and regular soft drinks should be avoided. b. Gestational diabetes increases the risk that the mother will develop diabetes later in life. c. Gestational diabetes usually resolves after the baby is born. d. Insulin injections may be necessary. e. The baby will likely be born with diabetes f. The mother should strive to gain no more weight during the pregnancy.

"ANS: A, B, C, D Gestational diabetes can occur between the 16th and 28th week of pregnancy. If not responsive to diet and exercise, insulin injections may be necessary. Concentrated sugars should be avoided. Weight gain should continue, but not in excessive amounts. Usually, gestational diabetes disappears after the infant is born. However, diabetes can develop 5 to 10 years after the pregnancy"

"Prediabetes is associated with all of the following except: " a. Increased risk of developing type 2 diabetes b. Impaired glucose tolerance c. Increased risk of heart disease and stroke d. Increased risk of developing type 1 diabetes"

"ANSWER: D Persons with elevated glucose levels that do not yet meet the criteria for diabetes are considered to have prediabetes and are at increased risk of developing type 2 diabetes. Weight loss and increasing physical activity can help people with prediabetes prevent or postpone the onset of type 2 diabetes."

"Which instruction about insulin administration should the nurse give to a client?

"Always follow the same order when drawing the different insulins into the syringe.

During a diabetes screening program, a patient tells the nurse, "My mother died of complications of type 2 diabetes. Can I inherit diabetes?" The nurse explains that "a.) as long as the patient maintains normal weight and exercises, type 2 diabetes can be prevented. b.) the patient is at a higher than normal risk for type 2 diabetes and should have periodic blood glucose level testing. c.) there is a greater risk for children developing type 2 diabetes when the father has type 2 diabetes. d.) although there is a tendency for children of people with type 2 diabetes to develop diabetes, the risk is higher for those with type 1 diabetes."

"B Rationale: Offspring of people with type 2 diabetes are at higher risk for developing type 2 diabetes. The risk can be decreased, but not prevented, by maintenance of normal weight and exercising. The risk for children of a person with type 1 diabetes to develop diabetes is higher when it is the father who has the disease. Offspring of people with type 2 diabetes are more likely to develop diabetes than offspring of those with type 1 diabetes."

Polydipsia and poly uria related to diabetes mellitus are primarily due to: "a.The release of ketones from cells during fat metabolism b. Fluid shifts resulting from exposure to high levels of hyperglycemia c. Damage to the kidneys from exposure to high levels of glucose d. changes in RBCs resulting from attachemnt of excessibe glucose to hemoglobin"

"Coorect answer: d. Rationale: The osmotic effect of glucose produces the manifesatiaions of polydispsia and poly uria."

"The client, an 18-year-old female, 5'4'' tall, weighing 113 kg, comes to the clinic for a wound on her lower leg that has not healed for the last two (2) weeks. Which diseaseprocess would the nurse suspect that the client has developed? "1.Type 1 diabetes. 2.Type 2 diabetes. 3.Gestational diabetes. 4.Acanthosis nigricans"

"Correct Answer: 2 Type 2 diabetes is a disorder that usually occurs around the age of 40, but it is now being detected in children and young adultsas a result of obesity and sedentary life-styles. Wounds that do not heal are a hall-mark sign of Type 2 diabetes. This client weighs 248.6 pounds and is short"

Which statement by the patient with type 2 diabetes is accurate. "a. ""I am supposed to have a meal or snak if I drink alcohol"" b. ""I am not allowed to eat any sweets because of my diabetes."" c. I do not need to watch what I eat because my diabetes is not the bad kind."" d. The amunt of fat in my diet is not important; it is just the carbohydrates that raise my blood sugar."""

"Correct Answer: A Alcohol should be consumed with food to reduce the risk of hypoglycemia."

A client with diabetes melllitus has a blood glucose of 644mg/dl. The nurse intreprets that this client is most at risk of developing which type of acid base imbalance? "A. Metabolic acidosis B. Metabolic alkalosis C. Respiratory Acidosis D. Respiratory Alkalosis"

"Correct Answer: A, Metabolic Acidosis Rationale: DM can lead to metabolic acidosis. When the body does not have sufficient circulating insulin, the blood glucose level rises. At the same time, the cells of the body use all available glucose. The body then breaks down glycogen and fat for fuel. The by-products of fat metabolism are acidotic and can lead to the condition known as diabetic ketoacidosis."

"(SELECT ALL THAT APPLY) A client is placed on hypocalcemia precautions after removal of the parathyroid gland for cancer. The nurse should observe the client for which symptoms?

(1) Numbness (3) Tingling (4) Muscle twitching and spasms

"Which of the following persons would most likely be diagnosed with diabetes mellitus? A 44-year-old.. "A. Caucasian Woman B. Asian Woman C. African-American woman D. Hispanic Male

"Correct answer: African-American woman Rationale: Age-specific prevalence of diagnosed diabetes mellitus (DM) is higher for African-Americans and Hispanics than for Caucasians. Among those younger than 75, black women had the highest incidence."

"Excessive thirst and volume of very dilute urine may be symptoms of: "A. Urinary tract infection B. Diabetes insipidus C. Viral gastroenteritis D.Hypoglycemia"

"Correct answer: B Diabetes insipidus is a condition in which the kidneys are unable to conserve water, often because there is insufficient antidiuretic hormone (ADH) or the kidneys are unable to respond to ADH. Although diabetes mellitus may present with similar symptoms, the disorders are different. Diabetes insipidus does not involve hyperglycemia."

"A patient with type 1 diabetes has received diet instruction as part of the treatment plan. The nurse determines a need for additional instruction when the patient says, "a. ""I may have an occasional alcoholic drink if I include it in my meal plan."" b. ""I will need a bedtime snack because I take an evening dose of NPH insulin."" c. ""I will eat meals as scheduled, even if I am not hungry, to prevent hypoglycemia."" d. ""I may eat whatever I want, as long as I use enough insulin to cover the calories.

"D. ""I may eat whatever I want, as long as I use enough insulin to cover the calories."" Rationale: Most patients with type 1 diabetes need to plan diet choices very carefully. Patients who are using intensified insulin therapy have considerable flexibility in diet choices but still should restrict dietary intake of items such as fat, protein, and alcohol. The other patient statements are correct and indicate good understanding of the diet instruction."

"The principal goals of therapy for older patients who have poor glycemic control are: "A. Enhancing quality of life. B. Decreasing the chance of complications. C. Improving self-care through education. D. All of the above."

"D. All of the above. Rationale: The principal goals of therapy for older persons with diabetes mellitus and poor glycemic control are enhancing quality of life, decreasing the chance of complications, improving self-care through education, and maintaining or improving general health status."

"A nurse shoud recognize which symptom as a cardinal sign of diabetes mellitus? "a. Nausea b. Seizure c. Hyperactivity d. Frequent urination

"D. Frequent Urination Polyphagia, polyuria, polydipsia, and weight loss are cardinal signs of DM. Other signs include irritability, shortened attention span, lowered frustration tolerance, fatigue, dry skin, blurred vision, sores that are slow to heal, and flushed skin."

"When an older adult is admitted to the hospital with a diagnosis of diabetes mellitus and complaints of rapid-onset weight loss, elevated blood glucose levels, and polyphagia, the gerontology nurse should anticipate which of the following secondary medical diagnoses? "1.Impaired glucose tolerance 2.Gestational diabetes mellitus 3.Pituitary tumor 4. Pancreatic tumor

"Pancreatic tumor Rationale: The onset of hyperglycemia in the older adult can occur more slowly. When the older adult reports rapid-onset weight loss, elevated blood glucose levels, and polyphagia, the healthcare provider should consider pancreatic tumor."

A 54-year-old patient admitted with type 2 diabetes, asks the nurse what "type 2" means. Which of the following is the most appropriate response by the nurse? "1. ""With type 2 diabetes, the body of the pancreas becomes inflamed." 2. "With type 2 diabetes, insulin secretion is decreased and insulin resistance is increased." 3. "With type 2 diabetes, the patient is totally dependent on an outside source of insulin." 4. "With type 2 diabetes, the body produces autoantibodies that destroy b-cells in the pancreas.""

"Right Answer: 2 Rationale: In type 2 diabetes mellitus, the secretion of insulin by the pancreas is reduced and/or the cells of the body become resistant to insulin"

Blood sugar is well controlled when Hemoglobin A1C is... "a. Below 7% b. Between 12%-15% c. Less than 180 mg/dL d. Between 90 and 130 mg/dL"

"a. Below 7% A1c measures the percentage of hemoglobin that is glycated and determines average blood glucose during the 2 to 3 months prior to testing. Used as a diagnostic tool, A1C levels of 6.5% or higher on two tests indicate diabetes. A1C of 6% to 6.5% is considered prediabetes."

"A client who is started on metformin and glyburide would have initially presented with which symptoms? "a. Polydispisa, polyuria, and weight loss b. weight gain, tiredness, & bradycardia c. irritability, diaphoresis, and tachycardia d. diarrhea, abdominal pain, and weight loss

"a. Polydispisa, polyuria, and weight loss"Symptoms of hyperglycemia include polydipsia, polyuria, and weight loss. Metformin and sulfonylureas are commonly ordered medications. Weight gain, tiredness, and bradycardia are symptoms of hypothyroidism. Irritability, diaphoresis, and tachycardia are symptoms of hypoglycemia. Symptoms of Crohn's disease include diarrhea, abdominal pain, and weight loss."

"(SELECT ALL THAT APPLY) A 45-year-old female client is admitted to the hospital with Cushing's syndrome. Which nursing interventions are appropriate for this client?

(1) Assess for peripheral edema (3) Measure intake and output (5) Weigh the client daily

"(SELECT ALL THAT APPLY) A client visiting the clinic is scheduled for an outpatient thyroid scan in 2 weeks. Which instructions should the nurse include in her client teaching to ensure that this client is prepared for the test?

(1) Stop using iodized salt or iodized salt substitutes 1 week before the scan. (2) Stop eating seafood 1 week before the scan. (4) Don't take any prescribed thyroid medication on the day of the scan.

"(SELECT ALL THAT APPLY) A client is diagnosed with a goiter after traveling in a foreign country for 3 months. During her trip, the client wasn't able to tolerate food. Which signs and symptoms would the nurse expect to see in this client?

(2) Dizziness when raising her arms above her head (3) Dysphagia (5) Respiratory distress

"(SELECT ALL THAT APPLY) A 48-year-old female client is seen in the clinic for newly diagnosed hypothyroidism. Which topics should the nurse include in a client teaching plan?

(2) High-fiber, low-calorie diet (4) Use of stool softeners (5) Thyroid hormone replacements

"(PUT IN ORDER) A client is ordered to receive 20 units of isophane insulin suspension (Humulin N) and 5 units of regular insulin (Humulin R) by subcutaneous injection. Place in chronological order the steps to take when mixing different types of insulin in a syringe. Use all the options.

(2) Inject 20 units of air into the Humulin N Vile (4) Withdrawl the syringe; don't withdrawl the insulin, (5) Inject 5 units of air into the Humulin R vile (1) Invert the vial and withdrawl the Humulin R Dose (3) Insert the syringe needle into the Humulin N vial (6) Invert the vial and withdral the Humulin N dose.

"(SELECT ALL THAT APPLY) A 56-year-old female client is being discharged after having a thyroidectomy. Which discharge instructions are appropriate for this client?

(2) Take thyroid replacement medication, as ordered. (3) Watch for changes in body functioning, such as lethargy, restlessness, sensitivity to cold, and dry skin. Report them to the physician.

"(SELECT ALL THAT APPLY) A client with type 2 diabetes mellitus needs instruction on proper foot care. Which instructions should the nurse include in client teaching?

(2) Wear cotton socks. (3) Apply foot powder after bathing. (5) See a podiatrist regularly to have your feet checked.

What question would the nurse ask a pt prior to an IVP?

***1. Are you allergic to shellfish? 2. do you have burning on urination? 3. have you ever had kidney stones? 4. why are you having this test?

632. The nurse is reviewing the laboratory results in a client with cirrhosis and notes that the ammonia level is elevated. Which diet does the nurse anticipate to be prescribed for this client?

***1. Low-protein diet 2. High-protein diet 3. Moderate-fat diet 4. High-carbohydrate diet

634. The client with hiatal hernia chronically experiences heartburn following meals. The nurse plans to teach the client to avoid which action because it is contraindicated with a hiatal hernia?

***1. Lying recumbent following meals 2. Taking in small, frequent, bland meals 3. Raising the head of bed on 6-inch blocks 4. Taking H2-receptor antagonist medication

629. The nurse is monitoring a client for the early signs and symptoms of dumping syndrome. Which of the following indicate this occurrence?

***1. Sweating and pallor 2. Bradycardia and indigestion 3. Double vision and chest pain 4. Abdominal cramping and pain

A pt hospitalized w/ sever ascites due to cirrhosis develops a fever and confusion. The nurse should do which of the following?

***1. auscultate bowel sounds and palpate for abdominal tenderness 2. Inquire about headache and check for nuchal rigidity 3. observe for neck vein distention and auscultate lung sounds 4. measure abdominal girth and percuss for shifting dullness

Following surgery, a pt has not voided for 12 hours. What assessment should the nurse make?

***1. palpate for bladder distention. 2. ausculatate for bowel sounds. 3. inspect for edema of the urethra 4. percuss for gastric tympany

A 58-year old woman presents at her primary care provider's office w/ symptoms of frequency, urgency, nocturia, dysuria, and cloudy, rust-colored urine for the thrid time in the past 2 years. Th nurse shold plan to include which of the following in her teaching for this patinet? Select all that paply.

***1. return to the office in 10 days for follow-up culture 2. preprocedure insturction for an IVP ***3. the potential benefits of estrogen vaginal cream ***4. recommendations for perineal cleansing 5. recommendations for screening cystoscopy

The physicain has ordered omeprazole 20 mg twice daily, clarithromycin 500 mg twice daily, and amoxicillin 1 g daiy for a pt w/ PUD. It is most important for the nurse to instruct the pt to do which of the following?

***1. stop the drugs immediately and notify the physician if a rash, hives, or itching develop. 2. consume 8 oz of yogurt or buttermilk daily while taking these drugs 3. take drugs on empty stomach, 1 hour before breakfast and at least 2 hours after dinner. 4. take drugs w/ full glass of water

"What insulin type can be given by IV? Select all that apply: "A. Glipizide (Glucotrol) B. Lispro (Humalog) C. NPH insulin D. Glargine (Lantus) E. Regular insulin

E) Regular insulinThe only insulin that can be given by IV is regular insulin.

How many doses of radioactive iodine can be given? What is the purpose? What precautions should be taken? What complication should be watched for?

1. The purpose is to destroy thyroid cells. Follow radioactive precautions: stay away from babies for 24 hours - don't kiss anyone for24 hrs. watch for thyroid storm.

"Which of the following is accurate pertaining to physical exercise and type 1 diabetes mellitus? "1. Physical exercise can slow the progression of diabetes mellitus. 2. Strenuous exercise is beneficial when the blood glucose is high. 3. Patients who take insulin and engage in strenuous physical exercise might experience hyperglycemia. 4. Adjusting insulin regimen allows for safe participation in all forms of exercise."

1) physical exercise can slow the progression of diabetes mellitusRationale: Physical exercise slows the progression of diabetes mellitus, because exercise has beneficial effects on carbohydrate metabolism and insulin sensitivity. Strenuous exercise can cause retinal damage, and can cause hypoglycemia. Insulin and foods both must be adjusted to allow safe participation in exercise.

A nurse is caring for a cient with type 1 diabetes mellitus. which client complaint would alert the nurse to the presence of a possible hypoglycemic reaction? "1. Tremors 2. Anorexia 3. Hot, dry skin 4. Muscle cramps

1) tremorsdecreased blood glucose levels produce autonomic nervous system symptoms, which are manifested classically as nervousness, irritability, and tremors. option 3 is more likely for hyperglycemia, and options 2 and 4 are unrelated to the signs of hypoglycemia.

"A client is admitted to the hospital with signs and symptoms of diabetes mellitus. Which findings is the nurse most likely to observe in this client? Select all that apply: "1. Excessive thirst 2. Weight gain 3. Constipation 4. Excessive hunger 5. Urine retention 6. Frequent, high-volume urination

1, 4, 6 Rationale: Classic signs of diabetes mellitus include polydipsia (excessive thirst), polyphagia (excessive hunger), and polyuria (excessive urination). Because the body is starving from the lack of glucose the cells are using for energy, the client has weight loss, not weight gain. Clients with diabetes mellitus usually don't present with constipation. Urine retention is only a problem is the patient has another renal-related condition.

648. A client with a peptic ulcer is diagnosed with a Helicobacter pylori infection. The nurse is teaching the client about the medications prescribed, including clarithromycin (Biaxin), esomeprazole (Nexium), and amoxicillin (Amoxil). Which statement by the client indicates the best understanding of the medication regimen?

1. "My ulcer will heal because these medications will kill the bacteria." 2. "These medications are only taken when I have pain from my ulcer." *** 3. "The medications will kill the bacteria and stop the acid production." 4. "These medications will coat the ulcer and decrease the acid production in my stomach."

The nurse identifies which of the follwing nursing diagnoses as highest priority for the pt admitted with PUD and possible perforation?

1. Acute Pain 2. Ineffective Health maintainence 3. Nausea ***4. Impaired Tissue Integrity: GI

A pt w/ a hx of PUD suddenly begins to complain of severe abd pain. The nurse should do which of the following? Select all that apply.

1. Administer the prescribed PPI. 2. Obtain an order for narcotic analgesic ***3. Withhold oral food and fluids. ***4. Place the pt in Fowler's position ***5. Notify the physician

When planning care for pt w/ stomatitis, the n urse identifies which of the following as a priority intervention?

1. Assist to cleanse mouth w/ mouthwash following meals. 2. allow pt to select appealing foods from a menu ***3. provide viscous lidocaine to relieve mouth pain before meals 4. refer to pt to smoking cessation program

626. The nurse is monitoring a client with a diagnosis of peptic ulcer. Which assessment finding would most likely indicate perforation of the ulcer?

1. Bradycardia 2. Numbness in the legs 3. Nausea and vomiting ***4. A rigid, board-like abdomen

620. The nurse is planning to teach the client with gastroesophageal reflux disease about substances that will increase the lower esophageal sphincter pressure. Which item should the nurse include on this list?

1. Coffee 2. Chocolate 3. Fatty foods ***4. Nonfat milk

A 23-year old woman presents to the urgency clinic w/ symptoms of a UTI. The nursing hx reveals that the pt was treated 3 months ago for a UTI. What additional question should the nurse ask?

1. Did you complete your antibiotic prescription for your first UTI? ***2. What form of birth control are you using? 3. does your partner have similar symptoms? 4. How much fluid do you drink each day?

631. The nurse is reviewing the record of a client with a diagnosis of cirrhosis and notes that there is documentation of the presence of asterixis. How should the nurse assess for its presence?

1. Dorsiflex the client's foot. 2. Measure the abdominal girth. ***3. Ask the client to extend the arms. 4. Instruct the client to lean forward.

"What will the nurse teach the client with diabetes regarding exercise in his or her treatment program? 1. During exercise the body will use carbohydrates for energy production, which in turn will decrease the need for insulin. 2. With an increase in activity, the body will use more carbohydrates; therefore more insulin will be required. 3. The increase in activity results in an increase in the use of insulin; therefore the client should decrease his or her carbohydrate intake. 4. Exercise will improve pancreatic circulation and stimulate the islets of Langerhans to increase the production of intrinsic insulin.

1. During exercise the body will use carbohydrates for energy production, which in turn will decrease the need for insuli"Rationale: As carbohydrates are used for energy, insulin needs decrease. Therefore during exercise, carbohydrate intake should be increased to cover the increased energy requirements. The beneficial effects of regular exercise may result in a decreased need for diabetic medications in order to reach target blood glucose levels. Furthermore, it may help to reduce triglycerides, LDL cholesterol levels, increase HDLs, reduce blood pressure, and improve circulation."

A pt hospitalized w/ cirrhosis, ascites, and mild hepatic encephalopathy suddenly vomits 200 mL of bright red blood. which of the following should the nurse do first?

1. Insert a NG tube ***2. Place in Fowler's position 3. contact the physician 4. check stool for occult blood

At a local health fair, a man remarks to the nurse that his urine occasionally appears pink. He ownders if this is anything to be concerned about. How should the nurse respond?

1. Instruct the man to notify his physician if he develops pain or difficulty voiding. ***2. advise the man to make an appointment to see his physician. 3. instruct the man to track the relationship between urine color and his activities. 4. Tell the man to increase his fluid intake to 2 1/2 to 3 quarts per day.

621. The client has undergone esophagogastroduodenoscopy. The nurse places highest priority on which item as part of the client's care plan?

1. Monitoring the temperature 2. Monitoring complaints of heartburn 3. Giving warm gargles for a sore throat ***4. Assessing for the return of the gag reflex

646. The client who chronically uses nonsteroidal anti-inflammatory drugs (NSAIDs) has been taking misoprostol (Cytotec). The nurse determines that the medication is having the intended therapeutic effect if which of the following is noted?

1. Resolved diarrhea ***2. Relief of epigastric pain 3. Decreased platelet count 4. Decreased white blood cell count

633. The nurse is doing an admission assessment on a client with a history of duodenal ulcer. To determine whether the problem is currently active, the nurse should assess the client for which symptom(s) of duodenal ulcer?

1. Weight loss 2. Nausea and vomiting ***3. Pain relieved by food intake 4. Pain radiating down the right arm

645. The client with a gastric ulcer has a prescription for sucralfate (Carafate), 1 g by mouth four times daily. The nurse schedules the medication for which times?

1. With meals and at bedtime 2. Every 6 hours around the clock 3. One hour after meals and at bedtime ***4. One hour before meals and at bedtime

Recognizing the risk for urolithiasis in the immobilized pt, the nurse apprpriately plans to do which of the following?

1. administer a Ca supplement 2. regularly monitor urine pH 3. maintain an indwelling urinary cath ***4. increase fluid intake to 3000 mL per day

A nurse is caring for a man who has returned to the unit from the recovery room following a TURP. His urinary drainage bad is filled with dark red fluid w/ obvious clots. He is having painful bladder spasms. What would the nurse do first?

1. assess his I&O since surgery 2. administer pain medication in the form of a B&O suppository ***3. report your assessments to his urologist 4. nothing, because these manifestations are espected following a TURP

The nurse caring for a pt scheduled for an abdominal paracentesis instructs the pt to do which of the following?

1. avoid eating or drinking fluid for 6 hrs prior to the procedure 2. scrub the abd w/ antiseptic soap before the procedure ***3. empty the bladder before the procedure 4. report excess flatus following the procedure to the physician

The nurse evaluates his teaching of a pt w/ acute stress gastritis as effective when the pt states that sh will do which of the following?

1. avoid using aspirin or NSAIDs for routine pain reliefe

The enlarging prostate in BPH typically is manifested by assessment of problems w/ which of the following?

1. bowel eleimination ***2. urinary eliminatin 3. peripheral vascular function 4. skin integrity

Before beginning the physical assessment of the renal system, then urse should ask the pt to do which of the following?

1. empty the bladder 2. takd several deep breaths ***3. provide a urine specimen 4. drink several glassess of water

what are the four things glucocorticoids do?

1. mood changer (insomnia, depression, psychotic and euphoric) 2. alter defense mechanism - immunosuppressed (remember sugar likes bacteria - diabetics have lots of uti's) high risk for infection (don't put with contageous pt) 3. Breakdown fats and proteins (think this could make them metabolic acidodic with DKA - the body is starving for glucose so it breaks down fats and proteins which produce keytones - the only way to get rid of these is through exhaling / exersize - increase resp.) (there is a note also of small arms and legs with big belly for breakdown of fats and proteins) 4. inhibit insulin - hyperglycemic and do blood glucose monitoring

A pt admitted w/ possible kidney stones develops sudden complaints of acute crampy pain on the left side that radiates into the groin. He is nauseated, and vomits clear fluid. On voiding, his urine is pink. The most appropriate response by the nurse is which of the following?

1. obtain a bladder scan to assess for residual urine 2. adminster the rpescribed narcotic analgesic ***3. notify the physician 4. strain all urine

The nurse caring for a pt preparing to under gemodialysis includes which of the following in the plan of care? select all that apply

1. obtain wt and orthostatic vital signs 3. monitor serum creatinine, BUN, and hematocrit levels

What diagnostic tests are used to differentiate BPH from prostate cancer? Slect all that apply.

1. pelvic u/s ***2. digital rectal exam 3. blood chemistry ***4. PSA level 5. sperm count

Which of the following nursing interventions are of highest priority when caring for a pt w/ small bowel obstruction?

1. placing the pt in semi-Fowler's position ***2. maintaining NG sunction 3. keeping strict I&O records 4. administering prescribed analgesics

During a health hx interview, a pt tells the nurse that she has to get up to void several times a night. This finding is documented as which of the following?

1. polyuria ***2. nocturia 3. dysuria 4. hematuria

What diagnostic test can be used to determine GFR as well as glomerular damage?

1. routine urinalysis 2. renal scan ***3. creatinine clearance 4. renal biopsy

Why should a trach be set up after a thyroidectomy (3)?

1.) Hypocalcemia - no PTH > serum Ca decreases > decreased sedative properties > check chovsteks/trosseaus 2.) swallowing problems 3.) laryngeal nerve damage

What are the 4 functions of glucocorticoids?

1.) change your mood 2.) Alter defense mechanisms (by decreases immune system) 3.) breaksdown protein/fat 4.) inhibits insulin (start doing accuchecks even if not diabetic).

"An agitated, confused client arrives in the emergency department. The client's history includes type 1 diabetes, hypertension, and angina pectoris. Assessment reveals pallor, diaphoresis, headache, and intense hunger. A stat blood glucose sample measures 42 mg/dl, and the client is treated for an acute hypoglycemic reaction. After recovery, the nurse teaches the client to treat hypoglycemia by ingesting:

10 to 15 g of a simple carbohydrate.

"The nurse is caring for a client with type 1 diabetes who exhibits confusion, light-headedness, and aberrant behavior. The client is still conscious. The nurse should first administer:

15 to 20 g of a fast-acting carbohydrate such as orange juice.

A nurse is preparing a teaching plan for a client with diabetes Mellitus regarding proper foot care. Which instruction is included in the plan? 1. Soak feet in hot water 2. apply a moisturizing lotion to dry feet but not between the toes 3. Always have a podiatrist cut your toenails, never cut them yourself 4. avoid using mild soap on the feet

2. The client is instructed to use a moisturizing lotion on the feet and to avoid applying the lotion between the toes.

"The nurse administered 28 units of Humulin N, an intermediate-acting insulin, to a client diagnosed with Type 1 diabetes at 1600. Which action should the nurse implement? "1. Ensure the client eats the bedtime snack. 2. Determine how much food the client ate at lunch. 3. Perform a glucometer reading at 0700. 4. Offer the client protein after administering insulin.

1: ensure the client eats the bedtime snack"1. Humulin N peaks in 6-8 hours, making the client at risk for hypoglycemia around midnight, which is why the client should receive a bedtime snack. This snack will prevent nighttime hypoglycemia. (Correct) 2. The food intake at lunch will not affect the client's blood glucose level at midnight. 3. The client's glucometer reading should be done around 2100 to assess the effectiveness of insulin at 1600. 4. Humulin N is an intermediate-acting insulin that has an onset in 2-4 hours but does not peak until 6-8 hours."

Rule: When on a medication that fluctuates weight and weight has to be monitored - keep weight within ________ lbs of thier normal weight.

2 - 3 lbs - WEIGHT is very important in adjusting medication

A nurse is preparing a plan of care for a client with diabetes mellitus who has hyerglycemia. The priority nursing diagnosis would be: 1. Deficient knowledge 2. Deficient fluid volume 3. Compromised family coping 4. Imbalanced nutrition less than body requirements

2) deficient fluid volumeAn increased blood glucose level will cause the kidneys to excrete the glucose in the urine. This glucose is accompanied by fluids and electrolytes, causing an osmotic diuresis leading to dehydration. This fluid loss must be replaced when it becomes severe.

The nurse teaching a pt. w/ GERD includes which of the following instructions? Select all that apply

2. Elevate the head of the bed on 6-8 inch blocks 5. avoid lying down for several hours after eating

"The nurse is discharging a client diagnosed with diabetes insipidus. Which statementmade by the client warrants further intervention? "1."I will keep a list of my medications in my wallet and wear a Medi bracelet." 2."I should take my medication in the morning and leave it refrigerated at home." 3."I should weigh myself every morning and record any weight gain." 4."If I develop a tightness in my chest, I will call my health-care provider."

2."I should take my medication in the morning and leave it refrigerated at home.""1.The client should keep a list of medication being taken and wear a Medic Alert bracelet. 2. Medication taken for DI is usually every 8-12 hours, depending on the client. Theclient should keep the medication close at hand. 3.The client is at risk for fluid shifts. Weighing every morning allows the client to follow thefluid shifts. Weight gain could indicate too much medication. 4.Tightness in the chest could be an indicator that the medication is not being tolerated; if this occurs the client should call the health-care provider"

"A client with Hashimoto's thyroiditis and a history of two myocardial infarctions and coronary artery disease is to receive levothyroxine (Synthroid). Because of the client's cardiac history, the nurse would expect that the client's initial dose for the thyroid replacement would be:

25 mcg/day.

"The client diagnosed with type 1 diabetse is receiving Humalog, a rapid-acting insulin, by sliding scale. The order reads blood glucose level: <150, zero (0) units; 151 to 200, three (3) units; 201 to 250, six (6 units); >251, contact health-care provider. The unlicensed assistive personnel (UAP) reports to thenurse the client's glucometer reading is 189. How much insulin should the nurse administer to the client?

3 unitsThe client's result is 189, which is between 151 and 200, so the nurse should administer 3 units of Humalog insulin subcutaneously.

The nurse is caring for a client with long-term Type 2 diabetes and is assessing the feet. Which assessment data would warrant immediate intervention by the nurse? "1)The client has crumbling toenails 2)The client has athlete's feet 3)The client has a necrotic big toe 4)The client has thickened toenails."

3) Nectrotic big toe"1)Crumbling toenails indicate tinea unguium, which is a fungus infection of the toenail. 2)Athlete's foot is a fungal infection that is not life threatening. 3)A necrotic big toe indicates "dead" tissue. The client does not feel pain in the lower extremity and does not realize there has been an injury and therefore does not seek treatment. Increased blood glucose levels decrease oxygen supply that is needed to heal the wound and increase the risk for developing an infection. 4)Big, thick toenails are fungal infections and would not require immediate intervention by the nurse; 50% of the adult population has this."

An adult with a diagnosis of hpothyroidism has been prescribed thyroid replacement therapy with levothyroxin. After 2 days, the client calls to complain that she feels no better. The nurse's response should be based on the fact that

Effect of medication are not seen or felt ofr about 3 to 5 days.

"A nurse performs a physical assessment on a client with type 2 diabetes mellitus. Findings include a fasting blood glucose of 120 mg/dL, temp of 101 F, pulse of 88 bpm, respirations of 22, and blood pressure of 100/72. Which finding would be of most concern to the nurse? "1. Pulse 2. Respiration 3. Temperature 4. Blood pressure"

3) temp. An elevated temperature may indicate infection. Infection is a leading cause of hyperglycemic hyperosmolar nonketotic syndrome or diabetic ketoacidosis. The other findings noted in the question are within normal limits.

"A client with newly diagnosed type 2 diabetes mellitus is admitted to the metabolic unit. The primary goal for this admission is education. Which of the following goals should the nurse incorporate into her teaching plan?

Exercise and a weight reduction diet

A client with type I diabetes is placed on an insulin pump. The most appropriate short-term goal when teaching this client to control the diabetes is: "1) adhere to the medical regimen 2) remain normoglycemic for 3 weeks 3) demonstrate the correct use of the administration equipment. 4) list 3 self care activities that are necessary to control the diabetes"

3.) is correct "1) this is not a short-term goal 2) this is measurable, but it's a long-term goal 3) this is a short-term goal, client oriented, necessary for the client to control the diabetes, and measurable when the client performs a return demonstration for the nurse 4) although this is measurable and a short-term goal, it is not the one with the greatest priority when a client has an insulin pump that must be mastered before discharge"

"The nurse is teaching a class on atherosclerosis. Which statement describes the scien-tific rationale as to why diabetes is a risk factor for developing atherosclerosis? (1.Glucose combines with carbon monoxide, instead of with oxygen, and this leads tooxygen deprivation of tissues.2.Diabetes stimulates the sympathetic nervous system, resulting in peripheralconstriction that increases the development of atherosclerosis.3.Diabetes speeds the atherosclerotic process by thickening the basement membraneof both large and small vessels.4.The increased glucose combines with the hemoglobin, which causes deposits of plaque in the lining of the vessels.

3.Diabetes speeds the atherosclerotic process by thickening the basement membraneof both large and small vessels."1.Glucose does not combine with carbonmonoxide.2.Vasoconstriction is not a risk factor for devel-oping atherosclerosis. 3.This is the scientific rationale why diabetesmellitus is a modifiable risk factor for atherosclerosis. 4.When glucose combines with the hemoglobinin a laboratory test called glycosylated hemo-globin, the result can determine the client'saverage glucose level over the past three (3)months"

"After a 3-month trial of dietary therapy, a client with type 2 diabetes still has blood glucose levels above 180 mg/dl. The physician adds glyburide (DiaBeta), 2.5 mg P.O. daily, to the treatment regimen. The nurse should instruct the client to take glyburide:

30 minutes before breakfast.

"The nurse administered isophane insulin suspension (NPH) to a diabetic client at 7 a.m. At what time would the nurse expect the client to be most at risk for a hypoglycemic reaction?

4 p.m.

Of which of the following symptoms might an older woman with diabetes mellitus complain? wps.prenhall.com 1) anorexia 2)pain intolerance 3) weight loss 4) perineal itching

4) perineal itchingRationale: Older women might complain of perineal itching due to vaginal candidiasis.

A client with diabetes mellitus demonstratees acute anxiety when first admitted for the treatment of hyperglycemia. The most appropriate intervention to decrease the client's anxiety would be to 1. administer a sedative 2. make sure the client knows all the correct medical terms to understand what is happening 3. ignore the signs and symptoms of anxiety so that they will soon disappear 4. convey empathy, trust, and respect toward the client

4. The most appropriate intervention is to address the client's feelings related to the anxiety

a nurse is interviewing a client with type 2 diabetes mellitus. which statement by the client indicated an understanding of the treatment for this disorder? "1. ""i take oral insulin instead of shots"" 2. ""by taking these medications I am able to eat more"" 3. ""when I become ill, I need to increase the number of pills I take"" 4. ""the medications I'm taking help release the insulin I already make""

4.)Clients with type 2 diabetes mellitus have decreased or imparied insulin secretion. Oral hypoglycemic agents are given to these clients to facilitate glucose uptake. Insulin injections may be given during times of stress-induced hyperglycemia. Oral insulin is not available because of the breakdown of the insulin by digestion. Options 1, 2 and 3 are incorrect

"The nurse is discussing the importance of exercising to a client diagnosed with Type 2diabetes whose diabetes is well controlled with diet and exercise. Which informationshould the nurse include in the teaching about diabetes? "1.Eat a simple carbohydrate snack before exercising. 2.Carry peanut butter crackers when exercising. 3.Encourage the client to walk 20 minutes three (3) times a week. 4.Perform warmup and cooldown exercises

4.Perform warmup and cooldown exercises "The client diagnosed with Type 2 diabetes whois not taking insulin or oral agents does notneed extra food before exercise.2.The client with diabetes who is at risk forhypoglycemia when exercising should carry asimple carbohydrate, but this client is not atrisk for hypoglycemia.3.Clients with diabetes that is controlled by dietand exercise must exercise daily at the sametime and in the same amount to control theglucose level. 4. [correct] All clients who exercise should perform warmup and cooldown exercises to helpprevent muscle strain and injury"

"The nurse teaches a diabetic client that diet plays a crucial role in managing diabetes mellitus. When evaluating dietary intake, the nurse knows the client is eating the right foods if total daily caloric intake consists of:

55% to 60% carbohydrate, 30% fat, and 10% to 15% protein.

"Every morning a client with type 1 diabetes receives 15 units of Humulin 70/30. What does this type of insulin contain?

70% NPH insulin and 30% regular insulin

How long will hypothyroid patients be taking meds such as Levothyroxine (Synthroid), Thyroglobulin (Proloid), & Liothyronin (Cytomel)?

FOR LIFE!

Hyperthyroidism - (T/F) Beta-blockers can be given to asthmatics or diabetics.

False.

A 63-year-old patient is newly diagnosed with type 2 diabetes. When developing an education plan, the nurse's first action should be to a. assess the patient's perception of what it means to have type 2 diabetes. b. demonstrate how to check glucose using capillary blood glucose monitoring. c. ask the patient's family to participate in the diabetes education program. d. discuss the need for the patient to actively participate in diabetes management.

A Rationale: Before planning education, the nurse should assess the patient's interest in and ability to self-manage the diabetes. After assessing the patient, the other nursing actions may be appropriate, but planning needs to be individualized to each patient.

A patient with type 2 diabetes that is controlled with diet and metformin (Glucophage) also has severe rheumatoid arthritis (RA). During an acute exacerbation of the patient's arthritis, the health care provider prescribes prednisone (Deltasone) to control inflammation. The nurse will anticipate that the patient may a. require administration of insulin while taking prednisone. b. develop acute hypoglycemia during the RA exacerbation. c. have rashes caused by metformin-prednisone interactions. d. need a diet higher in calories while receiving prednisone.

A Rationale: Glucose levels increase when patients are taking CORTICOsteroids, and insulin may be required to control blood glucose. Hypoglycemia is not a complication of RA exacerbation or prednisone use. Rashes are not an adverse effect caused by taking metformin and prednisone simultaneously. The patient is likely to have an increased appetite when taking prednisone, but it will be important to avoid weight gain for the patient with RA.

Intramuscular glucagon is administered to an unresponsive patient for treatment of hypoglycemia. Which action should the nurse take after the patient regains consciousness? a. Give the patient a snack of cheese and crackers. b. Have the patient drink a glass of orange juice or nonfat milk. c. Administer a continuous infusion of 5% dextrose for 24 hours. d. Assess the patient for symptoms of hyperglycemia.

A Rationale: Rebound hypoglycemia can occur after glucagon administration, but having a meal containing complex carbohydrates plus protein and fat will help prevent hypoglycemia. Orange juice and nonfat milk will elevate blood sugar rapidly, but the cheese and crackers will stabilize blood sugar. Administration of glucose intravenously might be used in patients who were unable to take in nutrition orally. The patient should be assessed for symptoms of hypoglycemia after glucagon administration.

Cardiac monitoring is initiated for a patient in diabetic ketoacidosis (DKA). The nurse recognizes that this measure is important to identify a. electrocardiographic (ECG) changes and dysrhythmias related to hypokalemia. b. fluid overload resulting from aggressive fluid replacement. c. the presence of hypovolemic shock related to osmotic diuresis. d. cardiovascular collapse resulting from the effects of hyperglycemia.

A Rationale: The hypokalemia associated with metabolic acidosis can lead to potentially fatal dysrhythmias such as ventricular tachycardia and ventricular fibrillation, which would be detected with ECG monitoring. Fluid overload, hypovolemia, and cardiovascular collapse are possible complications of DKA, but cardiac monitoring would not detect theses.

Corticosteroid

Florinef (fludrocortsone)-prevents the release of substances in the body taht cause inflamation. Used to treat conditions in which the body does not produce enough of its own steroids such as addisons disease, help restore electrolyte balance.

A patient with type 1 diabetes has been using self-monitoring of blood glucose (SMBG) as part of diabetes management. During evaluation of the patient's technique of SMBG, the nurse identifies a need for additional teaching when the patient a. chooses a puncture site in the center of the finger pad. b. washes the puncture site using soap and water. c. says the result of 130 mg indicates good blood sugar control. d. hangs the arm down for a minute before puncturing the site.

A Rationale: The patient is taught to choose a puncture site at the side of the finger pad. The other patient actions indicate that teaching has been effective.

A patient with cancer of the liver has severe ascites, and the health care provider plans a paracentesis to relieve the fluid pressure on the diaphragm. To prepare the patient for the procedure, the nurse a. asks the patient to empty the bladder. b. positions the patient on the right side. c. obtains informed consent for the procedure. d. assists the patient to lie flat in bed.

A Rationale: The patient should empty the bladder to decrease the risk of bladder perforation during the procedure. The patient would be positioned in Fowler's position and would not be able to lie flat without compromising breathing. The health care provider is responsible for obtaining informed consent.

While hospitalized and recovering from an episode of diabetic ketoacidosis, the patient calls the nurse and reports feeling anxious, nervous, and sweaty. Based on the patient's report, the nurse should a. obtain a glucose reading using a finger stick. b. administer 1 mg glucagon subcutaneously. c. have the patient eat a candy bar. d. have the patient drink 4 ounces of orange juice.

A Rationale: The patient's clinical manifestations are consistent with hypoglycemia and the initial action should be to check the patient's glucose with a finger stick or order a stat blood glucose. If the glucose is low, the patient should ingest a rapid-acting carbohydrate, such as orange juice. Glucagon might be given if the patient's symptoms become worse or if the patient is unconscious. Candy bars contain fat, which would slow down the absorption of sugar and delay the response to treatment.

When assessing the patient experiencing the onset of symptoms of type 1 diabetes, which question should the nurse ask? a. "Have you lost any weight lately?" b. "Do you crave fluids containing sugar?" c. "How long have you felt anorexic?" d. "Is your urine unusually dark-colored?"

A Rationale: Weight loss occurs because the body is no longer able to absorb glucose and starts to break down protein and fat for energy. The patient is thirsty but does not necessarily crave sugar- containing fluids. Increased appetite is a classic symptom of type 1 diabetes. With the classic symptom of polyuria, urine will be very dilute.

A client has recently been diagnosed with Type I diabetes and asks the nurse for help formulating a nutrition plan. Which of the following recommendations would the nurse make to help the client increase calorie consumption to offset absorption problems? a. Eating small meals with two or three snacks may be more helpful in maintaining blood glucose levels than three large meals. b. Eat small meals with two or three snacks throughout the day to keep blood glucose levels steady c. Increase consumption of simple carbohydrates d. Skip meals to help lose weight

A Eating small meals with two or three snacks may be more helpful in maintaining blood glucose levels than three large meals.

21. A 63-year-old patient is newly diagnosed with type 2 diabetes. When developing an education plan, the nurse's first action should be to a. assess the patient's perception of what it means to have type 2 diabetes. b. demonstrate how to check glucose using capillary blood glucose monitoring. c. ask the patient's family to participate in the diabetes education program. d. discuss the need for the patient to actively participate in diabetes management.

A Rationale: Before planning education, the nurse should assess the patient's interest in and ability to self-manage the diabetes. After assessing the patient, the other nursing actions may be appropriate, but planning needs to be individualized to each patient. Cognitive Level: Application Text Reference: p.1264 Nursing Process: Planning NCLEX: Health Promotion and Maintenance

A patient in the outpatient clinic has positive serologic testing for anti-HCV. Which action by the nurse is appropriate? a. Schedule the patient for HCV genotype testing. b. Teach the patient that the HCV will resolve in 2 to 4 months. c. Administer immune globulin and the HCV vaccine. d. Instruct the patient on self-administration of -interferon.

A Rationale: Genotyping of HCV has an important role in managing treatment and is done before drug therapy with -interferon or other medications is started. HCV has a high percentage of conversion to the chronic state so the nurse should not teach the patient that the HCV will resolve in 2 to 4 months. Immune globulin or vaccine is not available for HCV.

18. A patient with type 2 diabetes that is controlled with diet and metformin (Glucophage) also has severe rheumatoid arthritis (RA). During an acute exacerbation of the patient's arthritis, the health care provider prescribes prednisone (Deltasone) to control inflammation. The nurse will anticipate that the patient may a. require administration of insulin while taking prednisone. b. develop acute hypoglycemia during the RA exacerbation. c. have rashes caused by metformin-prednisone interactions. d. need a diet higher in calories while receiving prednisone.

A Rationale: Glucose levels increase when patients are taking corticosteroids, and insulin may be required to control blood glucose. Hypoglycemia is not a complication of RA exacerbation or prednisone use. Rashes are not an adverse effect caused by taking metformin and prednisone simultaneously. The patient is likely to have an increased appetite when taking prednisone, but it will be important to avoid weight gain for the patient with RA. Cognitive Level: Application Text Reference: pp. 1258, 1267 Nursing Process: Planning NCLEX: Physiological Integrity

After the home health nurse has taught a patient and family about how to use glargine and regular insulin safely, which action by the patient indicates that the teaching has been successful? a. The patient disposes of the open insulin vials after 4 weeks. b. The patient draws up the regular insulin in the syringe and then draws up the glargine. c. The patient stores extra vials of both types of insulin in the freezer until needed. d. The patient's family prefills the syringes weekly and stores them in the refrigerator.

A Rationale: Insulin can be stored at room temperature for 4 weeks. Glargine should not be mixed with other insulins or prefilled and stored. Freezing alters the insulin molecule and should not be done.

36. After the home health nurse has taught a patient and family about how to use glargine and regular insulin safely, which action by the patient indicates that the teaching has been successful? a. The patient disposes of the open insulin vials after 4 weeks. b. The patient draws up the regular insulin in the syringe and then draws up the glargine. c. The patient stores extra vials of both types of insulin in the freezer until needed. d. The patient's family prefills the syringes weekly and stores them in the refrigerator.

A Rationale: Insulin can be stored at room temperature for 4 weeks. Glargine should not be mixed with other insulins or prefilled and stored. Freezing alters the insulin molecule and should not be done. Cognitive Level: Application Text Reference: p. 1261 Nursing Process: Evaluation NCLEX: Physiological Integrity

The nurse identifies the collaborative problem of potential complication: electrolyte imbalance for a patient with severe acute pancreatitis. Assessment findings that alert the nurse to electrolyte imbalances associated with acute pancreatitis include a. muscle twitching and finger numbness. b. paralytic ileus and abdominal distention. c. hypotension. d. hyperglycemia.

A Rationale: Muscle twitching and finger numbness indicate hypocalcemia, a potential complication of acute pancreatitis. The other data indicate other complications of acute pancreatitis but are not indicators of electrolyte imbalance.

A patient using a split mixed-dose insulin regimen asks the nurse about the use of intensive insulin therapy to achieve tighter glucose control. The nurse should teach the patient that a. intensive insulin therapy requires three or more injections a day in addition to an injection of a basal long-acting insulin. b. intensive insulin therapy is indicated only for newly diagnosed type 1 diabetics who have never experienced ketoacidosis. c. studies have shown that intensive insulin therapy is most effective in preventing the macrovascular complications characteristic of type 2 diabetes. d. an insulin pump provides the best glucose control and requires about the same amount of attention as intensive insulin therapy.

A Rationale: Patients using intensive insulin therapy must check their glucose level four to six times daily and administer insulin accordingly. A previous episode of ketoacidosis is not a contraindication for intensive insulin therapy. Intensive insulin therapy is not confined to type 2 diabetics and would prevent microvascular changes as well as macrovascular changes. Intensive insulin therapy and an insulin pump are comparable in glucose control.

14. A patient using a split mixed-dose insulin regimen asks the nurse about the use of intensive insulin therapy to achieve tighter glucose control. The nurse should teach the patient that a. intensive insulin therapy requires three or more injections a day in addition to an injection of a basal long-acting insulin. b. intensive insulin therapy is indicated only for newly diagnosed type 1 diabetics who have never experienced ketoacidosis. c. studies have shown that intensive insulin therapy is most effective in preventing the macrovascular complications characteristic of type 2 diabetes. d. an insulin pump provides the best glucose control and requires about the same amount of attention as intensive insulin therapy.

A Rationale: Patients using intensive insulin therapy must check their glucose level four to six times daily and administer insulin accordingly. A previous episode of ketoacidosis is not a contraindication for intensive insulin therapy. Intensive insulin therapy is not confined to type 2 diabetics and would prevent microvascular changes as well as macrovascular changes. Intensive insulin therapy and an insulin pump are comparable in glucose control. Cognitive Level: Application Text Reference: p. 1263 Nursing Process: Implementation NCLEX: Physiological Integrity

28. Intramuscular glucagon is administered to an unresponsive patient for treatment of hypoglycemia. Which action should the nurse take after the patient regains consciousness? a. Give the patient a snack of cheese and crackers. b. Have the patient drink a glass of orange juice or nonfat milk. c. Administer a continuous infusion of 5% dextrose for 24 hours. d. Assess the patient for symptoms of hyperglycemia.

A Rationale: Rebound hypoglycemia can occur after glucagon administration, but having a meal containing complex carbohydrates plus protein and fat will help prevent hypoglycemia. Orange juice and nonfat milk will elevate blood sugar rapidly, but the cheese and crackers will stabilize blood sugar. Administration of glucose intravenously might be used in patients who were unable to take in nutrition orally. The patient should be assessed for symptoms of hypoglycemia after glucagon administration. Cognitive Level: Application Text Reference: p. 1282 Nursing Process: Implementation NCLEX: Physiological Integrity

11. The nurse has been teaching the patient to administer a dose of 10 units of regular insulin and 28 units of NPH insulin. The statement by the patient that indicates a need for additional instruction is, a. "I need to rotate injection sites among my arms, legs, and abdomen each day." b. "I will buy the 0.5-ml syringes because the line markings will be easier to see." c. "I should draw up the regular insulin first after injecting air into the NPH bottle." d. "I do not need to aspirate the plunger to check for blood before I inject the insulin."

A Rationale: Rotating sites is no longer necessary because all insulin is now purified human insulin, and the risk for lipodystrophy is low. The other patient statements are accurate and indicate that no additional instruction is needed. Cognitive Level: Application Text Reference: p. 1262 Nursing Process: Evaluation NCLEX: Health Promotion and Maintenance

A homeless patient with severe anorexia, fatigue, jaundice, and hepatomegaly is diagnosed with viral hepatitis and has just been admitted to the hospital. In planning care for the patient, the nurse assigns the highest priority to the patient outcome of a. maintaining adequate nutrition. b. establishing a stable home environment. c. increasing activity level. d. identifying the source of exposure to hepatitis.

A Rationale: The highest priority outcome is to maintain nutrition because adequate nutrition is needed for hepatocyte regeneration. Finding a home for the patient and identifying the source of the infection would be appropriate activities, but they do not have as high a priority as having adequate nutrition. Although the patient's activity level will be gradually increased, rest is indicated during the acute phase of hepatitis.

22. Cardiac monitoring is initiated for a patient in diabetic ketoacidosis (DKA). The nurse recognizes that this measure is important to identify a. electrocardiographic (ECG) changes and dysrhythmias related to hypokalemia. b. fluid overload resulting from aggressive fluid replacement. c. the presence of hypovolemic shock related to osmotic diuresis. d. cardiovascular collapse resulting from the effects of hyperglycemia.

A Rationale: The hypokalemia associated with metabolic acidosis can lead to potentially fatal dysrhythmias such as ventricular tachycardia and ventricular fibrillation, which would be detected with ECG monitoring. Fluid overload, hypovolemia, and cardiovascular collapse are possible complications of DKA, but cardiac monitoring would not detect theses. Cognitive Level: Application Text Reference: p. 1281 Nursing Process: Assessment NCLEX: Physiological Integrity

20. A patient with type 1 diabetes has been using self-monitoring of blood glucose (SMBG) as part of diabetes management. During evaluation of the patient's technique of SMBG, the nurse identifies a need for additional teaching when the patient a. chooses a puncture site in the center of the finger pad. b. washes the puncture site using soap and water. c. says the result of 130 mg indicates good blood sugar control. d. hangs the arm down for a minute before puncturing the site.

A Rationale: The patient is taught to choose a puncture site at the side of the finger pad. The other patient actions indicate that teaching has been effective. Cognitive Level: Application Text Reference: p. 1270 Nursing Process: Evaluation NCLEX: Health Promotion and Maintenance

17. When teaching a patient with type 2 diabetes about taking glipizide (Glucotrol), the nurse determines that additional teaching about the medication is needed when the patient says, a. "Since I can take oral drugs rather than insulin, my diabetes is not serious and won't cause many complications." b. "If I overeat at a meal, I will still take just the usual dose of medication." c. "If I become ill, I may have to take insulin to control my blood sugar." d. "I should check with my doctor before taking any other medications because there are many that will affect glucose levels."

A Rationale: The patient should understand that type 2 diabetes places the patient at risk for many complications and that good glucose control is as important when taking oral agents as when using insulin. The other statements are accurate and indicate good understanding of the use of glipizide. Cognitive Level: Application Text Reference: p. 1275 Nursing Process: Evaluation NCLEX: Health Promotion and Maintenance

26. While hospitalized and recovering from an episode of diabetic ketoacidosis, the patient calls the nurse and reports feeling anxious, nervous, and sweaty. Based on the patient's report, the nurse should a. obtain a glucose reading using a finger stick. b. administer 1 mg glucagon subcutaneously. c. have the patient eat a candy bar. d. have the patient drink 4 ounces of orange juice.

A Rationale: The patient's clinical manifestations are consistent with hypoglycemia and the initial action should be to check the patient's glucose with a finger stick or order a stat blood glucose. If the glucose is low, the patient should ingest a rapid-acting carbohydrate, such as orange juice. Glucagon might be given if the patient's symptoms become worse or if the patient is unconscious. Candy bars contain fat, which would slow down the absorption of sugar and delay the response to treatment. Cognitive Level: Application Text Reference: p. 1282 Nursing Process: Implementation NCLEX: Physiological Integrity

16. Glyburide (Micronase, DiaBeta, Glynase) is prescribed for a patient whose type 2 diabetes has not been controlled with diet and exercise. When teaching the patient about glyburide, the nurse explains that a. glyburide stimulates insulin production and release from the pancreas. b. the patient should not take glyburide for 48 hours after receiving IV contrast media. c. glyburide should be taken even when the blood glucose level is low in the morning. d. glyburide decreases glucagon secretion.

A Rationale: The sulfonylureas stimulate the production and release of insulin from the pancreas. If the glucose level is low, the patient should contact the health care provider before taking the glyburide, since hypoglycemia can occur with this category of medication. Metformin should be held for 48 hours after administration of IV contract, but this is not necessary for glyburide. Cognitive Level: Application Text Reference: pp. 1265-1266 Nursing Process: Implementation NCLEX: Physiological Integrity

Amitriptyline (Elavil) is prescribed for a diabetic patient with peripheral neuropathy who has burning foot pain occurring mostly at night. Which information should the nurse include when teaching the patient about the new medication? a. Amitriptyline will help prevent the transmission of pain impulses to the brain. b. Amitriptyline will improve sleep and make you less aware of nighttime pain. c. Amitriptyline will decrease the depression caused by the pain. d. Amitriptyline will correct some of the blood vessel changes that cause pain.

A Rationale: Tricyclic antidepressants decrease the transmission of pain impulses to the spinal cord and brain. Tricyclics also improve sleep quality and are used for depression, but that is not the major purpose for their use in diabetic neuropathy. The blood vessel changes that contribute to neuropathy are not affected by tricyclics.

32. Amitriptyline (Elavil) is prescribed for a diabetic patient with peripheral neuropathy who has burning foot pain occurring mostly at night. Which information should the nurse include when teaching the patient about the new medication? a. Amitriptyline will help prevent the transmission of pain impulses to the brain. b. Amitriptyline will improve sleep and make you less aware of nighttime pain. c. Amitriptyline will decrease the depression caused by the pain. d. Amitriptyline will correct some of the blood vessel changes that cause pain.

A Rationale: Tricyclic antidepressants decrease the transmission of pain impulses to the spinal cord and brain. Tricyclics also improve sleep quality and are used for depression, but that is not the major purpose for their use in diabetic neuropathy. The blood vessel changes that contribute to neuropathy are not affected by tricyclics. Cognitive Level: Application Text Reference: p. 1285 Nursing Process: Implementation NCLEX: Physiological Integrity

5. When assessing the patient experiencing the onset of symptoms of type 1 diabetes, which question should the nurse ask? a. "Have you lost any weight lately?" b. "Do you crave fluids containing sugar?" c. "How long have you felt anorexic?" d. "Is your urine unusually dark-colored?"

A Rationale: Weight loss occurs because the body is no longer able to absorb glucose and starts to break down protein and fat for energy. The patient is thirsty but does not necessarily crave sugar- containing fluids. Increased appetite is a classic symptom of type 1 diabetes. With the classic symptom of polyuria, urine will be very dilute. Cognitive Level: Application Text Reference: pp. 1255, 1258 Nursing Process: Assessment NCLEX: Physiological Integrity

When a client learned that the symptoms of diabetes were caused by high levels of blood glucose the client decided to stop eating carbohydrates. In this instance, the nurse would be concerned that the client would develop what complication? a. acidosis b. atherosclerosis c. glycosuria d. retinopathy

A When a client's carbohydrate consumption is inadequate ketones are produced from the breakdown of fat. These ketones lower the pH of the blood, potentially causing acidosis that can lead to a diabetic coma.

A 60 year old patient has an abrupt onset of anorexia, nausea and vomiting, hepatomegaly, and abnormal liver function studies. Serologic testing is negative for viral causes of hepatitis. During assessment of the patient, it is most important for the nurse to question the patient regarding A. any prior exposure to people with jaundice B. the use of all prescription and OTC (over the counter) medications C. treatment of chronic diseases with corticosteriods D. exposure to children recently immunized for hepatitis B

A and D assess for exposure to hepatitis. Hepatitis was ruled out this is inappropriate. C is incorrect because corticosteroids do not commonly cause liver disease B is correct because overdose of medications can cause liver disease.

hypothyroidism

A condition of hyposecretion of the thyroid gland causing low thyroid levels in the blood that result in sluggishness, slow pulse, and often obesity,

hyperthyriodism

A excessive secretion of thyriod hormones resulting from different factors including autoimmune stimulation (graves disease) excessive TSH by the pituitary gland,neoplasms and excessive TH medication

Addison's disease

A high protein, high carb diet and hydrocortisone treat ________

When caring for a client in a thyroid crisis, the nurse would question an order for?

A hyper thermia blanket

thyroid storm

A life-threatening complication of thyroidectomy which involves fever, tachycardia, delirium,and irritability

"Laboratory studies indicate that a client's blood glucose level is 185 mg/dl. Two hours have passed since the client ate breakfast. Which test would yield the most conclusive diagnostic information about the client's glucose utilization?

A test of serum glycosylated hemoglobin (Hb A1c)

The client diagnosed with Type I diabetes is found lying unconscious on the floor of the bathroom. Which interventions should the nurse implement first? A. Administer 50% dextrose IVP. B. Notify the health-care provider. C. Move the client to ICD. D. Check the serum glucose level.

A) admin 50% dextrose IVPThe nurse should assume the client is hypoglycemic and administer IVP dextrose, which will rouse the client immediately. If the collapse is the result of hyperglycemia, this additional dextrose will not further injure the client.

"When assessing the patient experiencing the onset of symptoms of type 1 diabetes, which question should the nurse ask? a. ""Have you lost any weight lately?"" b. ""Do you crave fluids containing sugar?"" c. ""How long have you felt anorexic?"" d. ""Is your urine unusually dark-colored?""

A) lost any weight?"a. Weight loss occurs because the body is no longer able to absorb glucose and starts to break down protein and fat for energy. b. The patient is thirsty but does not necessarily crave sugar- containing fluids. c. Increased appetite is a classic symptom of type 1 diabetes. d. With the classic symptom of polyuria, urine will be very dilute."

which are symptoms of hypoglycemia? A. irritability, B. drowsiness c. Abdominal pain D. nausea and vomiting

A. Irritability: signs of hypoglycemia include irritability, shaky feeling, hunger, headache, dizziness. Other symptoms are hyperglycemia.

"A client with a history of hypertension is diagnosed with primary hyperaldosteronism. This diagnosis indicates that the client's hypertension is caused by excessive hormone secretion from which gland?

Adrenal cortex

Sex hormones are secretes by ___________ and this is related to what d/o

Adrenal cortex and cushings.

What is the treatment for cushing's disease and what should be given throughout the client's lifetime (on one condition)? What does the client need in their diet pre-treatment (4)?

Adrenalectomy (unilateral or bilateral). If both are removed > lifetime replacement of steroids. Client needs more potassium, less sodium, more protein, and more calcium.

"The risk factors for type 1 diabetes include all of the following except: "a. Diet b. Genetic c. Autoimmune d. Environmental"

A: Type 1 diabetes is a primary failure of pancreatic beta cells to produce insulin. It primarily affects children and young adults and is unrelated to diet.

Intermediate Acting Insulin

AM NPH -onset: 1-2 hours -peak: 4-12 hours -duration: 18-24 hours Lente -onset: 1-2.5 hours -peak: 3-10 hours -duration: 18-24 hours

"The nurse is teaching the client about risk factors for diabetes mellitus. Which risk factor for diabetes mellitus is nonmodifiable?

Advanced age

"During the first 24 hours after a client is diagnosed with addisonian crisis, which task should the nurse perform frequently?

Assess vital signs.

The nurse is educating a pregnant client who has gestational diabetes. Which of the following statements should the nurse make to the client? Select all that apply. a. Cakes, candies, cookies, and regular soft drinks should be avoided. b. Gestational diabetes increases the risk that the mother will develop diabetes later in life. c. Gestational diabetes usually resolves after the baby is born. d. Insulin injections may be necessary. e. The baby will likely be born with diabetes f. The mother should strive to gain no more weight during the pregnancy.

ANS: A, B, C, D Gestational diabetes can occur between the 16th and 28th week of pregnancy. If not responsive to diet and exercise, insulin injections may be necessary. Concentrated sugars should be avoided. Weight gain should continue, but not in excessive amounts. Usually, gestational diabetes disappears after the infant is born. However, diabetes can develop 5 to 10 years after the pregnancy.

An external insulin pump is prescribed for a client with DM. The client asks the nurse about the functioning of the pump. The nurse bases the response on the information that the pump: "a. Gives small continuous dose of regular insulin subcutaneously, and the client can self-administer a bolus with an additional dosage from the pump before each meal. b. Is timed to release programmed doses of regular or NPH insulin into the bloodstream at specific intervals. c. Is surgically attached to the pancreas and infuses regular insulin into the pancreas, which in turn releases the insulin into the bloodstream. d. Continuously infuses small amounts of NPH insulin into the bloodstream while regularly monitoring blood glucose levels"

ANSWER A. An insulin pump provides a small continuous dose of regular insulin subcutaneously throughout the day and night, and the client can self-administer a bolus with additional dosage from the pump before each meal as needed. Regular insulin is used in an insulin pump. An external pump is not attached surgically to the pancreas.

For a client w/ addison's disease, what should be focused on (what is lost)? Thus, since this is insufficient, what is lost and what is retained? Treatment will involve the mineralocorticoid drug _________ (Florinef)(aldosterone). _____ is very important in adjusting their medication.

Aldosterone. Na/Water is lost, K+ is retained. Fludrocortisone. Weight.

"Which of the following is the most common cause of hyperaldosteronism?

An adrenal adenoma

"The nurse is assessing a client with Cushing's syndrome. Which observation should the nurse report to the physician immediately?

An irregular apical pulse

An adolescent with Type 1 diabetes is experiencing a problem with diabetic ketoacidosis. Which lab results reflect this condition?

Arterial blood ph of 7.28\

"A client brings her food journal containing her dietary intake for the past 3 days to the diabetic clinic. A nurse notes that despite dietary teaching about carbohydrate intake, the client consumed 3 servings of bread each day. What should the nurse do with this information?

Ask the diabetes educator to review with the client ways to decrease carbohydrate intake.

"A female client who weighs 210 lb (95 kg) and has been diagnosed with hyperglycemia tells the nurse that her husband sleeps in another room because her snoring keeps him awake. The nurse notices that she has large hands and a hoarse voice. Which of the following would the nurse suspect as a possible cause of the client's hyperglycemia?

Acromegaly

Pituitary

Acromegaly Parlodel (bromocripitine) Sondostatin (octreotide)-injection Dostirex (cabergaline) Somavent Reduce amount of prolacting in body, check GH measurments, teach hyperglycemia

*Adrenalcortical insufficiency (not enough steroids) is the same as ________ disease. What three things are important to remember about this disease?

Addison's disease (think ADD steroids).--1.) not enough steroids 2.) hyperkalemia (no aldosterone) 3.) shock!

"The nurse understands that for the parathyroid hormone to exert its effect, what must be present?

Adequate vitamin D level

"Which statement about fluid replacement is accurate for a client with hyperosmolar hyperglycemic nonketotic syndrome (HHNS)?

Administer 2 to 3 L of I.V. fluid over 2 to 3 hours.

A client with type 2 diabetes is sc heduled for major abdominal surgery. How will the nurse anticipate controlling blood glucose levels in this client during the immediate postoperative period?

Administer insulin on a sliding scale basis.

"1. A patient with newly diagnosed type 2 diabetes mellitus asks the nurse what ""type 2"" means in relation to diabetes. The nurse explains to the patient that type 2 diabetes differs from type 1 diabetes primarily in that with type 2 diabetes a. the pt is totally dependent on an outside source of insulin b. there is a decreased insulin secretion and cellular resistance to insulin that is produced c. the immune system destroys the pancreatic insulin-producing cells d. the insulin precurosr that is secreted by the pancreas is not activated by the liver

Answer B - Rationale: In type 2 diabetes, the pancreas produces insulin, but the insulin is insufficient for the body's needs or the cells do not respond to the insulin appropriately. The other information describes the physiology of type 1 diabetes

"The nurse caring for a 54-year-old patient hospitalized with diabetes mellitus would look for which of the following laboratory test results to obtain information on the patient's past glucose control? a. prealbumin level b. urine ketone level c. fasting glucose level d. glycosylated hemoglobin level

Answer d: A glycosylated hemoglobin level detects the amount of glucose that is bound to red blood cells (RBCs). When circulating glucose levels are high, glucose attaches to the RBCs and remains there for the life of the blood cell, which is approximately 120 days. Thus the test can give an indication of glycemic control over approximately 2 to 3 months.

A patient is admitted to the hospital with a diagnosis of primary hyperparathyroidism. A nurse checking the patient's lab results would expect which of the following changes in laboratory findings? A. Elevated serum calcium. B. Low serum parathyroid hormone (PTH). C. Elevated serum vitamin D. D. Low urine calcium.

Answer: A The parathyroid glands regulate the calcium level in the blood. In hyperparathyroidism, the serum calcium level will be elevated. Parathyroid hormone levels may be high or normal but not low. The body will lower the level of vitamin D in an attempt to lower calcium. Urine calcium may be elevated, with calcium spilling over from elevated serum levels. This may cause renal stones.

When obtaining a health history from a patient with acute pancreatitis, the nurse asks the patient specifically about a history of A. smoking B. alcohol use C. diabetes mellitus D. high-fat dietary intake

Answer: B pancreatitis is associated with alcoholism

"The nurse is developing a teaching plan for a client diagnosed with diabetes insipidus. The nurse should include information about which hormone lacking in clients with diabetes insipidus?

Antidiuretic hormone (ADH)

What 4 classes of medications are used to treat hyperthyroidism/graves disease?

Antithyroids. Iodine Compounds. Beta Blockers. Radioactive Iodine.

A newly diagnosed type 1 diabetic patient likes to run 3 miles several mornings a week. Which teaching will the nurse implement about exercise for this patient? a. "You should not take the morning NPH insulin before you run." b. "Plan to eat breakfast about an hour before your run." c. "Afternoon running is less likely to cause hypoglycemia." d. "You may want to run a little farther if your glucose is very high."

B Rationale: Blood sugar increases after meals, so this will be the best time to exercise. NPH insulin will not peak until mid-afternoon and is safe to take before a morning run. Running can be done in either the morning or afternoon. If the glucose is very elevated, the patient should postpone the run.

A patient with type 1 diabetes who uses glargine (Lantus) and lispro (Humalog) insulin develops a sore throat, cough, and fever. When the patient calls the clinic to report the symptoms and a blood glucose level of 210 mg/dl, the nurse advises the patient to a. use only the lispro insulin until the symptoms of infection are resolved. b. monitor blood glucose every 4 hours and notify the clinic if it continues to rise. c. decrease intake of carbohydrates until glycosylated hemoglobin is less than 7%. d. limit intake to non-calorie-containing liquids until the glucose is within the usual range.

B Rationale: Infection and other stressors increase blood glucose levels and the patient will need to test blood glucose frequently, treat elevations appropriately with insulin, and call the health care provider if glucose levels continue to be elevated. Discontinuing the glargine will contribute to hyperglycemia and may lead to DKA. Decreasing carbohydrate or caloric intake is not appropriate as the patient will need more calories when ill. Glycosylated hemoglobins are not used to test for short-term alterations in blood glucose.

A patient with cirrhosis has a massive hemorrhage from esophageal varices. In planning care for the patient, the nurse gives the highest priority to the goal of a. controlling bleeding. b. maintenance of the airway. c. maintenance of fluid volume. d. relieving the patient's anxiety.

B Rationale: Maintaining gas exchange has the highest priority because oxygenation is essential for life. The airway is compromised by the bleeding in the esophagus and aspiration easily occurs. The other goals would also be important for this patient, but they are not as high a priority as airway maintenance.

A patient with cirrhosis has 4+ pitting edema of the feet and legs and massive ascites. The data indicate that it is most important for the nurse to monitor the patient's a. temperature. b. albumin level. c. hemoglobin. d. activity level.

B Rationale: The low oncotic pressure caused by hypoalbuminemia is a major pathophysiologic factor in the development of ascites and edema. The other parameters should also be monitored, but they are not contributing factors to the patient's current symptoms.

A patient with type 2 diabetes has sensory neuropathy of the feet and legs and peripheral vascular disease evidenced by decreased peripheral pulses and dependent rubor. The nurse teaches the patient that a. the feet should be soaked in warm water on a daily basis. b. flat-soled leather shoes are the best choice to protect the feet from injury. c. heating pads should always be set at a very low temperature. d. over-the-counter (OTC) callus remover may be used to remove callus and prevent pressure.

B Rationale: The patient is taught to avoid high heels and that leather shoes are preferred. The feet should be washed, but not soaked, in warm water daily. Heating pad use should be avoided. Commercial callus and corn removers should be avoided; the patient should see a specialist to treat these problems.

What drugs will they put you on? what will it cause you to do? gain or loose

Fludrocortisone (florinef) which is aldosterone - notice it has "cortisone" in it - this is a steriod - aldosterone is a mineralocorticoide.. a steriod. / Will cause you to gain weight - start retaining water and sodium with aldosterone.

A patient is admitted with an abrupt onset of jaundice, nausea and vomiting, hepatomegaly, and abnormal liver function studies. Serologic testing is negative for viral causes of hepatitis. Which question by the nurse is most appropriate? a. "Have you been around anyone with jaundice?" b. "Do you use any prescription or over-the-counter (OTC) drugs?" c. "Are you taking corticosteroids for any reason?" d. "Is there any history of IV drug use?"

B Rationale: The patient's symptoms, lack of antibodies for hepatitis, and the ABRUPT onset of symptoms suggest toxic hepatitis, which can be caused by commonly used OTC drugs such as acetaminophen (Tylenol). Exposure to a jaundiced individual and a history of IV drug use are risk factors for VIRAL hepatitis. Corticosteroid use does not cause the symptoms listed.

The nurse identifies a nursing diagnosis of risk for impaired skin integrity for a patient with cirrhosis who has ascites and 4+ pitting edema of the feet and legs. An appropriate nursing intervention for this problem is to a. restrict dietary protein intake. b. arrange for a pressure-relieving mattress. c. perform passive range of motion QID. d. turn the patient every 4 hours.

B Rationale: The pressure-relieving mattress will decrease the risk for skin breakdown for this patient. Dietary protein intake may be increased in patients with ascites to improve oncotic pressure. Turning the patient every 4 hours will not be adequate to maintain skin integrity. Passive range of motion will not take pressure off areas like the sacrum that are vulnerable to breakdown.

When obtaining a health history from a patient with acute pancreatitis, the nurse asks the patient specifically about a history of a. cigarette smoking. b. alcohol use. c. diabetes mellitus. d. high-protein diet.

B Rationale: Alcohol use is one of the most common risk factors for pancreatitis in the United States. Cigarette smoking, diabetes, and high-protein diets are not risk factors.

31. A newly diagnosed type 1 diabetic patient likes to run 3 miles several mornings a week. Which teaching will the nurse implement about exercise for this patient? a. "You should not take the morning NPH insulin before you run." b. "Plan to eat breakfast about an hour before your run." c. "Afternoon running is less likely to cause hypoglycemia." d. "You may want to run a little farther if your glucose is very high."

B Rationale: Blood sugar increases after meals, so this will be the best time to exercise. NPH insulin will not peak until mid-afternoon and is safe to take before a morning run. Running can be done in either the morning or afternoon. If the glucose is very elevated, the patient should postpone the run. Cognitive Level: Application Text Reference: p. 1269 Nursing Process: Implementation NCLEX: Physiological Integrity

10. A 1200-calorie diet and exercise are prescribed for a patient with newly diagnosed type 2 diabetes. The patient tells the nurse, "I hate to exercise! Can't I just follow the diet to keep my glucose under control?" The nurse teaches the patient that the major purpose of exercise for diabetics is to a. increase energy and sense of well-being, which will help with body image. b. facilitate weight loss, which will decrease peripheral insulin resistance. c. improve cardiovascular endurance, which is important for diabetics. d. set a successful pattern, which will help in making other needed changes.

B Rationale: Exercise is essential to decrease insulin resistance and improve blood glucose control. Increased energy, improved cardiovascular endurance, and setting a pattern of success are secondary benefits of exercise, but they are not the major reason. Cognitive Level: Application Text Reference: p. 1269 Nursing Process: Implementation NCLEX: Physiological Integrity

12. A patient with type 1 diabetes has an unusually high morning glucose measurement, and the health care provider wants the patient evaluated for possible Somogyi effect. The nurse will plan to a. administer an increased dose of NPH insulin in the evening. b. obtain the patient's blood glucose at 3:00 in the morning. c. withhold the nighttime snack and check the glucose at 6:00 AM. d. check the patient for symptoms of hypoglycemia at 2:00 to 4:00 AM.

B Rationale: In the Somogyi effect, the patient's blood glucose drops in the early morning hours (in response to excess insulin administration), which causes the release of hormones that result in a rebound hyperglycemia. It is important to check the blood glucose in the early morning hours to detect the initial hypoglycemia. An increased evening NPH dose or holding the nighttime snack will further increase the risk for early morning hypoglycemia. Information about symptoms of hypoglycemia will not be as accurate as checking the patient's blood glucose in determining whether the patient has the Somogyi effect. Cognitive Level: Application Text Reference: pp. 1263-1264 Nursing Process: Planning NCLEX: Physiological Integrity

1. A patient with newly diagnosed type 2 diabetes mellitus asks the nurse what "type 2" means in relation to diabetes. The nurse explains to the patient that type 2 diabetes differs from type 1 diabetes primarily in that with type 2 diabetes a. the patient is totally dependent on an outside source of insulin. b. there is decreased insulin secretion and cellular resistance to insulin that is produced. c. the immune system destroys the pancreatic insulin-producing cells. d. the insulin precursor that is secreted by the pancreas is not activated by the liver.

B Rationale: In type 2 diabetes, the pancreas produces insulin, but the insulin is insufficient for the body's needs or the cells do not respond to the insulin appropriately. The other information describes the physiology of type 1 diabetes. Cognitive Level: Application Text Reference: p. 1255 Nursing Process: Implementation NCLEX: Physiological Integrity

25. A patient with type 1 diabetes who uses glargine (Lantus) and lispro (Humalog) insulin develops a sore throat, cough, and fever. When the patient calls the clinic to report the symptoms and a blood glucose level of 210 mg/dl, the nurse advises the patient to a. use only the lispro insulin until the symptoms of infection are resolved. b. monitor blood glucose every 4 hours and notify the clinic if it continues to rise. c. decrease intake of carbohydrates until glycosylated hemoglobin is less than 7%. d. limit intake to non-calorie-containing liquids until the glucose is within the usual range.

B Rationale: Infection and other stressors increase blood glucose levels and the patient will need to test blood glucose frequently, treat elevations appropriately with insulin, and call the health care provider if glucose levels continue to be elevated. Discontinuing the glargine will contribute to hyperglycemia and may lead to DKA. Decreasing carbohydrate or caloric intake is not appropriate as the patient will need more calories when ill. Glycosylated hemoglobins are not used to test for short-term alterations in blood glucose. Cognitive Level: Application Text Reference: p. 1272 Nursing Process: Implementation NCLEX: Health Promotion and Maintenance

3. During a diabetes screening program, a patient tells the nurse, "My mother died of complications of type 2 diabetes. Can I inherit diabetes?" The nurse explains that a. as long as the patient maintains normal weight and exercises, type 2 diabetes can be prevented. b. the patient is at a higher than normal risk for type 2 diabetes and should have periodic blood glucose level testing. c. there is a greater risk for children developing type 2 diabetes when the father has type 2 diabetes. d. although there is a tendency for children of people with type 2 diabetes to develop diabetes, the risk is higher for those with type 1 diabetes.

B Rationale: Offspring of people with type 2 diabetes are at higher risk for developing type 2 diabetes. The risk can be decreased, but not prevented, by maintenance of normal weight and exercising. The risk for children of a person with type 1 diabetes to develop diabetes is higher when it is the father who has the disease. Offspring of people with type 2 diabetes are more likely to develop diabetes than offspring of those with type 1 diabetes. Cognitive Level: Application Text Reference: p. 1256 Nursing Process: Implementation NCLEX: Physiological Integrity

34. The health care provider orders oral glucose tolerance testing for a patient seen in the clinic. Which information from the patient's health history is most important for the nurse to communicate to the health care provider? a. The patient had a viral illness 2 months ago. b. The patient uses oral contraceptives. c. The patient runs several days a week. d. The patient has a family history of diabetes.

B Rationale: Oral contraceptive use may falsely elevate oral glucose tolerance test (OGTT) values. A viral 2 months previously illness may be associated with the onset of type 1 diabetes but will not falsely impact the OGTT. Exercise and a family history of diabetes both can affect blood glucose but will not lead to misleading information from the OGTT. Cognitive Level: Application Text Reference: p. 1267 Nursing Process: Assessment NCLEX: Physiological Integrity

A diabetic patient is started on intensive insulin therapy. The nurse will plan to teach the patient about mealtime coverage using _____ insulin. a. NPH b. lispro c. detemir d. glargine

B Rationale: Rapid or short acting insulin is used for mealtime coverage for patients receiving intensive insulin therapy. NPH, glargine, or detemir will be used as the basal insulin.

15. A diabetic patient is started on intensive insulin therapy. The nurse will plan to teach the patient about mealtime coverage using _____ insulin. a. NPH b. lispro c. detemir d. glargine

B Rationale: Rapid or short acting insulin is used for mealtime coverage for patients receiving intensive insulin therapy. NPH, glargine, or detemir will be used as the basal insulin. Cognitive Level: Application Text Reference: p. 1260 Nursing Process: Planning NCLEX: Physiological Integrity

A 32-year-old patient has early alcoholic cirrhosis diagnosed by a liver biopsy. When planning patient teaching, the priority information for the nurse to include is the need for a. vitamin B supplements. b. abstinence from alcohol. c. maintenance of a nutritious diet. d. long-term, low-dose corticosteroids.

B Rationale: The disease progression can be stopped or reversed by alcohol abstinence. The other interventions may be used when cirrhosis becomes more severe to decrease symptoms or complications, but the priority for this patient is to stop the progression of the disease.

13. A patient receives a daily injection of 70/30 NPH/regular insulin premix at 7:00 AM. The nurse expects that a hypoglycemic reaction is most likely to occur between a. 8:00 and 10:00 AM. b. 4:00 and 6:00 PM. c. 7:00 and 9:00 PM. d. 10:00 PM and 12:00 AM.

B Rationale: The greatest insulin effect with this combination occurs mid afternoon. The patient is not at a high risk at the other listed times, although hypoglycemia may occur. Cognitive Level: Comprehension Text Reference: p. 1260 Nursing Process: Evaluation NCLEX: Physiological Integrity

35. Which of these laboratory values noted by the nurse when reviewing the chart of a diabetic patient indicates the need for further assessment of the patient? a. Fasting blood glucose of 130 mg/dl b. Noon blood glucose of 52 mg/dl c. Glycosylated hemoglobin of 6.9% d. Hemoglobin A1C of 5.8%

B Rationale: The nurse should assess the patient with a blood glucose level of 52 mg/dl for symptoms of hypoglycemia, and give the patient some carbohydrate-containing beverage such as orange juice. The other values are within an acceptable range for a diabetic patient. Cognitive Level: Application Text Reference: pp. 1281-1282 Nursing Process: Assessment NCLEX: Physiological Integrity

The nurse is caring for a client who has exophthalmos associated with the thyroid disease. What is the cause of exophthalmos?

Fluid and edema in the tissues behind the eye increases pressure.

30. A patient with type 2 diabetes has sensory neuropathy of the feet and legs and peripheral vascular disease evidenced by decreased peripheral pulses and dependent rubor. The nurse teaches the patient that a. the feet should be soaked in warm water on a daily basis. b. flat-soled leather shoes are the best choice to protect the feet from injury. c. heating pads should always be set at a very low temperature. d. over-the-counter (OTC) callus remover may be used to remove callus and prevent pressure.

B Rationale: The patient is taught to avoid high heels and that leather shoes are preferred. The feet should be washed, but not soaked, in warm water daily. Heating pad use should be avoided. Commercial callus and corn removers should be avoided; the patient should see a specialist to treat these problems. Cognitive Level: Application Text Reference: p. 1287 Nursing Process: Implementation NCLEX: Health Promotion and Maintenance

A patient with type 2 diabetes is scheduled for an outpatient coronary arteriogram. Which information obtained by the nurse when admitting the patient indicates a need for a change in the patient's regimen? a. The patient's most recent hemoglobin A1C was 6%. b. The patient takes metformin (Glucophage) every morning. c. The patient uses captopril (Capoten) for hypertension. d. The patient's admission blood glucose is 128 mg/dl.

B Rationale: To avoid lactic acidosis, metformin should not be used for 48 hours after IV contrast media are administered. The other patient data indicate that the patient is managing the diabetes appropriately.

33. A patient with type 2 diabetes is scheduled for an outpatient coronary arteriogram. Which information obtained by the nurse when admitting the patient indicates a need for a change in the patient's regimen? a. The patient's most recent hemoglobin A1C was 6%. b. The patient takes metformin (Glucophage) every morning. c. The patient uses captopril (Capoten) for hypertension. d. The patient's admission blood glucose is 128 mg/dl.

B Rationale: To avoid lactic acidosis, metformin should not be used for 48 hours after IV contrast media are administered. The other patient data indicate that the patient is managing the diabetes appropriately. Cognitive Level: Application Text Reference: p. 1266 Nursing Process: Assessment NCLEX: Physiological Integrity

"Which outcome indicates that treatment of a client with diabetes insipidus has been effective?

Fluid intake is less than 2,500 ml/day.

"During a class on exercise for clients with diabetes mellitus, a client asks the nurse how often he should exercise. Which answer by the nurse is appropriate?

Follow a regular, individualized exercise plan.

A home health nurse is at the home of a client with diabetes and arthritis. The client has difficulty drawing up insulin. It would be most appropriate for the nurse to refer the client to: "A) A social worker from the local hospital B) An occupational therapist from the community center C) A physical therapist from the rehabilitation agency D) Another client with diabetes mellitus and takes insulin"

B) An occupational therapist can assist a client to improve the fine motor skills needed to prepare an insulin injection.

One of the benefits of Glargine (Lantus) insulin is its ability to: "a.Release insulin rapidly throughout the day to help control basal glucose.b. Release insulin evenly throughout the day and control basal glucose levels.c. Simplify the dosing and better control blood glucose levels during the day.d. Cause hypoglycemia with other manifestation of other adverse reactions.

B)Release insulin evenly throughout the day and control basal glucose levels"Glargine (Lantus) insulin is designed to release insulin evenly throughout the day and control basal glucose levels.

Grave's disease

Form of hyperthyroidism causing exophthalmos

List at least 4 signs and symptoms of hypothyroidism (temperature, GI).

GI is slow. Weight increased. Fatigued. Cold skin. Speech is slow. Lack of expression.

"The guidelines for Carbohydrate Counting as medical nutrition therapy for diabetes mellitus includes all of the following EXCEPT: a. Flexibility in types and amounts of foods consumed b. Unlimited intake of total fat, saturated fat and cholesterol c. Including adequate servings of fruits, vegetables and the dairy group d. Applicable to with either Type 1 or Type 2 diabetes mellitusb. Unlimited intake of total fat, saturated fat and cholesterol"

B. You want to be careful of how much you eat in any food group.

Which of the following things must the nurse working with diabetic clients keep in mind about Hyperglycemic Hyperosmolar Nonketotic Syndrome (HHNS)? A. This syndrome occurs mainly in people with Type I Diabetes B. It has a higher mortality rate than Diabetic Ketoacidosis C. The client with HHNS is in a state of overhydration D. This condition develops very rapidly

B. It has a higher mortality rate than Diabetic Ketoacidosis HHNS occurs only in people with Type II Diabetes. It is a medical emergency and has a higher mortality rate than Diabetic Ketoacidosis. This condition develops very slowly over hours or days.

The nurs is administering metformin to a client. What nursing observation would cause the nurse concern regarding side effects of the medication?

Gastrointestianal upset

Glucagon Hydrochloride

Given to type 1 DM

"A client is taking Humulin NPH insulin daily every morning. The nurse instructs the client that the mostlikely time for a hypoglycemic reaction to occur is: A) 2-4 hours after administration B) 4-12 hours after administration C) 16-18 hours after administration D) 18-24 hours after administration

B: Rationale: Humulin is an intermediate acting insulin. The onset of action is 1.5 hours, it peaks in 4-12 hours, and its duration is 24 hours. Hypoglycemic reactions to insulin are most likely to occur during the peak time.

The nurse is teaching the parent of a diabetic child how to recognize the signs and symptoms of hypoglycemia. Which signs and symptoms should the nurse discuss with the parent?

Behavioral changes, increased heart rate, sweating and tremors

"The nurse expects to note an elevated serum glucose level in a client with hyperosmolar hyperglycemic nonketotic syndrome (HHNS). Which other laboratory finding should the nurse anticipate?

Below-normal serum potassium level

Pheochromocytoma

Benign tumor of adrenal medulla that causes hypersecretion of epinephrine and norepinephrine causing a hypertensive crisis, tachycardia, diaphoresis, apprehension, h/a, flight or fight, hyperglycemia

After a thyroidectomy, how should bleeding be checked for? What position should the patient be placed in? What is important, nutrition-wise? How should laryngeal nerve damage be assessed? Why?

Bleeding should be checked behind the neck. Semi-fowler's position to decrease pooling of blood. Make sure patient gets a lot of calories due to hyperthyroidism. Nerve damage is checked by looking for hoarsness in voice (in unilateral nerve damage), breathing difficulties(in bilateral nerve damage), or inability to speak (in bilateral nerve damage). ---Nerve damage could lead to vocal cord paralysis > airway obstruction > requires trach

The nurse would not which finding on the physical assessment of a client wiht a diagnosis of Cushing Disease?

Buffalo hump, Thinning hair, hirsutism, gynicomastia

A diagnosis of hyperglycemic hyperosmolar nonketotic coma (HHNC) is made for a patient with type 2 diabetes who is brought to the emergency department in an unresponsive state. The nurse will anticipate the need to a. administer glargine (Lantus) insulin. b. initiate oxygen by nasal cannula. c. insert a large-bore IV catheter. d. give 50% dextrose as a bolus.

C Rationale: HHNC is initially treated with large volumes of IV fluids to correct hypovolemia. Regular insulin is administered, not a long-acting insulin. There is no indication that the patient requires oxygen. Dextrose solutions will increase the patient's blood glucose and would be contraindicated.

. A patient with cirrhosis who is being treated with spironolactone (Aldactone) and furosemide (Lasix) has a serum sodium level of 135 mEq/L (135 mmol/L) and serum potassium 3.2 mEq/L (3.2 mmol/L). Before notifying the health care provider, the nurse should a. administer the furosemide and withhold the spironolactone. b. give both drugs as scheduled. c. administer the spironolactone. d. withhold both drugs until talking with the health care provider.

C Rationale: Spironolactone is a potassium-sparing diuretic and will help to increase the patient's potassium level. The nurse does not need to talk with the doctor before giving the spironolactone, although the health care provider should be notified about the low potassium value. The furosemide will further decrease the patient's potassium level and should be held until the nurse talks with the health care provider.

Hypoparathyroidism=____________=_____________

Hypocalcemia=hyperphosphatemia

A patient with severe cirrhosis has an episode of bleeding esophageal varices. To detect possible complications of the bleeding episode, it is most important for the nurse to monitor a. prothrombin time. b. bilirubin levels. c. ammonia levels. d. potassium levels.

C Rationale: The blood in the GI tract will be absorbed as protein and may result in an increase in ammonia level since the liver cannot metabolize protein well. The prothrombin time, bilirubin, and potassium levels should also be monitored, but these will not be affected by the bleeding episode.

The doctor is interested in how well a client has controlled their blood glucose since their last visit. What lab values could the nurse evaluate to determine how well the client has controlled their blood glucose over the past three months?

C HbgA1c is a blood test used to determine how well blood glucose has been controlled for the last three months.

The client tells the nurse that the client really misses having sugar with tea in the morning. What is an alternative that the nurse could advise them to help sweeten their drink. a. Oatrim c. sucralose b. Olestra d. tannin

C Aspartame is the generic name for a sweetener composed of two amino acids, phenylalanine and aspartic acid. Olestra and Oatrim are fat replacers and tannin is an acid found in some foods such as tea.

A patient hospitalized with possible acute pancreatitis has severe abdominal pain and nausea and vomiting. The nurse would expect the diagnosis to be confirmed with laboratory testing that reveals elevated serum a. calcium. b. bilirubin. c. amylase. d. potassium.

C Rationale: Amylase is elevated early in acute pancreatitis. Changes in bilirubin, calcium, and potassium levels are not diagnostic for pancreatitis.

When assessing the neurologic status of a patient with a diagnosis of hepatic encephalopathy, the nurse asks the patient to a. stand on one foot. b. ambulate with the eyes closed. c. extend both arms. d. perform the Valsalva maneuver.

C Rationale: Extending the arms allows the nurse to check for asterixis, a classic sign of hepatic encephalopathy. The other tests might also be done as part of the neurologic assessment but would not be diagnostic for hepatic encephalopathy.

38. A diabetic patient has a new order for inhaled insulin (Exubera). Which information about the patient indicates that the nurse should contact the patient before administering the Exubera? a. The patient has a history of a recent myocardial infarction. b. The patient's blood glucose is 224 mg/dl. c. The patient uses a bronchodilator to treat emphysema. d. The patient's temperature is 101.4° F.

C Rationale: Exubera is not recommended for patients with emphysema. The other data do not indicate any contraindication to using Exubera. Cognitive Level: Application Text Reference: p. 1263 Nursing Process: Assessment NCLEX: Physiological Integrity

24. A diagnosis of hyperglycemic hyperosmolar nonketotic coma (HHNC) is made for a patient with type 2 diabetes who is brought to the emergency department in an unresponsive state. The nurse will anticipate the need to a. administer glargine (Lantus) insulin. b. initiate oxygen by nasal cannula. c. insert a large-bore IV catheter. d. give 50% dextrose as a bolus.

C Rationale: HHNC is initially treated with large volumes of IV fluids to correct hypovolemia. Regular insulin is administered, not a long-acting insulin. There is no indication that the patient requires oxygen. Dextrose solutions will increase the patient's blood glucose and would be contraindicated. Cognitive Level: Application Text Reference: p. 1281 Nursing Process: Planning NCLEX: Physiological Integrity

A type 1 diabetic patient who was admitted with severe hypoglycemia and treated tells the nurse, "I did not have any of the usual symptoms of hypoglycemia." Which question by the nurse will help identify a possible reason for the patient's hypoglycemic unawareness? a. "Do you use any calcium-channel blocking drugs for blood pressure?" b. "Have you observed any recent skin changes?" c. "Do you notice any bloating feeling after eating?" d. "Have you noticed any painful new ulcerations or sores on your feet?"

C Rationale: Hypoglycemic unawareness is caused by autonomic neuropathy, which would also cause delayed gastric emptying. Calcium-channel blockers are not associated with hypoglycemic unawareness, although -adrenergic blockers can prevent patients from having symptoms of hypoglycemia. Skin changes can occur with diabetes, but these are not associated with autonomic neuropathy. If the patient can feel painful areas on the feet, neuropathy has not occurred.

29. A type 1 diabetic patient who was admitted with severe hypoglycemia and treated tells the nurse, "I did not have any of the usual symptoms of hypoglycemia." Which question by the nurse will help identify a possible reason for the patient's hypoglycemic unawareness? a. "Do you use any calcium-channel blocking drugs for blood pressure?" b. "Have you observed any recent skin changes?" c. "Do you notice any bloating feeling after eating?" d. "Have you noticed any painful new ulcerations or sores on your feet?"

C Rationale: Hypoglycemic unawareness is caused by autonomic neuropathy, which would also cause delayed gastric emptying. Calcium-channel blockers are not associated with hypoglycemic unawareness, although -adrenergic blockers can prevent patients from having symptoms of hypoglycemia. Skin changes can occur with diabetes, but these are not associated with autonomic neuropathy. If the patient can feel painful areas on the feet, neuropathy has not occurred. Cognitive Level: Application Text Reference: p. 1281 Nursing Process: Assessment NCLEX: Physiological Integrity

A patient who has just been diagnosed with type 2 diabetes is 5 ft 4 in (160 cm) tall and weighs 182 pounds (82 kg). A nursing diagnosis of imbalanced nutrition: more than body requirements is developed. Which patient outcome is most important for this patient? a. The patient will have a diet and exercise plan that results in weight loss. b. The patient will state the reasons for eliminating simple sugars in the diet. c. The patient will have a glycosylated hemoglobin level of less than 7%. d. The patient will choose a diet that distributes calories throughout the day.

C Rationale: The complications of diabetes are related to elevated blood glucose, and the most important patient outcome is the reduction of glucose to near-normal levels. The other outcomes are also appropriate but are not as high in priority.

7. A patient who has just been diagnosed with type 2 diabetes is 5 ft 4 in (160 cm) tall and weighs 182 pounds (82 kg). A nursing diagnosis of imbalanced nutrition: more than body requirements is developed. Which patient outcome is most important for this patient? a. The patient will have a diet and exercise plan that results in weight loss. b. The patient will state the reasons for eliminating simple sugars in the diet. c. The patient will have a glycosylated hemoglobin level of less than 7%. d. The patient will choose a diet that distributes calories throughout the day.

C Rationale: The complications of diabetes are related to elevated blood glucose, and the most important patient outcome is the reduction of glucose to near-normal levels. The other outcomes are also appropriate but are not as high in priority. Cognitive Level: Application Text Reference: p. 1273 Nursing Process: Planning NCLEX: Physiological Integrity

6. During a clinic visit 3 months following a diagnosis of type 2 diabetes, the patient reports following a reduced-calorie diet. The patient has not lost any weight and did not bring the glucose-monitoring record. The nurse will plan to obtain a(n) a. fasting blood glucose level. b. urine dipstick for glucose. c. glycosylated hemoglobin level. d. oral glucose tolerance test.

C Rationale: The glycosylated hemoglobin (Hb A1C) test shows the overall control of glucose over 90 to 120 days. A fasting blood level indicates only the glucose level at one time. Urine glucose testing is not an accurate reflection of blood glucose level and does not reflect the glucose over a prolonged time. Oral glucose tolerance testing is done to diagnose diabetes, but is not used for monitoring glucose control once diabetes has been diagnosed. Cognitive Level: Application Text Reference: pp. 1258-1259 Nursing Process: Planning NCLEX: Physiological Integrity

A diabetic patient is admitted with ketoacidosis and the health care provider writes all of the following orders. Which order should the nurse implement first? a. Start an infusion of regular insulin at 50 U/hr. b. Give sodium bicarbonate 50 mEq IV push. c. Infuse 1 liter of normal saline per hour. d. Administer regular IV insulin 30 U.

C Rationale: The most urgent patient problem is the hypovolemia associated with DKA, and the priority is to infuse IV fluids. The other actions can be accomplished after the infusion of normal saline is initiated.

23. A diabetic patient is admitted with ketoacidosis and the health care provider writes all of the following orders. Which order should the nurse implement first? a. Start an infusion of regular insulin at 50 U/hr. b. Give sodium bicarbonate 50 mEq IV push. c. Infuse 1 liter of normal saline per hour. d. Administer regular IV insulin 30 U.

C Rationale: The most urgent patient problem is the hypovolemia associated with DKA, and the priority is to infuse IV fluids. The other actions can be accomplished after the infusion of normal saline is initiated. Cognitive Level: Application Text Reference: p. 1280 Nursing Process: Implementation NCLEX: Physiological Integrity

The nurse teaches the diabetic patient who rides a bicycle to work every day to administer morning insulin into the a. thigh. b. buttock. c. arm. d. abdomen.

D Rationale: Patients should be taught not to administer insulin into a site that will be exercised because exercise will increase the rate of absorption. The thigh, buttock, and arm are all exercised by riding a bicycle.

"A diabetic patient has a serum glucose level of 824 mg/dL (45.7 mmol/L) and is unresponsive. Following assessment of the patient, the nurse suspects diabetic ketoacidosis rather than hyperosmolar hyperglycemic syndrome based on the finding of "a. polyuria b. severe dehydration c. rapid, deep respirations ) d. decreased serum potassium"

C is correct, Signs and symptoms of DKA include manifestations of dehydration such as poor skin turgor, dry mucous membranes, tachycardia, and orthostatic hypotension. Early symptoms may include lethargy and weakness. As the patient becomes severely dehydrated, the skin becomes dry and loose, and the eyeballs become soft and sunken. Abdominal pain is another symptom of DKA that may be accompanied by anorexia and vomiting. Kussmaul respirations (i.e., rapid, deep breathing associated with dyspnea) are the body's attempt to reverse metabolic acidosis through the exhalation of excess carbon dioxide. Acetone is identified on the breath as a sweet, fruity odor. Laboratory findings include a blood glucose level greater than 250 mg/dL, arterial blood pH less than 7.30, serum bicarbonate level less than 15 mEq/L, and moderate to large ketone levels in the urine or blood ketones.

In educating a client with diabetes, what response would reveal need for further education? "A. I should avoid tights B. I should take good care of my toe nails C. I should not go more than 3 days without washing my feet D. I should avoid going barefoot and should wear clean socks

C)I should not go more than 3 days w/o washing my feet"The recommended self-care routine is to wash feet on a daily basis without soaking and carefully cleaning."

A patient is admitted with diabetes mellitus, has a glucose level of 380 mg/dl, and a moderate level of ketones in the urine. As the nurse assesses for signs of ketoacidosis, which of the following respiratory patterns would the nurse expect to find?"A-Central apnea B-Hypoventilation C-Kussmaul respirations D- Cheyne-Stokes respirations"

C-Kussmaul respirationsIn diabetic ketoacidosis, the lungs try to compensate for the acidosis by blowing off volatile acids and carbon dioxide. This leads to a pattern of Kussmaul respirations, which are deep and nonlabored.

"A nurse is caring for a client admitted to the emergency department with diabetic ketoacidosis (DKA). In the acute phase, the priority nursing action is to prepare to: "A. Correct the acidosis B. Administer 5% dextrose intravenously C. Administer regular insulin inraVenously D. Apply a monitor for an electrocardiogram."

C. Administer regular insulin inraVenously Lack (absolute or relative) of insulin is the primary cause of DKA. Treatment consists of insulin administration (regular insulin), intravenous fluid administration (normal saline initially), and potassium replacement, followed by correcting acidosis. Applying an electrocardiogram monitor is not a priority action.

The nurse is working with an overweight client who has a high-stress job and smokes. This client has just received a diagnosis of Type II Diabetes and has just been started on an oral hypoglycemic agent. Which of the following goals for the client which if met, would be most likely to lead to an improvement in insulin efficiency to the point the client would no longer require oral hypoglycemic agents? a. Comply with medication regimen 100% for 6 months b. Quit the use of any tobacco products by the end of three months c. Lose a pound a week until weight is in normal range for height and exercise 30 minutes daily d. Practice relaxation techniques for at least five minutes five times a day for at least five months

C. Lose a pound a week until weight is in normal range for height and exercise 30 minutes daily When type II diabetics lose weight through diet and exercise they sometimes have an improvement in insulin efficiency sufficient to the degree they no longer require oral hypoglycemic agents.

A patient with cirrhosis is being treated with spironolactone (Aldactone) tid and furosemide (Lasix) bid. The patient's most recent laboratory results indicate a serum sodium of 134 mEq/L (134 mmol/L) and a serum potassium of 3.2 mEq/L (3.2 mmol/L). Before notifying the physician, the nurse should A. administer only the furosemide B. administer both drugs as ordered C. administer only the spironolactone D. Withhold the furosemide and spironolactone

C. administer only the spironolactone The potassium level is dangerously low. Lasix is potassium depleting, while spironolactone is potassium sparing. You would hold the Lasix and call the physician. This is a good NCLEX question that integrates this course with pharmacology.

A thirty five year old male has been an insulin-dependent diabetic for five years and now is unable to urinate. Which of the following would you most likely suspect? A: Atherosclerosis B: Diabetic nephropathy C: Autonomic neuropathy D: Somatic neuropathy

C: Autonomic neuropathy

Tx for hypothyroid:

Meds: All have thyroid in a form in the name: Levothyroxine (Synthroid), Thyroglobulin (Proloid), Liothyronine (cytomel)... They will take these forever... watch for CAD

Risk factors for type 2 diabetes include all of the following except: a. Advanced age b. Obesity c. Smoking d. Physical inactivity

C: smoking Additional risk factors for type 2 diabetes are a family history of diabetes, impaired glucose metabolism, history of gestational diabetes, and race/ethnicity. African-Americans, Hispanics/Latinos, Asian Americans, Native Hawaiians, Pacific Islanders, and Native Americans are at greater risk of developing diabetes than whites.

people with hypothyroidism tend to also have:

CAD... though they don't know why (Corinary Artery Disease)

A client is brought to the emergency department in an unresponsive state, and a diagnosis of hyperglycemic hyperosmolar nonketotic syndrome is made. The nurse would immediately prepare to initiate which of the following anticipated physician's prescriptions? 1. Endotracheal intubation 2. 100 units of NPH insulin 3. Intravenous infusion of normal saline 4. Intravenous infusion of sodium bicarbonate

CORRECT ANSWER: 3. Intravenous infusion of normal saline Rationale: The primary goal of treatment is hyperglycemic hyperosmolar nonketotic syndrome (HHNS) is to rehydrate the client to restore the fluid volume and to correct electrolyte deficiency. Intravenous fluid replacement is similar to that administered in diabetic keto acidosis (DKA) and begins with IV infusion of normal saline. Regular insulin, not NPH insulin, would be administered. The use of sodium bicarbonate to correct acidosis is avoided because it can precipitate a further drop in serum potassium levels. Intubation and mechanical ventilation are not required to treat HHNS.

Tx for hypothyroidism

Calcium - may recieve it IV / and phosphorus binding drugs (becuase of that inverse relationship) binding means you can poop out the phospherous)

Steroids decrease serum _____ by making you excrete it through the ____ ______.

Calcium. GI Tract.

"The nursing staff has just been trained how to use and care for a new blood glucose monitor. Which nursing intervention demonstrates proper use of a blood glucose monitor?

Calibrate the machine after installing a new battery.

"While administering morning medications, a nurse enters the room of a client who recently had a thyroidectomy. She observes that the client is sitting up in bed but appears unresponsive. After confirming unresponsiveness, what should the nurse do next?

Call for help.

A client is prescribed levothyroxine daily. An important instruction to give the client regarding administration of this drug is ?

Call the physician immediately at the onset of palpitations or nervousness.

"A client is diagnosed with syndrome of inappropriate antidiuretic hormone (SIADH). The nurse informs the client that the physician will prescribe diuretic therapy and restrict fluid and sodium intake to treat the disorder. If the client doesn't comply with the recommended treatment, which complication may arise?

Cerebral edema

The nurse is caring for a client postthyroidectomy. Which of the follwoing observations sould cause the nurse the most concern?

Client's wrist spontaneously flexes whent he blood pressure cuff is tightened.

Fertility

Clomid (clomiphene citrate)-to induce ovulation -pt teaching-multiple birth, take basal temp Parladel (bromocriptate mesylate)-amenorrhea, hypogonadism (infertility) -pt teaching-baseline & periodic evals of cardiac, hepatic, renal ftn are recommended, may take up to 8 wks.

"The nurse should expect to administer which medication to a client with gout?

Colchicine

"A client's blood glucose level is 45 mg/dl. The nurse should be alert for which signs and symptoms?

Coma, anxiety, confusion, headache, and cool, moist skin

"Which of the following would indicate that a client has developed water intoxication secondary to treatment for diabetes insipidus?

Confusion and seizures

A client has been receiving corticosteroids for the past 5 years for treatment of rheumatoid arthritus. The client is now being admitted for a major surgical procedure. What would be important for the nurse to assess after the clkient returns form surgery?

Continued orders for corticosteroids at or above level the clint was receiving before surgery.

SIADH

Continued release of antidiuretic hormone resulting in water intoxication

Pituitary

Controlled primarily by the hypothalamus; termed :master gland: as it directly affects the function of other endocrine glands

Pituitary Gland

Controlled primarily by the hypothalamus; termed :master gland: as it directly affects the function of other endocrine glands

"A client with Cushing's syndrome is admitted to the medical-surgical unit. While collecting data, the nurse notes that the client is agitated and irritable, has poor memory, reports loss of appetite, and appears disheveled. These findings are consistent with which problem?

Depression

A nurse is caring for a client with type 1 diabetes mellitus. Which client complaint would alert the nurse to the presence of a possible hypoglycemic reaction ? A. Tremors B. Anorexia C. Hot, Dry skin D. Muscle cramps

Correct Answer A Decreased blood glucose levels produce autonomic nervous system symptoms, which are manifested classsically as nervousness, irritability, and tremors. Option C is more likely to occur with hyperglycemia. Options B and D are unrealted to the signs of hyperglycemia

A client with DKA is being treated in the ED. What would the nurse suspect? 1. Comatose state 2. Decreased Urine Output 3. Increased respirations and an increase in pH. 4. Elevated blood glucose level and low plasma bicarbonate level.

Correct Answer: 4 Rationale: In DKA the arteriole pH is lower than 7.35, plasma bicarbonate is lower than 15 mEq/L, the blood glucose is higher than 250, and ketones are present in the blood and urine. The client would be experiencing polyuria and Kussmauls respirations would be present. A comatose state may occur if DKA is not treated.

A client is admitted to the medical-surgical floor with a diagnosis of acute pancreatitis. His blood pressure is 136/76 mm Hg, pulse 96 beats/minute, respirations 22 breaths/minute, temperature 99°F (38.3°C), and he has been experiencing severe vomiting for 24 hours. His past medical history reveals hyperlipidemia and alcohol abuse. The physician prescribes a nasogastric (NG) tube for the client. Which of the following is the primary purpose for insertion of the NG tube? A. Empty the stomach of fluids and gas to relieve vomiting. B. Prevent spasms at the sphincter of Oddi. C. Prevent air from forming in the small and large intestines. D. Remove bile from the gallbladder.

Correct answer: A An NG tube is no longer routinely inserted to treat pancreatitis, but if the client has protracted vomiting, the NG tube is inserted to drain fluids and gas and relieve vomiting. An NG tube doesn't prevent spasms at the sphincter of Oddi (a valve in the duodenum that controls the flow of digestive enzymes) or prevent air from forming in the small and large intestine. The common bile duct connects to the pancreas and the gall bladder, and a T tube rather than an NG tube would be used to collect bile drainage from the common bile duct.

A client with cirrhosis of the liver develops ascites. Which of the following orders would the nurse expect? A. Restrict fluid to 1000 mL per day. B. Ambulate 100 ft. three times per day. C. High-sodium diet. D. Maalox 30 ml P.O. BID.

Correct answer: A Fluid restriction is a primary treatment for ascites. Restricting fluids decreases the amount of fluid present in the body, thereby decreasing the fluid that accumulates in the peritoneal space. A high sodium diet would increase fluid retention. Physical activities are usually restricted until ascites is relieved. Loop diuretics (such as furosemide) are usually ordered, and Maalox® (a bismuth subsalicylate) may interfere with the action of the diuretics.

The nurse is doing teaching with the family of a client with liver failure. Which of the following foods should the nurse advise them to limit in the client's diet? A. Meats and beans. B. Butter and gravies. C. Potatoes and pasta. D. Cakes and pastries.

Correct answer: A Meats and beans are high-protein foods and are restricted with liver failure. In liver failure, the liver is unable to metabolize protein adequately, causing protein by-products to build up in the body rather than be excreted. This causes problems such as hepatic encephalopathy (neurologic syndrome that develops as a result of rising blood ammonia levels). Although other nutrients, such as fat and carbohydrates, may be regulated, it's most important to limit protein in the diet of the client with liver failure.

A client who recently underwent cranial surgery develops syndrome of inappropriate antidiuretic hormone (SIADH). Which of the following symptoms should the nurse anticipate? A. Edema and weight gain. B. Excessive urinary output. C. Fluid loss and dehydration. D. Low urine specific gravity.

Correct answer: A Syndrome of inappropriate antidiuretic hormone (SIADH) results in an abnormally high release of antidiuretic hormone, which causes water retention as serum sodium levels fall, leading to edema and weight gain. Because of fluid retention, urine output is low. Fluid is restricted to prevent fluid overload rather than replaced. As the urine becomes more concentrated, the specific gravity increases. Other symptoms include nausea, vomiting, seizures, altered mentation, and coma. SIADH is most common with diseases of the hypothalamus but can also occur with heart failure, Guillain-Barré syndrome, meningitis, encephalitis, head trauma, or brain tumors. It may also be triggered by medications.

Hypothyroid s/sx mimic what other d/o?

Depression

The adrenal medulla and cortex synthesizes what hormones?

Medulla = the catecholamines, epinephrine and norepinephrine. Adrenal cortex = glucocorticoids, mineral corticoids, & sex hormones

The nurse is caring for a client with cirrhosis of the liver. The client has developed ascites and requires a paracentesis. Which of the following symptoms is associated with ascites and should be relieved by the paracentesis? A. Pruritus. B. Dyspnea. C. Jaundice. D. Peripheral neuropathy.

Correct answer: B Ascites (fluid buildup in the abdomen) puts pressure on the diaphragm, resulting in difficulty breathing and dyspnea. Paracentesis (surgical puncture of the abdominal cavity to aspirate fluid) is done to remove fluid from the abdominal cavity and thus reduce pressure on the diaphragm in order to relieve the dyspnea. Pruritus, jaundice, and peripheral neuropathy are signs of cirrhosis that aren't relieved or treated by paracentesis.

A 37-year-old forklift operator presents with shakiness, sweating, anxiety, and palpitations and tells the nurse he has type 1 diabetes mellitus. Which of the follow actions should the nurse do first? A. Inject 1 mg of glucagon subcutaneously. B. Administer 50 mL of 50% glucose I.V. C. Give 4 to 6 oz (118 to 177 mL) of orange juice. D. Give the client four to six glucose tablets.

Correct answer: C Because the client is awake and complaining of symptoms, the nurse should first give him 15 grams of carbohydrate to treat hypoglycemia. This could be 4 to 6 oz of fruit juice, five to six hard candies such as Lifesavers, or 1 tablespoon of sugar. When a client has worsening symptoms of hypoglycemia or is unconscious, treatment includes 1 mg of glucagon subcutaneously or intramuscularly, or 50 mL of 50% glucose I.V. The nurse may also give two to three glucose tablets for a hypoglycemic reaction.

"Which of the following factors are risks for the development of diabetes mellitus? (Select all that apply.) "a) Age over 45 years b) Overweight with a waist/hip ratio >1 c) Having a consistent HDL level above 40 mg/dl d) Maintaining a sedentary lifestyle

Correct: a,b,d"Rationale: Aging results in reduced ability of beta cells to respond with insulin effectively. Overweight with waist/hip ratio increase is part of the metabolic syndrome of DM II. There is an increase in atherosclerosis with DM due to the metabolic syndrome and sedentary lifestyle.

cortex, medulla

Corticoids and sex hormones are secreted from the _______ of the adrenal glands, while the catecholamines are secreted from the _________

When Hypothyroid is present at birth it's called:

Cretinism

Although they don't know why people with hypothyroid get Coronary Artery Disease there is one interesting connection:

Cretinism is Congenital Hypothyroid - from birth - very dangerous and can lead to slowed mental and physical development if undetected.

Myxedema present at birth is known as _______. If a patient states thatt a baby doesn't ever cry, wake up, rarely eats and is the best baby in the world, how would you respond?

Cretinism. Dx w/ myxedema!

"A client is diagnosed with diabetes mellitus. Which data collection finding best supports a nursing diagnosis of Ineffective coping related to diabetes mellitus?

Crying whenever diabetes is mentioned

What syndrome (exogenous)/disease (endogenous) involves the body having too much steroids?

Cushing's syndrome/disease

During a teaching session, the nurse tells the client that 50% to 60% of daily calories should come from carbohydrates. What should the nurse say about the types of carbohydrates that can be eaten? a. Simple carbohydrates are absorbed more rapidly than complex carbohydrates. b. Simple sugars cause a rapid spike in glucose levels and should be avoided c. Simple sugars should never be consumed by someone with diabetes. d. Try to limit simple sugars to between 10% and 20% of daily calories.

D It is recommended that carbohydrates provide 50% to 60% of the daily calories. Approximately 40% to 50% should be from complex carbohydrates. The remaining 10% to 20% of carbohydrates could be from simple sugars. Research provides no evidence that carbohydrates from simple sugars are digested and absorbed more rapidly than are complex carbohydrates, and they do not appear to affect blood sugar control.

A hospitalized diabetic patient receives 12 U of regular insulin mixed with 34 U of NPH insulin at 7:00 AM. The patient is away from the nursing unit for diagnostic testing at noon, when lunch trays are distributed. The most appropriate action by the nurse is to a. save the lunch tray to be provided upon the patient's return to the unit. b. call the diagnostic testing area and ask that a 5% dextrose IV be started. c. ensure that the patient drinks a glass of milk or orange juice at noon in the diagnostic testing area. d. request that the patient be returned to the unit to eat lunch if testing will not be completed promptly.

D Rationale: Consistency for mealtimes assists with regulation of blood glucose, so the best option is for the patient to have lunch at the usual time. Waiting to eat until after the procedure is likely to cause hypoglycemia. Administration of an IV solution is unnecessarily invasive for the patient. A glass of milk or juice will keep the patient from becoming hypoglycemic but will cause a rapid rise in blood glucose because of the rapid absorption of the simple carbohydrate in these items.

A patient recovering from DKA asks the nurse how acidosis occurs. The best response by the nurse is that a. insufficient insulin leads to cellular starvation, and as cells rupture they release organic acids into the blood. b. when an insulin deficit causes hyperglycemia, then proteins are deaminated by the liver, causing acidic by-products. c. excess glucose in the blood is metabolized by the liver into acetone, which is acidic. d. an insulin deficit promotes metabolism of fat stores, which produces large amounts of acidic ketones.

D Rationale: Ketoacidosis is caused by the breakdown of fat stores when glucose is not available for intracellular metabolism. The other responses are inaccurate.

A patient with type 1 diabetes has received diet instruction as part of the treatment plan. The nurse determines a need for additional instruction when the patient says, a. "I may have an occasional alcoholic drink if I include it in my meal plan." b. "I will need a bedtime snack because I take an evening dose of NPH insulin." c. "I will eat meals as scheduled, even if I am not hungry, to prevent hypoglycemia." d. "I may eat whatever I want, as long as I use enough insulin to cover the calories."

D Rationale: Most patients with type 1 diabetes need to plan diet choices very carefully. Patients who are using intensified insulin therapy have considerable flexibility in diet choices but still should restrict dietary intake of items such as fat, protein, and alcohol. The other patient statements are correct and indicate good understanding of the diet instruction.

ADH-Antidiuretic hormone

Desmopressin (DDAVP) -Acts on kidneys & blood vessels, helps prevent the loss of water from the body by reducing urine output, raises BP by constricting blood vessels Tx-DI Vasopressin (Pitressin) -Helps prevent the loss of water from the body by reducing urine ouput.

A program of weight loss and exercise is recommended for a patient with impaired fasting glucose (IFG). When teaching the patient about the reason for these lifestyle changes, the nurse will tell the patient that a. the high insulin levels associated with this syndrome damage the lining of blood vessels, leading to vascular disease. b. although the fasting plasma glucose levels do not currently indicate diabetes, the glycosylated hemoglobin will be elevated. c. the liver is producing excessive glucose, which will eventually exhaust the ability of the pancreas to produce insulin, and exercise will normalize glucose production. d. the onset of diabetes and the associated cardiovascular risks can be delayed or prevented by weight loss and exercise.

D Rationale: The patient with IFG is at risk for developing type 2 diabetes, but this risk can be decreased with lifestyle changes. Glycosylated hemoglobin levels will not be elevated in IFG and the Hb A1C test is not included in prediabetes testing. Elevated insulin levels do not cause the damage to blood vessels that can occur with IFG. The liver does not produce increased levels of glucose in IFG

A patient screened for diabetes at a clinic has a fasting plasma glucose level of 120 mg/dl (6.7 mmol/L). The nurse will plan to teach the patient about a. use of low doses of regular insulin. b. self-monitoring of blood glucose. c. oral hypoglycemic medications. d. maintenance of a healthy weight.

D Rationale: The patient's impaired fasting glucose indicates pre-diabetes and the patient should be counseled about LIFESTYLE CHANGES to prevent the development of type 2 diabetes. The patient with prediabetes does not require insulin or the oral hypoglycemics for glucose control and does not need to self-monitor blood glucose.

A health care provider who has not been immunized for hepatitis B is exposed to the hepatitis B virus (HBV) through a needle stick from an infected patient. The infection control nurse informs the individual that treatment for the exposure should include a. baseline hepatitis B antibody testing now and in 2 months. b. active immunization with hepatitis B vaccine. c. hepatitis B immune globulin (HBIG) injection. d. both the hepatitis B vaccine and HBIG injection.

D Rationale: The recommended treatment for exposure to hepatitis B in unvaccinated individuals is to receive both HBIG and the hepatitis B vaccine, which would provide temporary passive immunity and promote active immunity. Antibody testing may also be done, but this would not provide protection from the exposure.

19. A hospitalized diabetic patient receives 12 U of regular insulin mixed with 34 U of NPH insulin at 7:00 AM. The patient is away from the nursing unit for diagnostic testing at noon, when lunch trays are distributed. The most appropriate action by the nurse is to a. save the lunch tray to be provided upon the patient's return to the unit. b. call the diagnostic testing area and ask that a 5% dextrose IV be started. c. ensure that the patient drinks a glass of milk or orange juice at noon in the diagnostic testing area. d. request that the patient be returned to the unit to eat lunch if testing will not be completed promptly.

D Rationale: Consistency for mealtimes assists with regulation of blood glucose, so the best option is for the patient to have lunch at the usual time. Waiting to eat until after the procedure is likely to cause hypoglycemia. Administration of an IV solution is unnecessarily invasive for the patient. A glass of milk or juice will keep the patient from becoming hypoglycemic but will cause a rapid rise in blood glucose because of the rapid absorption of the simple carbohydrate in these items. Cognitive Level: Analysis Text Reference: p. 1268 Nursing Process: Implementation NCLEX: Physiological Integrity

27. A patient recovering from DKA asks the nurse how acidosis occurs. The best response by the nurse is that a. insufficient insulin leads to cellular starvation, and as cells rupture they release organic acids into the blood. b. when an insulin deficit causes hyperglycemia, then proteins are deaminated by the liver, causing acidic by-products. c. excess glucose in the blood is metabolized by the liver into acetone, which is acidic. d. an insulin deficit promotes metabolism of fat stores, which produces large amounts of acidic ketones.

D Rationale: Ketoacidosis is caused by the breakdown of fat stores when glucose is not available for intracellular metabolism. The other responses are inaccurate. Cognitive Level: Application Text Reference: pp. 1278-1279 Nursing Process: Implementation NCLEX: Physiological Integrity

9. A patient with type 1 diabetes has received diet instruction as part of the treatment plan. The nurse determines a need for additional instruction when the patient says, a. "I may have an occasional alcoholic drink if I include it in my meal plan." b. "I will need a bedtime snack because I take an evening dose of NPH insulin." c. "I will eat meals as scheduled, even if I am not hungry, to prevent hypoglycemia." d. "I may eat whatever I want, as long as I use enough insulin to cover the calories."

D Rationale: Most patients with type 1 diabetes need to plan diet choices very carefully. Patients who are using intensified insulin therapy have considerable flexibility in diet choices but still should restrict dietary intake of items such as fat, protein, and alcohol. The other patient statements are correct and indicate good understanding of the diet instruction. Cognitive Level: Application Text Reference: p. 1268 Nursing Process: Evaluation NCLEX: Physiological Integrity

37. The nurse teaches the diabetic patient who rides a bicycle to work every day to administer morning insulin into the a. thigh. b. buttock. c. arm. d. abdomen.

D Rationale: Patients should be taught not to administer insulin into a site that will be exercised because exercise will increase the rate of absorption. The thigh, buttock, and arm are all exercised by riding a bicycle. Cognitive Level: Application Text Reference: p. 1262 Nursing Process: Implementation NCLEX: Physiological Integrity

8. A college student who has type 1 diabetes normally walks each evening as part of an exercise regimen. The student now plans to take a swimming class every day at 1:00 PM. The clinic nurse teaches the patient to a. delay eating the noon meal until after the swimming class. b. increase the morning dose of neutral protamine Hagedorn (NPH) insulin on days of the swimming class. c. time the morning insulin injection so that the peak occurs while swimming. d. check glucose level before, during, and after swimming.

D Rationale: The change in exercise will affect blood glucose, and the patient will need to monitor glucose carefully to determine the need for changes in diet and insulin administration. Because exercise tends to decrease blood glucose, patients are advised to eat before exercising. Increasing the morning NPH or timing the insulin to peak during exercise may lead to hypoglycemia, especially with the increased exercise. Cognitive Level: Application Text Reference: p. 1269 Nursing Process: Implementation NCLEX: Health Promotion and Maintenance

4. A program of weight loss and exercise is recommended for a patient with impaired fasting glucose (IFG). When teaching the patient about the reason for these lifestyle changes, the nurse will tell the patient that a. the high insulin levels associated with this syndrome damage the lining of blood vessels, leading to vascular disease. b. although the fasting plasma glucose levels do not currently indicate diabetes, the glycosylated hemoglobin will be elevated. c. the liver is producing excessive glucose, which will eventually exhaust the ability of the pancreas to produce insulin, and exercise will normalize glucose production. d. the onset of diabetes and the associated cardiovascular risks can be delayed or prevented by weight loss and exercise.

D Rationale: The patient with IFG is at risk for developing type 2 diabetes, but this risk can be decreased with lifestyle changes. Glycosylated hemoglobin levels will not be elevated in IFG and the Hb A1C test is not included in prediabetes testing. Elevated insulin levels do not cause the damage to blood vessels that can occur with IFG. The liver does not produce increased levels of glucose in IFG. Cognitive Level: Application Text Reference: p. 1255 Nursing Process: Implementation NCLEX: Physiological Integrity

2. A patient screened for diabetes at a clinic has a fasting plasma glucose level of 120 mg/dl (6.7 mmol/L). The nurse will plan to teach the patient about a. use of low doses of regular insulin. b. self-monitoring of blood glucose. c. oral hypoglycemic medications. d. maintenance of a healthy weight.

D Rationale: The patient's impaired fasting glucose indicates prediabetes and the patient should be counseled about lifestyle changes to prevent the development of type 2 diabetes. The patient with prediabetes does not require insulin or the oral hypoglycemics for glucose control and does not need to self-monitor blood glucose. Cognitive Level: Application Text Reference: p. 1255 Nursing Process: Planning NCLEX: Physiological Integrity

A client in the recovery room after a thyroidectomy is asked frequentlyh by the nurse to speak. The rationale for the nurse continuing to evaluate the client's voice is to.

Detect spasms or edema in the area of the vocal cords.

The nurse is caring for patients in the student health center. A patient confides to the nurse that the patient's boyfriend informed her that he tested positive for Hepatitis B. Which of the following responses by the nurse is BEST? a. "That must have been a real shock to you" b. "You should be tested for Hepatitis B" c. "You'll receive the Hepatitis B immune globulin HBIG d. "Have you had unprotected sex with your boyfriend"

D. Hepatitis B is transmitted through parenteral drug abuse and sexual contact. Determine exposure before implementing.

When taking a health history, the nurse screens for manifestations suggestive of diabetes type I. Which of the following manifestations are considered the primary manifestations of diabetes type I and would be most suggestive of diabetes type I and require follow-up investigation? a. Excessive intake of calories, rapid weight gain, and difficulty losing weight b. Poor circulation, wound healing, and leg ulcers, c. Lack of energy, weight gain, and depression d. An increase in three areas: thirst, intake of fluids, and hunger

D. An increase in three areas: thirst, intake of fluids, and hunger The primary manifestations of diabetes type I are polyuria (increased urine output), polydipsia (increased thirst), polyphagia (increased hunger).

You are doing some teaching with a client who is starting on a sulfonylurea antidiabetic agent. The client mentions that he usually has a couple of beers each night and takes an aspirin each day to prevent heart attack and/or strokes. Which of the following responses would be best on the part of the nurse? a. As long as you only drink two beers and take one aspirin, this should not be a problem b. The aspirin is alright but you need to give up drinking any alcoholic beverages c. Aspirin and alcohol will cause the stomach to bleed more when on a sulfonylurea drug d. Taking alcohol and/or aspirin with a sulfonylurea drug can cause development of hypoglycemia

D. Taking alcohol and/or aspirin with a sulfonylurea drug can cause development of hypoglycemia Alcohol and/or aspirin taken with a sulfonylurea can cause development of hypoglycemia.

An external insulin pump is prescribed for a client with diabetes mellitus and the client asks the nurse about the functioning of the pump. The nurse bases the response on the information that the pump: a) is timed to release programmed doses of regular or NPH insulin into the bloodstream at specific intervals b) continuously infuses small amounts of NPH insulin into the bloodstream while regularly monitoring blood glucose levels c) is surgically attached to the pancreas and infuses regular insulin into the pancreas, which in turn releases the insulin into the bloodstream d) gives a small continuously dose of regular insulin subcutaneously, and the client can self-administer a bolus with an additional dose form the pump before each meal

D.) gives a small continuously dose of regular insulin subcutaneously, and the client can self-administer a bolus with an additional dose form the pump before each meal An insulin pump provides a small continuous dose of regular insulin subcutaneously throughout the day and night, and the client can self-administer a bolus with an additional dose from the pump before each meal as needed. Regular insulin is used in an insulin pump. An external pump is not attached surgically to the pancreas.

"A frail elderly patient with a diagnosis of type 2 diabetes mellitus has been ill with pneumonia. The cliet's intake has been very poor, and she is admitted to the hospital for observation and management as needed. What is the most likely problem with this patient? "A. Insulin resistance has developed. B. Diabetic ketoacidosis is occuring. C. Hypoglycemia unawareness is developing. D. Hyperglycemic hyperosmolar non-ketotic coma.

D.Illness, especially with the frail elderly patient whose appetite is poor, can result in dehydration and HHNC. Insulin resisitance is inidcated by a daily insulin requirement of 200 units or more. Diabetic ketoacidosis, an acute metabolic condition, usually is caused by absent or markedly decreased amounts of insulin.

Prediabetes is associated with all of the following except: a. Increased risk of developing type 2 diabetes b. Impaired glucose tolerance c. Increased risk of heart disease and stroke d. Increased risk of developing type 1 diabetes

D: Increased risk of developing type 1 diabetes Persons with elevated glucose levels that do not yet meet the criteria for diabetes are considered to have prediabetes and are at increased risk of developing type 2 diabetes. Weight loss and increasing physical activity can help people with prediabetes prevent or postpone the onset of type 2 diabetes.

"An obese Hispanic client, age 65, is diagnosed with type 2 diabetes mellitus. Which statement about diabetes mellitus is true?

Diabetes mellitus is three times more common in Hispanics than in Blacks or Whites.

18. The benefits of using an insulin pump include all of the following except: "a. By continuously providing insulin they eliminate the need for injections of insulin b. They simplify management of blood sugar and often improve A1C c. They enable exercise without compensatory carbohydrate consumption d. They help with weight loss

D: Using an insulin pump has many advantages, including fewer dramatic swings in blood glucose levels, increased flexibility about diet, and improved accuracy of insulin doses and delivery; however, the use of an insulin pump has been associated with weight gain.

"The nurse is planning care for a 52-year-old male client in acute addisonian crisis. Which nursing diagnosis should receive the highest priority?

Decreased cardiac output

Why are beta-blockers used in hyperthyroid patients (3)? In other words, what do they decrease?

Decreased myocardial contractility, decreased cardiac output, decreases HR/BP.

"A client is admitted with a serum glucose level of 618 mg/dl. The client is awake and oriented, with hot, dry skin; a temperature of 100.6° F (38.1° C); a heart rate of 116 beats/minute; and a blood pressure of 108/70 mm Hg. Based on these findings, which nursing diagnosis takes highest priority?

Deficient fluid volume related to osmotic diuresis

"A client with a serum glucose level of 618 mg/dl is admitted to the facility. He's awake and oriented. He has hot, dry skin and the following vital signs: a temperature of 100.6° F (38.1° C), a heart rate of 116 beats/minute, and a blood pressure of 108/70 mm Hg. Based on these findings, which nursing diagnosis takes highest priority?

Deficient fluid volume related to osmotic diuresis

"A client with long-standing type 1 diabetes is admitted to the hospital with unstable angina pectoris. After the client's condition stabilizes, the nurse evaluates the diabetes management regimen. The nurse learns that the client sees the physician every 4 weeks, injects insulin after breakfast and dinner, and measures blood glucose before breakfast and at bedtime. Consequently, the nurse should help formulate a nursing diagnosis of:

Deficient knowledge.

"A nurse is teaching a group of certified nursing assistants (CNAs) about blood glucose monitoring. Which finding indicates that the CNA understands how to use a blood glucose meter?

Demonstrating correct technique

"A client becomes upset when the physician diagnoses diabetes mellitus as the cause of his signs and symptoms. The client tells the nurse, ""This must be a mistake. No one in my family has ever had diabetes."" Based on this statement, the nurse suspects the client is using which coping mechanism?

Denial

Progestins

Depo-Prohera (shot) Primpro Megace (megestrol)-man made chemical similar to female progestron, used to treat loss of appetitie & wt loss in people with aids TX-advaced breast CA & endometrial CA

"When administering spironolactone (Aldactone) to a client who has had a unilateral adrenalectomy, the nurse should instruct the client about which possible adverse effect of the drug?

Menstrual irregularities

"The nurse is teaching a client with type 1 diabetes how to treat adverse reactions to insulin. To reverse a hypoglycemic reaction, the client ideally should ingest an oral carbohydrate. However, this treatment isn't always possible or safe. Therefore, the nurse should advise the client to keep which alternate treatment on hand?

Glucagon

Name the signs and symptoms of Cushing's Disease according to excess glucocorticoids (hint: think of 4 main functions & picturesss) (8)/sex hormones(3)/mineralcorticoids (3).

Glucocorticoids - growth arrest, depresssion to psychoses, thin extremities/skin, hyperglycemia, moon face, truncal obesity, buffalo hump.---Sex hormones: oil skin/acne, poor sex drive (libido), women with male traits.---Mineralcorticoids: high BP, CHF, weight gain

"The adrenal cortex is responsible for producing which substances?

Glucocorticoids and androgens

Cortisol is another fancy name for ________ or steroids. Thus, hyperocortisolism means too many ____.

Glucocorticoids. Steroids.

"A client tells the nurse that she has been working hard for the past 3 months to control her type 2 diabetes with diet and exercise. To determine the effectiveness of the client's efforts, which test should be performed?

Glycosylated hemoglobin level

A disease that causes hyperthyroidism is ________ disease. Name 6 out 10 s/s of hyperthyroidism (don't forget temp/GI/eyes/attention span).

Graves Disease. Nervousness, decreased attention span, irritable, increased sweat/heat, decreased weight, increased appetite, exophthalamos, GI = diarrhea, increased blood pressure, increased thyroid

"A middle-age female complains of anxiety, insomnia, weight loss, the inability to concentrate, and her eyes feeling ""gritty."" Thyroid function tests reveal the following: a thyroid-stimulating hormone (TSH) level of 0.02 units/ml, a thyroxine level of 20 g/dl, and a triiodothyronine level of 253 ng/dl. A 6-hour radioactive iodine uptake test showed a diffuse uptake of 85%. Based on these findings, the nurse would suspect:

Graves' disease.

"Which instructions should be included in the discharge teaching plan for a client after a thyroidectomy for Graves' disease?

Have regular follow-up care.

The nurse is caring for a client who is 8 hours post thyroid ectomy. What are important nursing interventions for this client?

Have the client speak every 2 hours to determine increasing level of hoarseness. Evaluate behind the neck for the presence of blood from the incision. Maintain the client ot semifowler's position. Check the incision for formation of a hematoma.

with too many mineralocorticoids, what may you see?

High BP, CHF, Weight gain and fluid volume overload

Insulin

Hormone produced by the pancreas which is central to regulating carbohydrate & fat metabolism in body. -Regular (rapid onset of action, short duration) begins to reduce bs with in 30 mins, peak 1-3 hours, lasts 6-8 hours. -NPH (slower onset, longer duration) onset 2 hours, peak 4-12, duration 18-26 hours. -Lantus

Thyroxine

Hormone which acts as a catalyst; influences metabolic rate, growth, and development

calcium

Hydration and diuretics can enhance excretion of ______________ in patients with hyperparathyroidism

"A client with a history of Addison's disease and flulike symptoms accompanied by nausea and vomiting over the past week is brought to the facility. The client's wife reports that she noticed that he acted confused and was extremely weak when he woke up in the morning. The client's blood pressure is 90/58 mm Hg, his pulse is 116 beats/minute, and his temperature is 101° F (38.3° C). A diagnosis of acute adrenal insufficiency is made. Which of the following would the nurse expect to administer by I.V. infusion?

Hydrocortisone

"Which of the following would the nurse expect to find in a client diagnosed with hyperparathyroidism?

Hypercalcemia

Other names for hyperparathyroidism would be (2). Other names for hypoparathyroidism would be (2).

Hypercalcemia, hypophosphatemia.---hypocalcemia, hyperphosphatemia.

"The nurse is caring for a client in acute addisonian crisis. Which laboratory data would the nurse expect to find?

Hyperkalemia

The majority of the S/S in Addison's disease are a result of ___________ initially. What are these symptoms again (think of progression)?

Hyperkalemia. Muscle twitching > muscle weakness > muscle paralysis.

Aldosteronism

Hypersecretion of aldosterone from adrenal cortex causing hypokalemia, hypernatremia, hypertension

Hyperparathyroidism

Hypersecretion of calcitonin resulting in hypercalcemia and hypophosphoremia causing loss of calcium from bones into serum, kidney stones and hyperuricemia, osteoporosis, muscle weakness, polyuria and polydipsia

Cushing's disease

Hypersecretion of glucocorticoids causing upper body obesity, moon face, poor skin integrity, osteoporosis, hyperglycemia, hypernatremia,hypokalemia, hirsutism, amenorrhea, elevated triglycerides, hypertension, and immunosuppression

Hyperthyroidism

Hypersecretion of thyroxine from immune system attacking thyroid gland causing anxiety, irritability, insomnia, tachycardia, tremors, diaphoresis, sensitivity to heat, weight loss, exophthalmos and photosensitivity, diarrhea,

"On the third day after a partial thyroidectomy, a client exhibits muscle twitching and hyperirritability of the nervous system. The client reports numbness and tingling of the mouth and fingertips. Suspecting a life-threatening electrolyte disturbance, the nurse notifies the surgeon immediately. Which electrolyte disturbance most commonly follows thyroid surgery?

Hypocalcemia

"What does a positive Chvostek's sign indicate?

Hypocalcemia

"A client with type 1 diabetes has a prescription for 5 units of U-100 regular insulin and 25 units of U-100 isophane insulin suspension (NPH) to be taken before breakfast. At about 4:30 p.m., the client experiences headache, sweating, tremor, pallor, and nervousness. What is the most probable cause of these signs and symptoms?

Hypoglycemia

Diabetes Insipidus

Hyposecretion of ADH causing polyuria, polydipsia, hypernatremia, weight loss, and dehydration

Addison's disease

Hyposecretion of adrenal cortex hormones (insufficiency of cortisol, aldosterone, and androgens)

Hypoparathyroidism

Hyposecretion of calcitonin resulting in hypocalcemia and hyperphosphatemia

Addisonian crisis involves severe _________ and ________ collapse.

Hypotension & Vascular

"A client is taking an oral antidiabetic agent, to treat type 2 diabetes. Which statement indicates the need for further client teaching about treatment of this disease?

I often skip lunch because I don't feel hungry.

"A client diagnosed with hyperosmolar hyperglycemic nonketotic syndrome (HHNS) is stabilized and prepared for discharge. When preparing the client for discharge and home management, which statement indicates that the client understands her condition and how to control it?

I should avoid becoming dehydrated and pay attention to my need to urinate, drink, or eat more than usual.

"A 78-year-old client with type 2 diabetes needs a kidney transplant. The client's daughter volunteers to donate a kidney, but the client voices concerns about her daughter's health to the nurse. Which response by the nurse is appropriate?

I'll notify your physician of your concerns and see if he can discuss the procedures with you.

"A client is evaluated for type 1 diabetes. Which client comment correlates best with this disorder?

I'm thirsty all the time. I just can't get enough to drink.

"Which nursing diagnosis takes highest priority for a client with hyperthyroidism?

Imbalanced nutrition: Less than body requirements related to thyroid hormone excess

Anti-Thyroid

Increase effect on anticoagulant, antidepressant, decrease glucose meds, dijoxin, and dilantin. Methimazole (Tapazole)-hyper, sudden withdrawl cuase thyroid crisis, don't increase K PTU (propythiouracil)-hyper, thyrotoxic crisis, watch for iodine, alter effectivness Radioactive Iodine-hyper Potassium Iodide

"For a client with hyperglycemia, which data collection finding best supports a nursing diagnosis of Deficient fluid volume?

Increased urine osmolarity

glucagon

Increases blood glucose by converting glycogen to glucose

"When instructing a client diagnosed with hyperparathyroidism about diet, the nurse should stress the importance of which action?

Increasing fluid intake

sex hormones

Influence development of sexual characteristics (androgens, estrogens)

"A client with type 1 diabetes is admitted to an acute care facility with diabetic ketoacidosis. To correct this acute diabetic emergency, which measure should the health care team take first?

Initiate fluid replacement therapy.

"A client, age 23, is diagnosed with type 1 diabetes. The physician prescribes 15 units of U-100 regular insulin and 35 units of U-100 isophane insulin suspension (NPH) to be taken before breakfast. The nurse checks the medication order, assembles the appropriate equipment, washes her hands, rotates the NPH insulin vial, puts on disposable gloves, and cleans the stoppers. To draw the two insulin doses into the single U-100 insulin syringe, which sequence should the nurse use?

Inject 35 units of air into the NPH vial; inject 15 units of air into the regular insulin vial and withdraw 15 units of regular insulin; and withdraw 35 units of NPH.

transsphenoidal hypophysectomy

Microsurgery in which an incision is made at the junction of the gums and upper lip. A surgical microscope is advanced and a special surgical instrument is used to excise all or part of the pituitary gland.

"The nurse explains to a client that she will administer his first insulin dose in his abdomen. How does absorption at the abdominal site compare to absorption at other sites?

Insulin is absorbed more rapidly at abdominal injection sites than at other sites.

What does Potassium Iodide (SSKI) & Strong iodine solution (Lugol's solution) do for hyperthyroid patients? What should be given with these iodine compounds? What about antithyroids?

Iodine compounds decrease the size and the vascularity of the gland - this is especially good for preop patient to decrease bleeding. Give iodine compounds in milk, juice, and use a straw - because iodine stains the teeth. ----Antithyroids inhibit the production of TH, therefore making the client euthyroid (aka normal thyroid).

Glipizide (Glucortrol)

Is an oral hypoglycemic agent administered to decrease the serum glucose level & the signs & symptoms of hyperglycemia.

What is the somogyi phenomenon? What is the treatment (2)?

It is when the patient has normal BG levels at night, but drops in the early morning hrs (2-3am), client's body attempts to compensate by producing counter-regulatory hormones to increase BG resulting in hyperglycemia. Decrease intermediate acting insulin (NPH insulin, Lente insulin) and increase bedtime snack.

"A client with type 1 diabetes has a highly elevated glycosylated hemoglobin (Hb) test result. In discussing the result with the client, the nurse would be most accurate in stating:

It tells us about your sugar control for the last 3 months.

"While reviewing the food diary of a client with type 2 diabetes, a nurse notices that the client typically skips breakfast. Which instruction by the nurse would be helpful for this client.

It's important to maintain a stable blood sugar throughout the day. Can I help you devise a plan so you can eat breakfast each day?

What is the rule when monitoring the usage of mineralcorticoids, such as Fludrocortisone?

Keep weight within + or - 2 lbs of their normal weight

thyroid storm

LIFE THREATENING crisis of uncontrolled hyperthyroidism caused by the release into the bloodstream of increased amount of thyroid hormone; , associated with: .

Addisons crisis

LIFE THREATENING triggered by surgey, acute systemic illness trauma, or abrupt withdrawl of long therm corticosteroids therapy

steroids

Large doses of ________ should be given at 0800 to simulate normal excretion by the body (2/3 morning and 1/3 night)

"The nurse teaches a client with newly diagnosed hypothyroidism about the need for thyroid hormone replacement therapy to restore normal thyroid function. Which thyroid preparation is the agent of choice for thyroid hormone replacement therapy?

Levothyroxine (Synthroid)

Thyroid

Levothyroxine (synthroid)-hyper, hormone replacement Liothyronine (cytomenl)t3-hypo Liotrix (thyrolar)-hypo

(Florinef) Fludocortisone Acetate

Long lasting oral med with mineralcorcoid & mod glucorticoid activity. -prescribed for long term tx addisons disease -help restore electrolyte balance

"A client with hyperparathyroidism declines surgery and is to receive hormone replacement therapy with estrogen and progesterone. Which instruction would be most important to include in the client's teaching plan?

Maintain a moderate exercise program.

"Which intervention is the most critical for a client with myxedema coma?

Maintaining a patent airway

"For a client with Graves' disease, which nursing intervention promotes comfort?

Maintaining room temperature in the low-normal range

Hyperthyroid s/sx mimic what other d/o?

Manic

Synthroid

Medication to treat hypothyroidism

"A client is prescribed prednisone (Deltasone) daily. Which statement best explains why the nurse should instruct the client to take this drug in the morning?

Morning administration of prednisone mimics the body's natural corticosteroid secretion pattern.

"Following a unilateral adrenalectomy, the nurse would assess for hyperkalemia shown by which of the following?

Muscle weakness

Hypothyroid is aka:

Myxedema

"A client receiving thyroid replacement therapy develops the flu and forgets to take her thyroid replacement medicine. The nurse understands that skipping this medication will put the client at risk for developing which life-threatening complication?

Myxedema coma

"Which sign suggests that a client with the syndrome of inappropriate antidiuretic hormone (SIADH) secretion is experiencing complications?

Neck vein distention

Hyperthyroid s/sx:

Nervous, Weight is down, sweaty and hot, exophthamos (irreversable), attention span decreases, appetite increases, irritable, GI is fast, B/P is elevated, thyroid is bigger

"Which of the following signs and symptoms would be seen in a client experiencing hypoglycemia?

Nervousness, diaphoresis, and confusion

"A family member is observed looking at the blood glucose flow sheet for a client in the next bed. Which of the following actions would be an appropriate measure for the nurse to take?

Notify the charge nurse about this breach in the client's personal health information.

"A client with type 2 diabetes hasn't received insulin coverage for his afternoon blood glucose levels for 2 days. After further investigation, a nurse discovers that the afternoon blood glucose levels were phoned in from the laboratory but weren't documented in the client's medical record. What should the nurse do with this information?

Notify the physician and complete an incident report.

"A nurse is caring for a client with type 1 diabetes, who underwent a right hemicolectomy for colon cancer the day before. A physician prescribes the following sliding scale of regular insulin coverage every 6 hours for the client. Which action should the nurse take if the client's glucose level is 181 mg/dl?

Notify the physician.

"The nurse is caring for a client who had a thyroidectomy and is at risk for hypocalcemia. What should the nurse do?

Observe for muscle twitching and numbness or tingling of the lips, fingers, and toes.

With sex hormone overload you may see

Oily skin/acne, women with male traits and poor sex drives or labido

1-2 hr

Onset of action of intermediate insulins

5-15 min

Onset of action of rapid acting insulins

30-60 min

Onset of action of short acting insulins

1-4 hr

Onset of long-lasting insulins

"A nurse administers glucagon to her diabetic client and then monitors the client for adverse drug reactions and interactions. Which type of drug interacts adversely with glucagon?

Oral anticoagulants

Long Acting Insulin

PM Lantus -onset: 1-1.5 hours -peak: none -duration: 20-24 hours

S/S of hypoparathyroidism----Not enough ____. Serum calc is _____. Serum phos is _____. Other s/s? Tx?

PTH > Down > Up. Other s/s: patient is not sedated. Tx 1.) IV calcium 2.) phosphorus binding drugs (phoslo/calcium acetate).

S/S of hyperparathyroidism---Too much _________. Serum calcium is _______. Serum phos is _____. Other s/s? Treatment?

PTH > UP > DOWN > hypercalcemia = sedative properties. --Partial parathyroidectomy

The parathyroids secrete _________ which makes you pull calcium from the ______ and places it in the _____. Therefore, serum calcium will go ______.

PTH > bone > blood > up

4-8 hr

Peak action of intermediate insulins

1-2 hr

Peak of action of rapid acting insulins

2-4 hr

Peak of action of short acting insulins

List two problems that occur with the adrenal medulla. What would happen w/ these things symptoms - BP, Pulse, & flushing/diaphoretic?

Pheochrmocytoma & benign tumors that secrete epinephrine/norepinephrine. BP would increase, pulse would increase, flushing/diaphoretics would increase.

"Which of the following is an adverse reaction to glipizide (Glucotrol)?

Photosensitivity

"The nursing care for the client in addisonian crisis should perform which intervention?

Place the client in a private room.

Calcium

Postoperatively for thyroidectomy, check for _____________ deficiency due to removal of parathyroid

mineralocorticoids

Regulate sodium and electrolyte balance (aldosterone, corticosterone, deoxycorticosterone)

Which electrolyte replacement should the nurse anticipate being ordered by thehealth-care provider in the client diagnosed with DKA who has just been admitted tothe ICD? 1.Glucose. 2.)Potassium. 3.Calcium. 4.Sodium

Potassium"1.Glucose is elevated in DKA; therefore, theHCP would not be replacing glucose. 2.(CORRECT)-->The client in DKA loses potassium from increased urinary output, acidosis, cata-bolic state, and vomiting. Replacement isessential for preventing cardiac dysrhyth-mias secondary to hypokalemia. 3.Calcium is not affected in the client with DKA.4.The IV that is prescribed 0.9% normal salinehas sodium, but it is not specifically orderedfor sodium replacement. This is an isotonicsolution. TEST-TAKING HINT: Option "1" should be elim-inated because the problem with DKA iselevated glucose so the HCP would not bereplacing it. The test taker should use physiol-ogy knowledge and realize potassium is in thecell."

Estrogens

Premarin (cream)-used to treat the vag. symptoms of menapause such as dryness, burning, irritation & painful sex. Estradiol-used to treat symptoms of menopaus, prevention of osteoporosin in post menopausal, replacement with ovarian failure.

"A client with type 1 diabetes takes 15 units of isophane insulin suspension (Humulin N) before breakfast and 8 units before dinner. During a follow-up visit, the nurse reevaluates the client's knowledge about insulin therapy and self-administration skills and learns that the client is unaware that certain over-the-counter (OTC) preparations and other medications may interact with insulin. The nurse should advise the client to avoid which OTC preparations?

Preparations containing salicylates

"After undergoing a subtotal thyroidectomy, a client develops hypothyroidism. The physician prescribes levothyroxine (Synthroid), 25 mcg P.O. daily. For which condition is levothyroxine the preferred agent?

Primary hypothyroidism

Insulin

Promotes conversion of fatty acids into fat

Insulin

Promotes conversion of glucose to glycogen for storage

"The nurse should expect a client with hypothyroidism to report which health concerns?

Puffiness of the face and hands

iodine

Radioactive __________ treatment shrinks thyroid gland prior to surgery

The nurs is anssessing an 80 year old client with type 2 diabetes. Which assessment finding would indicate a problem of hyperglycemia associated with older adult clients:

Rapid, deep respirations at a rate of 36 breaths a minute: lethargyl, tachycardia

"A client who was diagnosed with type 1 diabetes 14 years ago is admitted to the medical-surgical unit with abdominal pain. On admission, the client's blood glucose level is 470 mg/dl. Which finding is most likely to accompany this blood glucose level?

Rapid, thready pulse

DX adrenal medulla problems. A ________ acid test is a ______ hour _______ specimen test look for increased levels of ____/_____ (catecholamines). What should be done with this test?

Vanylmandelic acid test (VMA) > 24 Hour > Urine > Epi/Norepinephrine. w/ a 24 hour urine you should throw away the first voiding and keep the last voiding.

"When caring for a client with diabetes insipidus, the nurse expects to administer:

Vasopressin (Pitressin Synthetic).

"A nurse administers bromocriptine (Parlodel) to a client diagnosed with acromegaly. After administering the medication, the nurse realizes that she gave the medication to the wrong client. What could have been done to prevent this error?

Verifying the client's identity on the identification band and medication administration record before providing the medication

An adult client has diabetes and is beginning glipizide for glucose control. What would be important for the nurse to discuss with the client?

Wearing sunscreen and avoiding direct sunlight

"Which of the following would the nurse expect to assess in an elderly client with Hashimoto's thyroiditis?

Weight gain, decreased appetite, and constipation

"The nurse is collecting data on a client with hyperthyroidism. What findings should the nurse expect?

Weight loss, nervousness, and tachycardia

hyperkalemia

What condition can cause a peaked T wave?

"A client who has diabetic retinopathy comes to the emergency department with injuries sustained from tripping on a lamp cord. Which of the following statements would be appropriate for the nurse to make?

When you are ready to go home, would you like a home health care nurse to come, too? She can check for things that can be done to make sure you don't fall again.

Rapid Acting Insulin

With meals Humalong -onset: 15-30 mins -peak: 30-90 mins -duration: 3-5 hours

Short Acting Insulin

With meals Regular -onset: 30min-1 hour -peak: 2-5 hours -duration: 5-8 hours ACHS Type 1

"A client with type 1 diabetes tells a nurse in the clinic, ""I sometimes skip my insulin dose in the morning so I won't gain back any of the weight I've lost."" Which of the following would be an appropriate response for the nurse to make to this client?

You are worried about your weight? There are safer ways to prevent weight gain.

"A client with adrenal hypofunction has been asked to participate in a research study for a new medication. The client is unsure about participating in the study. What would be an appropriate response for the nurse to make to this client?

You have the right to refuse to participate in the study.

"A client with primary diabetes insipidus is prescribed desmopressin (DDAVP). Which instruction should the nurse provide before the client is discharged?

You may not be able to use desmopressin nasally if you have nasal discharge or blockage.

"A client whose physical findings suggest a hyperpituitary condition undergoes an extensive diagnostic workup. Test results reveal a pituitary tumor, which necessitates a transsphenoidal hypophysectomy. The evening before the surgery, the nurse reviews preoperative and postoperative instructions given to the client earlier. Which postoperative instruction should the nurse emphasize?

You must avoid coughing, sneezing, and blowing your nose.

"During preoperative teaching for a client who will undergo a subtotal thyroidectomy, the nurse should include which statement?

You must avoid hyperextending your neck after surgery.

"A client has just been diagnosed with type 1 diabetes. When teaching the client and family how diet and exercise affect insulin requirements, the nurse should include which guideline?

You'll need less insulin when you exercise or reduce your food intake.

s/sof hyperthyroidism

^ heart rate; ^circulating TH; ^ metabolic rate Caddiac output & peripheral blood flow ^ ; Lipids are depleted & glucose tolerance is decreased

Chvostek's

__________ sign is muscle spasms and twitching around mouth, throat, and cheeks associated with hypocalcemia

Trousseau's

____________ sign is when the pressure from the blood pressure cuff induces muscle spasms in distal extremities with hypocalcemia

Beta-blockers

________________ manage tachycardia, anxiety, and tremors associated with hyperthyroidism

"When collecting data on a client with pheochromocytoma, a tumor of the adrenal medulla that secretes excessive catecholamine, the nurse is most likely to detect:

a blood pressure of 176/88 mm Hg.

Most common cause of Cushing's syndrome

a corticotropin-secreting pituitary adenoma

"A 35-year-old female client who complains of weight gain, facial hair, absent menstruation, frequent bruising, and acne is diagnosed with Cushing's syndrome. Cushing's syndrome is most likely caused by:

a corticotropin-secreting pituitary adenoma.

Cushing's disease

a high protein, low carb, low sodium diet with potassium supplement treats _________________

S/S hypothyroidism

a lowered basal metabolism rate with obesity, dry itchy skin, slow pulse, fluid retention and edema hoareness, sluggishness, and goiter, decreased b/p ,slowed heart rate, respiratory rate, fatigue, cold intolerance and weight gain.

The nurse assisting in the admission of a client with diabetic ketoacidosis will anticipate the physician ordering which of the following types of intravenous solution if the client cannot take fluids orally? a. 0.45% normal saline solution b. Lactated Ringer's solution c. 0.9 normal saline solution d. 5% dextrose in water (D5W)

a. 0.45% normal saline solution

The nurse assisting in the admission of a client with diabetic ketoacidosis will anticipate the physician ordering which of the following types of intravenous solution if the client cannot take any fluids orally? " a. 0.45% normal saline solution b. Lactated Ringer's solution c. 0.9 normal saline solution d. 5% dextrose in water (D5W)"

a. 0.45% normal saline solution Helps to hydrate patient and keep electrolyte levels balanced

Which of the following factors are risks for the development of diabetes mellitus? (Select all that apply.) a. Age over 45 years b. Overweight with a waist/hip ratio >1 c. Having a consistent HDL level above 40 mg/dl d. Maintaining a sedentary lifestyle

a. Age over 45 years b. Overweight with a waist/hip ratio >1 d. Maintaining a sedentary lifestyle Aging results in reduced ability of beta cells to respond with insulin effectively. Overweight with waist/hip ratio increase is part of the metabolic syndrome of DM II. There is an increase in atherosclerosis with DM due to the metabolic syndrome and sedentary lifestyle.

Blood sugar is well controlled when Hemoglobin A1C is: a. Below 7% b. Between 12%-15% c. Less than 180 mg/dL d. Between 90 and 130 mg/dL

a. Below 7% A1c measures the percentage of hemoglobin that is glycated and determines average blood glucose during the 2 to 3 months prior to testing. Used as a diagnostic tool, A1C levels of 6.5% or higher on two tests indicate diabetes. A1C of 6% to 6.5% is considered prediabetes.

Which laboratory test should a nurse anticipate a physician would order when an older person is identified as high-risk for diabetes mellitus? (Select all that apply.) a. Fasting Plasma Glucose (FPG) b. Two-hour Oral Glucose Tolerance Test (OGTT) c. Glycosylated hemoglobin (HbA1C) d. Finger stick glucose three times daily

a. Fasting Plasma Glucose (FPG) b. Two-hour Oral Glucose Tolerance Test (OGTT) When an older person is identified as high-risk for diabetes, appropriate testing would include FPG and OGTT. A FPG greater than 140 mg/dL usually indicates diabetes. The OGTT is to determine how the body responds to the ingestion of carbohydrates in a meal. HbA1C evaluates long-term glucose control. A finger stick glucose three times daily spot-checks blood glucose levels.

A patient is admitted to the hospital with a diagnosis of Cushing syndrome. On physical assessment of the patient, the nurse would expect to find a. HTN, peripheral edema, and petechiae b. weight loss, buffalo hump, and moon face with acne c. abdominal and buttock striae, truncal obesity, and hypotension d. anorexia, signs of dehydration, and hyper pigmentation of the skin

a. HTN, peripheral edema, and petechiae (rationale- The effects of glucocorticoid excess include weight gain from accumulation and redistribution of adipose tissue, sodium and water retention, glucose intolerance, protein wasting, loss of bone structure, loss of collagen, and capillary fragility. Clinical manifestations of corticosteroid deficiency include hypotension, dehydration, weight loss, and hyperpigmentation of the skin.)

After having a transverse colostomy constructed for colon cancer, discharge planning for home care would include teaching about the ostomy appliance. Information appropriate for this intervention would include: a. Instructing the client to report redness, swelling, fever, or pain at the site to the physician for evaluation of infection b. Nothing can be done about the concerns of odor with the appliance. c. Ordering appliances through the client's health care provider d. The appliance will not be needed when traveling.

a. Instructing the client to report redness, swelling, fever, or pain at the site to the physician for evaluation of infection Signs and symptoms for monitoring infection at the ostomy site are a priority evaluation for clients with new ostomies. The remaining actions are not appropriate. There are supplies avaliable for clients to help control odor that may be incurred because of the ostomy. Although a prescription for ostomy supplies is needed, you can order the supplies from any medical supplier. Dependent on the location and trainability of the ostomy, appliances are almost always worn throughout the day and when traveling

When caring for a patient with primary hyperaldosteronism, the nurse would question a physician's order for the use of a. Lasix b. amiloride (midamor) c. spironolactone (aldactone) d. aminoglutethimide (cytadren)

a. Lasix37 (rationale- hyperaldosteronism is an excess of aldosterone, which is manifested by sodium and water retention and potassium excretion. Lasix is a potassium-wasting diuretic that would increase the potassium deficiency. Aminoglutethimide blocks aldosterone synthesis; amiloride is apotassium-sparing diuretic; and spironolactone blocks mineralocorticoid receptors in the kidney, increasing secretion of sodium and water and retention of potassium.)

A client has just had surgery for colon cancer. Which of the following disorders might the client develop? a. Peritonitis b. Diverticulosis c. Partial bowel obstruction d. Complete bowel obstruction

a. Peritonitis Bowel spillage could occur during surgery, resulting in peritonitis. Complete or partial bowel obstruction may occur before bowel resection. Diverticulosis doesn't result from surgery or colon cancer.

Which of the following is accurate pertaining to physical exercise and type 1 diabetes mellitus? a. Physical exercise can slow the progression of diabetes mellitus. b. Strenuous exercise is beneficial when the blood glucose is high. c. Patients who take insulin and engage in strenuous physical exercise might experience hyperglycemia. d. Adjusting insulin regimen allows for safe participation in all forms of exercise.

a. Physical exercise can slow the progression of diabetes mellitus. Physical exercise slows the progression of diabetes mellitus, because exercise has beneficial effects on carbohydrate metabolism and insulin sensitivity. Strenuous exercise can cause retinal damage, and can cause hypoglycemia. Insulin and foods both must be adjusted to allow safe participation in exercise.

Which one of the following methods/techniques will the nurse use when giving insulin to a thin person? [Hint] A. Pinch the skin up and use a 90 degree angle B. Use a 45 degree angle with the skin pinched up C. Massage the area of injection after injecting the insulin D. Warm the skin with a warmed towel or washcloth prior to the injection

a. Pinch the skin up and use a 90 degree angle The best angle for a thin person is 90 degrees with the skin pinched up. The area is not massaged and it is not necessary to warm it.

Physician's orders for a client with acute pancreatitis include the following: strict NPO, NG tube to low intermittent suction. The nurse recognizes that these interventions will: a. Reduce the secretion of pancreatic enzymes b. Decrease the client's need for insulin c. Prevent secretion of gastric acid d. Eliminate the need for analgesia

a. Reduce the secretion of pancreatic enzymes

Radiation therapy is used to treat colon cancer before surgery for which of the following reasons? a. Reducing the size of the tumor b. Eliminating the malignant cells c. Curing the cancer d. Helping the bowel heal after surgery

a. Reducing the size of the tumor Radiation therapy is used to treat colon cancer before surgery to reduce the size of the tumor, making it easier to be resected. Radiation therapy isn't curative, can't eliminate the malignant cells (though it helps define tumor margins), can could slow postoperative healing.

Which of the following diets is most commonly associated with colon cancer? a. low fiber, high fat b. low fat high fiber c. low protein, high carb d. low carb, high protein

a. a. low fiber, high fat low fiber, high fat diet reduced motility and increases the chance of constipation. The metabolic end products of this type of diet are carcinogenic. A LOW FAT HIGH FIBER diet is recommended to help avoid colon cancer. Carbohydrates and protein aren't necessarily associated with colon cancer.

A patient newly diagnosed with Type I DM is being seen by the home health nurse. The doctors orders include: 1200 calorie ADA diet, 15 units NPH insulin before breakfast, and check blood sugar qid. When the nurse visits the patient at 5 pm, the nurse observes the man performing blood sugar analysis. The result is 50 mg/dL. The nurse would expect the patient to be a. confused with cold, clammy skin an pulse of 110 b. lethargic with hot dry dkin and rapid deep respirations c. alert and cooperative with BP of 130/80 and respirations of 12 d. short of breath, with distended neck veins and bounding pulse of 96.

a. confused with cold, clammy skin an pulse of 110 hypoglycemia

A patient with acromegaly is treated with a transphenoidal hypophysectomy. Postoperatively, the nurse a. ensures that any clear nasal drainage is tested for glucose b. maintains the patient flat in bed to prevent cerebrospinal fluid leak c. assists the patient with toothbrushing Q4H to keep the surgical area clean d. encourages deep breathing and coughing to prevent respiratory complications

a. ensures that any clear nasal drainage is tested for glucose (Rationale- a transphenoidal hypophysectomy involves entry into the sella turcica through an incision in the upper lip and gingiva into the floor of the nose and the sphenoid sinuses. Postoperative clear nasal drainage with glucose content indicates CSF leakage from an open connection to the brain, putting the patient at risk for meningitis. After surgery, the patient is positioned with the head elevated to avoid pressure on the sella turcica, coughing and straining are avoided to prevent increased ICP and CSF leakage, and although mouth care is required Q4H toothbrushing should not be performed for 7-10post sx.)

Preoperative instructions for the patient scheduled for a subtotal thyroidectomy includes teaching the patient a. how to support the head with the hands when moving b. that coughing should due avoided to prevent pressure on the incision c. that the head and neck will need to remain immobile until the incision heals d. that any tingling around the lips or in the fingers after surgery is expected and temporary

a. how to support the head with the hands when moving (rationale- to prevent strain on the suture line postoperatively, the head must be manually supported while turning and moving in bed, but range-of-motion exercise for the head and neck are also taught preoperatively to be gradually implemented after surgery. There is no contraindication for coughing and deep breathing, and they should be carrier out postoperatively. Tingling around the lips or fingers is a sign of hypocalcemia, which may occur if the parathyroid glands are inadvertently removed during surgery, and should be reported immediately.)

During care of a patient with syndrome of inappropriate ADH (SIADH), the nurse should a. monitor neurologic status Q2H or more often if needed b. keep the head of the bed elevated to prevent ADH release c. teach the patient receiving treatment with diuretics to restrict sodium intake d. notify the physician if the patient's blood pressure decreases more than 20mmHg from baseline

a. monitor neurologic status Q2H or more often if needed Rationale- the patient with SIADH has marked dilution hyponatremia and should be monitored for decreased neurologic function and convulsions every 2 hours. ADH release is reduced by keeping the head of the bed flat to increase left atrial filling pressure, and sodium intake is supplemented because of hyponatremia and sodium loss caused by diuretics. A reduction in blood pressure indicates a reduction in total fluid volume and is an expected outcome of treatment.)

he goal for pre-prandial blood glucose for those with Type 1 diabetes mellitus is: a. <80 mg/dl b. < 130 mg/dl c. <180 mg/dl d. <6%

b. < 130 mg/dl

The nurse determines that the patient in acute adrenal insufficiency is responding favorably to treatment when a. the patient appears alert and oriented b. the patient's urinary output has increased c. pulmonary edema is reduced as evidenced by clear lung sounds d. laboratory tests reveal serum elevations of K and glucose and a decrease in sodium

a. the patient appears alert and oriented (rationale- confusion, irritability, disorientation, or depressioni s often present in the patient with Addison's dz, and a positive response to therapy would be indicated by a return to alertness and orientation. Other indication of response to therapy would be a decreased urinary output, decreased serum potassium, and increased serum sodium and glucose. The patient with Addison's would be very dehydrated and volume-depleted and would not have pulmonary edema.)

treatment for SIADH

address low sodim and intercellular swelling resrtict fluid to 1L/day for 3 to 10 days treat with demeclocycline a tetracycline antibiotic

Cortisol is a hormone of the __________

adrenal cortex

What glands help you handle stress?

adrenal glands. The adrenal medulla secretes epi and norepi

Epi and nor epi mean:

adrenaline and noradrenaline :)

Neuropathy

affects clients with DM

Depression

agitation, irritability, poor memory, loss of appetite and neglect of one's appearance

Mineralocorticoids are:

aldosterone

addison's disease

aldosterone secretion is reduced

Methimazole (Tapazole)

an antithyroid treatment used for hyperthyroidism

After the onset of symptoms of addisons disease of hyperkalemia starting with muscle twitching to flaccid parralysis, then what other symptoms arise?

anorexia / nausea, hyperpigmentation - bronzing color of the skin and mucous membranes, decreased bowel sounds / GI upset (also known of with K+ increase), white patchy area of depigmented skin aka vitiligo, hypotension - b/c of loss of fluid, decrease Na, increased K+ and low sugar-hypoglycemia (remember insulin carries glucose and potassium into the cell so if you have hyperkalemia you can give insulin/glucose to carry k+ into the cell)

Patients with type 1 diabetes mellitus may require which of the following changes to their daily routine during times of infection? "a. no change b. less insulin c. more insulin d. oral diabetic agents"

answer C: during times of infection and illness diabetic patients may need even more insulin to compensate for increased blood glucose levels.

What has lots of sodium?

anything processed - fruit juice and processed meats and chicken broth - good for addisons

acromegaly

associated with hyperglycemia, hypertension, diaphoresis, peripheral neuropathy and joint pain

The nurse evaluates her teaching as effective when the pt recovering from acute renal failure states that he will do which of the follwing?

avoid takng durgs that may be nephrotoxic

SIADH (syndrome of inappropriate antidiuretic hormone)

is characterized by high levels of ADH in the absence of serum hypo-osmolality. caused by the etopic production of ADH by malignant tumors,( oat cell carcinoma of the lungs ,pancreatic carcinoma, leukemia, or hodgkins disease)

Which of the following symptoms is a client with colon cancer most likely to exhibit? a. A change in appetite b. A change in bowel habits c. An increase in body weight d. An increase in body temperature

b. A change in bowel habits The most common complaint of the client with colon cancer is a change in bowel habits. The client may have anorexia, secondary abdominal distention, or weight loss. Fever isn't associated with colon cancer.

"Polydipsia and polyuria related to diabetes mellitus are primarily due to: "a. The release of ketones from cells during fat metabolism b. Fluid shifts resulting from the osmotic effect of hyperglycemia c. Damage to the kidneys from exposure to high levels of glucose d. Changes in RBCs resulting from attachment of excessive glucose to hemoglobin

b. Fluid shifts resulting from the osmotic effect of hyperglycemia Rationale: The osmotic effect of glucose produces the manifestations of polydipsia and polyuria.

"Polydipsia and polyuria related to diabetes mellitus are primarily due to: "a. The release of ketones from cells during fat metabolism b. Fluid shifts resulting from the osmotic effect of hyperglycemia c. Damage to the kidneys from exposure to high levels of glucose d. Changes in RBCs resulting from attachment of excessive glucose to hemoglobin"

b. Fluid shifts resulting from the osmotic effect of hyperglycemia Rationale: The osmotic effect of glucose produces the manifestations of polydipsia and polyuria.

The nurse is planning dietary changes for a client following an episode of pancreatitis. Which diet is suitable for the client? a. Low calorie, low carbohydrate b. High calorie, low fat c. High protein, high fat d. Low protein, high carbohydrate

b. High calorie, low fat

While preparing the client for a colonoscopy, the nurse's responsibilities include: a. Explaining the risks and benefits of the exam b. Instructing the client about the bowel preparation prior to the test c, Instructing the client about medication that will be used to sedate the client d. Explaining the results of the exam

b. Instructing the client about the bowel preparation prior to the test The nurse is responsible for instructing the client about the bowel preparation prior to the test. Answers 1, 3, 4 are the physician's responsibility.

Which of these laboratory values noted by the nurse when reviewing the chart of a diabetic patient indicates the need for further assessment of the patient? a. Fasting blood glucose of 130 mg/dl b. Noon blood glucose of 52 mg/dl c. Glycosylated hemoglobin of 6.9% d. Hemoglobin A1C of 5.8%

b. Noon blood glucose of 52 mg/dl The nurse should assess the patient with a blood glucose level of 52 mg/dl for symptoms of hypoglycemia, and give the patient some carbohydrate-containing beverage such as orange juice. The other values are within an acceptable range for a diabetic patient.

One of the benefits of Glargine (Lantus) insulin is its ability to: a. Release insulin rapidly throughout the day to help control basal glucose. b. Release insulin evenly throughout the day and control basal glucose levels. c. Simplify the dosing and better control blood glucose levels during the day. d. Cause hypoglycemia with other manifestation of other adverse reactions.

b. Release insulin evenly throughout the day and control basal glucose levels. Glargine (Lantus) insulin is designed to release insulin evenly throughout the day and control basal glucose levels.

The guidelines for Carbohydrate Counting as medical nutrition therapy for diabetes mellitus includes all of the following EXCEPT: a. Flexibility in types and amounts of foods consumed b. Unlimited intake of total fat, saturated fat and cholesterol c. Including adequate servings of fruits, vegetables and the dairy group d. Applicable to with either Type 1 or Type 2 diabetes mellitus

b. Unlimited intake of total fat, saturated fat and cholesterol

A 1200-calorie diet and exercise are prescribed for a patient with newly diagnosed type 2 diabetes. The patient tells the nurse, "I hate to exercise! Can't I just follow the diet to keep my glucose under control?" The nurse teaches the patient that the major purpose of exercise for diabetics is to a. increase energy and sense of well-being, which will help with body image. b. facilitate weight loss, which will decrease peripheral insulin resistance. c. improve cardiovascular endurance, which is important for diabetics. d. set a successful pattern, which will help in making other needed changes.

b. facilitate weight loss, which will decrease peripheral insulin resistance. Rationale: Exercise is essential to decrease insulin resistance and improve blood glucose control. Increased energy, improved cardiovascular endurance, and setting a pattern of success are secondary benefits of exercise, but they are not the major reason.

Addisons Disease

is disorder resulting from distruction or disfunction of the adrenal cortex resulting in a defeciency in cortisol, aldosterone, adrenal androgens and skin pigmentation

When the patient with parathyroid disease experiences symptoms of hypocalcemia, a measure that can be used to temporarily raise serum calcium levels is to a. administer IV normal saline b. have the patient rebreathe in a paper bag c. administer Lasix as ordered d. administer oral phosphorous supplements

b. have the patient rebreathe in a paper bag (rationale- rebreathing in a paper bag promotes carbon dioxide retention in the blood, which lowers pH and creates an acidosis. An academia enhances the solubility and ionization of calcium, increasing the proportion of total body calcium available in physiologically active form and relieving the symptoms of hypocalcemia. Saline promotes calcium excretion, as does Lasix. Phosphate levels in the blood are reciprocal to calcium and an increase in phosphate promotes calcium excretion.)

An appropriate nursing intervention for the patient with hyperparathyroidism is to a. pad side rails as a seizure precaution b. increase fluid intake to 3000 to 4000ml/day c. maintain bed rest to prevent pathologic fractures d. monitor the patient for Trousseau's phenomenon or Chvostek's sign

b. increase fluid intake to 3000 to 4000ml/day (Rationale-A high fluid intake is indicated in hyperparathyroidism to dilute hypercalcemia and flush the kidneys so that calcium stone formation is reduced.)

A patient with SIADH is treated with water restriction and administration of IV fluids. The nurses evaluates that treatment has been effective when the patient experiences a. increased urine output, decreased serum sodium, and increased urine specific gravity b. increased urine output, increased serum sodium, and decreased urine specific gravity c. decreased urine output, increased serum sodium, and decreased urine specific gravity d. decreased urine output, decreased serum sodium, and increased urine specific gravity

b. increased urine output, increased serum sodium, and decreased urine specific gravity (rationale- the patient with SIADH has water retention with hyponatremia, decreased urine output and concentrated urine with high specific gravity. improvement in the patient's condition reflected by increased urine output, normalization of serum sodium, and more water in the urine, decreasing the specific gravity.)

The most important nursing intervention during the medical and surgical treatment of the patient with a pheochromocytoma is a. administering IV fluids b. monitoring blood pressure c. monitoring I&O and daily weights d. administering B-adrenergic blocking agents

b. monitoring blood pressure38 (rationale- a pheochromocytoma is a catecholamine-producing tumor of the adrenal medulla, which may cause severe, episodic HTN; severe, pounding headache; and profuse sweating. Monitoring for dangerously high BP before surgery is critical, as is monitoring for BP fluctuation during medical and surgical tx.)

A patient who is admitted with acute hepatic encephalopathy and ascites receives instructions about appropriate diet. The nurse determines that the teaching has been effective when the patient's choice of foods from the menu includes a. an omelet with cheese and mushrooms and milk. b. pancakes with butter and honey and orange juice. c. baked beans with ham, cornbread, potatoes, and coffee. d. baked chicken with french-fries, low-fiber bread, and tea.

b. pancakes with butter and honey and orange juice. B Rationale: The patient with acute hepatic encephalopathy is placed on a LOW-protein diet to decrease ammonia levels. The other choices are all higher in protein and would not be as appropriate for this patient. In addition, the patient's ascites indicate that a low-sodium diet is needed and the other choices are all high in sodium.

A patient with hypothyroidism is treated with Synthroid. When teaching the patient about the therapy, the nurse a. explains that caloric intake must be reduced when drug therapy is started b. provides written instruction for all information related to the medication therapy c. assures the patient that a return to normal function will occur with replacement therapy d. informs the patient that medications must be taken until hormone balance is reestablished

b. provides written instruction for all information related to the medication therapy (rationale- because of the mental sluggishness, inattentiveness, and memory loss that occur with hypothyroidism, it is important to provide written instructions and repeat information when teaching the patient. Caloric intake can be increased when drug therapy is started, because of an increased metabolic rate, and replacement therapy must be taken for life. Although most patients return to a normal state with treatment, cardiovascular conditions and psychoses may persist.)

When caring for a patient with nephrogenic DI, the nurse would expect treatment to include a. fluid restriction b. thiazide diuretics c. a high-sodium diet d. chlorpropamide (DIabinese)

b. thiazide diuretics (Rationale- in nephrogenic Di the kidney is unable to respond to ADH, so vasopressin or hormone analogs are not effective. Thiazide diuretics slow the glomerular filtration rate in the kidney and produce a decrease in urine output. Low-sodium diets are also thought to decrease urine output. Fluids are not restricted, because the patient could become easily dehydrated.)

"When caring for a client who's being treated for hyperthyroidism, the nurse should:

balance the client's periods of activity and rest.

"When teaching a client with Cushing's syndrome about dietary changes, the nurse should instruct the client to increase intake of foods such as:

bananas and potatoes.

"The nurse is caring for a client who's hypoglycemic. This client will have a blood glucose level:

below 70 mg/dl.

What's the d/o that causes boluses of epi / norepi and what are they: and signs/systoms:

benign tumors - pheochromocytoma / increased B/P and increased HR and pulse

"The nurse is explaining the action of insulin to a client newly diagnosed with diabetes mellitus. During the teaching, the nurse reviews the process of insulin secretion in the body. The nurse is correct when stating that insulin is secreted from the:

beta cells of the pancreas.

HHNS

body is deficient of K from diuresis

"A client with a history of chronic hyperparathyroidism admits to being noncompliant. Based on initial data collection findings, the nurse realizes the client's risk for injury is related to:

bone demineralization resulting in pathologic fractures.

A client is brought to the emergency room in an unresponsive state, and a diagnosis of hyperglycemic hyperosmolar nonketotic syndrome is made. The nurse would immediately prepare to initiate which of the following anticipated physician's orders? a) endotracheal intubation b) 100 units of NPH insulin c) intravenous infusion of normal saline d) intravenous infusion of sodium bicarbonate

c) intravenous infusion of normal saline The primary goal of treatment in hyperglycemic hyperosmolar nonketotic syndrome (HHNS) is to rehydrate the client to restore fluid volume and to correct electrolyte deficiency. Intravenous fluid replacement is similar to that administered in diabetic ketoacidosis (DKA) and begins with IV infusion of normal saline. Regular insulin, not NPH insulin, would be administered. The use of sodium bicarbonate to correct acidosis is avoided because it can precipitate a further drop in serum potassium levels. Intubation and mechanical ventilation are not required to treat HHNS.

diabetic ketoacidosis

kussmaul's respirations, dry skin, hypotension, bradycardia

Which of the followig is an appropriate post op nursing intervention for the pt who has had a partial or total nephrectomy?

label and secur all caths, tubes, and drains

Which of the following persons would most likely be diagnosed with diabetes mellitus? A 44-year-old: a. Caucasian woman. b. Asian woman. c. African-American woman. d. Hispanic male.

c. African-American woman. Age-specific prevalence of diagnosed diabetes mellitus (DM) is higher for African-Americans and Hispanics than for Caucasians. Among those younger than 75, black women had the highest incidence.

The nurse is caring for the client diagnosed with ascites from hepatic cirrhosis. What information should the nurse report to the health-care provider? a. A decrease in the client's daily weight of one (1) pound. b. An increase in urine output after administration of a diuretic. c. An increase in abdominal girth of two (2) inches. d. A decrease in the serum direct bilirubin to 0.6 mg/dL.

c. An increase in abdominal girth of two (2) inches. Rationale: An increase in abdominal girth would indicate that the ascites is increasing, meaning that the client's condition is becoming more serious and should be reported to the health-care provider.

Which of the following diabetes drugs acts by decreasing the amount of glucose produced by the liver? a. Sulfonylureas b. Meglitinides c. Biguanides d. Alpha-glucosidase inhibitors

c. Biguanides Biguanides, such as metformin, lower blood glucose by reducing the amount of glucose produced by the liver. Sulfonylureas and Meglitinides stimulate the beta cells of the pancreas to produce more insulin. Alpha-glucosidase inhibitors block the breakdown of starches and some sugars, which helps to reduce blood glucose levels

The nurse is caring for a patient with a diagnosis of hypothyroidism. Which nursing diagnosis should the nurse most seriously consider when analyzing the needs of the patient? a. High risk for aspiration related to severe vomiting b. Diarrhea related to increased peristalsis c. Hypothermia related to slowed metabolic rate d. Oral mucous membrane, altered related to disease process

c. Hypothermia related to slowed metabolic rate Thyroid hormone deficiency results in reduction in the metabolic rate, resulting in hypothermia, and does predispose the older adult to a host of other health-related issues. One quarter of affected elderly experience constipation.

A patient with Addison's disease comes to the emergency department with complaints of N/V/D, and fever. The nurse would expect collaborative care to include a. parenteral injections of ACTH b. IV administration of vasopressors c. IV administration of hydrocortisone d. IV administration of D5W with 20mEq of KCl

c. IV administration of hydrocortisone (rationale- vomiting and diarrhea are early indicators of addisonian crisis and fever indicates an infection, which s causing additional stress for the patient. treatment of a crisis requires immediate glucocorticoid replacement, and IV hydrocortisone, fluids, sodium and glucose are necessary for 24hours. Addison's disease is a primary insufficiency of the adrenal gland, and ACTH is not effective, nor would vasopressors be effective with the fluid deficiency of Addison's. Potassium levels are increased in Addison's dz, and KCl would be contraindicated.)

"The nurse is working with an overweight client who has a high-stress job and smokes. This client has just received a diagnosis of Type II Diabetes and has just been started on an oral hypoglycemic agent. Which of the following goals for the client which if met, would be most likely to lead to an improvement in insulin efficiency to the point the client would no longer require oral hypoglycemic agents? "a. Comply with medication regimen 100% for 6 months b. Quit the use of any tobacco products by the end of three months c. Lose a pound a week until weight is in normal range for height and exercise 30 minutes daily d. Practice relaxation techniques for at least five minutes five times a day for at least five months"

c. Lose a pound a week until weight is in normal range for height and exercise 30 minutes daily When type II diabetics lose weight through diet and exercise they sometimes have an improvement in insulin efficiency sufficient to the degree they no longer require oral hypoglycemic agents.

During treatment of a patient with a Minnesota balloon tamponade for bleeding esophageal varices, which nursing action will be included in the plan of care? a. Encourage the patient to cough and deep breathe. b. Insert the tube and verify its position q4hr. c. Monitor the patient for shortness of breath. d. Deflate the gastric balloon q8-12hr.

c. Monitor the patient for shortness of breath. Rationale: The most common complication of balloon tamponade is aspiration pneumonia. In addition, if the gastric balloon ruptures, the esophageal balloon may slip upward and occlude the airway. Coughing increases the pressure on the varices and increases the risk for bleeding. The health care provider inserts the tube and verifies the position. The esophageal balloon is deflated every 8 to 12 hours to avoid necrosis, but if the gastric balloon is deflated, the esophageal balloon may occlude the airway.

Causes of primary hypothyroidism in adults include a. malignant or benign thyroid nodules b. surgical removal or failure of the pituitary gland c. surgical removal or radiation of thyroid gland d. autoimmune-induced atrophy of the gland

d. autoimmune-induced atrophy of the gland (rationale- both Graves disease and Hasimotos thyroiditis are autoimmune disorders that eventually destroy the thyroid gland, leading to primary hypothyroidism. Thyroid tumors most often result in hyperthyroidism. Secondary hypothyroidism occurs as a result of pituitary failure, and iatrogenic hypothyroidism results from thyroidectomy or radiation of the thyroid gland.)

A nurse cares for a client following a liver biopsy. Which nursing care plan reflects proper care? a. Position in a dorsal recumbent position, with one pillow under the head b. Bed rest for 24 hours, with a pressure dressing over the biopsy site c. Position to a right side-lying position, with a pillow under the biopsy site d. Neurological checks of lower extremities every hour

c. Position to a right side-lying position, with a pillow under the biopsy site Positioning the client in a right side-lying position with a pillow under the biopsy site reflects proper care. Answer 1 does not permit the necessary pressure applied to the biopsy site. B ed rest is only required for several hours. There is no reason to do neurological checks.

The nurse is having difficulty obtaining a capillary blood sample from a client's finger to measure blood glucose using a blood glucose monitor. Which procedure will increase the blood flow to the area to ensure an adequate specimen? a. Raise the hand on a pillow to increase venous flow. b. Pierce the skin with the lancet in the middle of the finger pad. c. Wrap the finger in a warm cloth for 30-60 seconds. d. Pierce the skin at a 45-degree angle.

c. Wrap the finger in a warm cloth for 30-60 seconds. The hand is lowered to increase venous flow. The finger is pierced lateral to the middle of the pad perpendicular to the skin surface.

A patient with DI is treated with nasal desmopression. The nurse recognize that the drug is not having an adequate therapeutic effect the the patient experiences a. headache and weight gain b. nasal irritation and nausea c. a urine specific gravity of 1.002 d. an oral intake greater than urinary output

c. a urine specific gravity of 1.002 (rationale- normal urine specific gravity is 1.003 to 1.030, and urine with a specific gravity of 1.002 is very dilute, indicating that there continues to be excessive loss of water and that treatment of DI is inadequate. H/A, weight gain, and oral intake greater the urinary output are signs of volume excess that occur with overmedication. Nasal irritation & nausea may also indicate overmedication.)

Mr. L. has a seven-year history of hepatic cirrhosis. He was brought to the emergency room because he began vomiting large amounts of dark-red blood. An Esophageal Balloon Tamponade tube was inserted to tamponade the bleeding esophageal varices. While the balloon tamponade is in place, the nurse caring for Mr. L. gives the highest priority to a. assessing his stools for occult blood. b. evaluating capillary refill in extremities. c. auscultating breath sounds. d. performing frequent mouth care.

c. auscultating breath sounds. Rationale: Airway obstruction and aspiration of gastric contents are potential serious complications of balloon tamponade. Frequent assessment of the client's respiratory status is the priority.

When providing discharge instructions to a patient following a subtotal thyroidectomy, the nurse advises the patient to a. never miss a daily dose of thyroid replacement therapy b. avoid regular exercise until thyroid function is normalized c. avoid eating foods such as soybeans, turnips, and rutabagas d. use warm salt water gargles several times a day to relieve throat pain

c. avoid eating foods such as soybeans, turnips, and rutabagas (Rationale- when a patient has had a subtotal thyroidectomy, thyroid replacement therapy is not given, because exogenous hormone inhibits pituitary production of TSH and delays or prevents the restoration of thyroid tissue regeneration. However, the patient should avoid goitrogens, foods that inhibit thyroid, such as soybeans, turnips, rutabagas, and peanut skins. REgular exercise stimulates the thyroid gland and is encourage. Salt water gargles are used for dryness and irritation of the mouth and throat following radioactive iodine therapy.)

To prevent complications in the patient with Cushing syndrome, the nurse monitors the patient for a. hypotension b. hypoglycemia c. cardiac arrhythmias d. decreased cardiac output

c. cardiac arrhythmias (rationale- electrolyte changes that occur in Cushing syndrome include sodium retention and potassium excretion by the kidney, resulting in hypokalemia, which may lead to cardiac arrhythmias or arrest. Hypotension, hypoglycemia, and decreased cardiac strength and output are characteristic of adrenal insufficiency.)

A patient received 6 units of REGULAR INSULIN 3 hours ago. The nurse would be MOST concerned if which of the following was observed? a. kussmaul respirations and diaphoresis b. anorexia and lethargy c. diaphoresis and trembling d. headache and polyuria

c. diaphoresis and trembling indicates hypoglycemia

A patient is admitted to the hospital in thyrotoxic crisis. On physical assessment of the patient, the nurse would expect to find a. hoarseness and laryngeal stridor b. bulging eyeballs and arrhythmias c. elevated temperature and signs of heart failure d. lethargy progressing suddenly to impairment of consciousness

c. elevated temperature and signs of heart failure (rationale- a hyperthyroid crisis results in marked manifestations of hyperthyroidism, with fever tachycardia, heart failure, shock, hyperthermia, agitation, N/V/D, delirium, and coma. Although exophthalmos may be present in the patient with Gravs' dz, it is not a significant factor in hyperthyroid crisis. Hoarsness and laryngeal stridor are characteristic of the tetany of hypoparathyroidism, and lethargy progressing to coma is characteristic of myxedema coma, a complication of hypothyroidism.

A patient is scheduled for bilateral adrenalectomy. During the postoperative period, the nurse would expect administration of corticosteroids to be a. reduced to promote wound healing b. withheld until symptoms of hypocortisolism appear c. increased to promote an adequate response to the stress of surgery d. reduced because excessive hormones are released during surgical manipulation of the glands

c. increased to promote an adequate response to the stress of surgery (rationale- although the patient with Cushing syndrome has excess corticosteroids, removal of the glands and the stress of surgery require that high doses of cortisone be administered postoperatively for several days. The nurse should monitor the patient postoperatively to detect whether large amounts of hormones were released during surgical manipulation and to ensure the healing is satisfactory.)

A patient with acute pancreatitis has a nasogastric (NG) tube to suction and is NPO. The nurse explains to the patient that the major purpose of this treatment is a. control of fluid and electrolyte imbalance. b. relief from nausea and vomiting. c. reduction of pancreatic enzymes. d. removal of the precipitating irritants.

c. reduction of pancreatic enzymes. Rationale: Pancreatic enzymes are released when the patient eats. NG suction and NPO status decrease the release of these enzymes. Fluid and electrolyte imbalances will be caused by NG suction and require that the patient receive IV fluids to prevent this. The patient's nausea and vomiting may decrease, but this is not the major reason for these treatments. The pancreatic enzymes that precipitate the pancreatitis are not removed by NG suction.

During assessment of the patient with acromegaly, the nurse would expect the patient to report a. infertility b. dry, irritated skin c. undesirable changes in appearance d. an increase in height of 2 to 3 inches per year

c. undesirable changes in appearance (Rationale- the increased production of growth hormone in acromegaly causes an increase in thickness and width of bones and enlargement of soft tissues, resulting in marked changes in facial features, oily and coarse skin, and speech difficulties. Height is not increased in adults with growth hormone excess because the epiphyses of the bones are closed, and infertility is not a common finding because growth hormone is usually the only pituitary hormone involved in acromegaly.)

addisonian crisis

caused by deficiencies of cortisol and aldosterone

glucocorticoid deficiency

causes decrease in cardiac output and vascular tone, leading to hypovolemia. pt becomes tachycardic and hypotensive and may develop shock and circulatory response.

hyperthyroidism

causes goiter, nervousness, heat intolerance and weight loss despite increased appetite

hypocalcemia

causes muscle twitching, numbness or thingling of the lips, fingers and toes (signs of hyperirritability of the nervous system)

cushings syndrome

condition caused by hypersecretion of cortisol by the adrenal cortex resulting in breakdown of muscle protein and redistribution of body fat,

"The nurse is providing dietary instructions to a client with hypoglycemia. To control hypoglycemic episodes, the nurse should recommend:

consuming a low-carbohydrate, high-protein diet and avoiding fasting.

s/s diabetes insipidus

excretion of laerge amounts of fluid (polyuria) extreme thirst(polydipsia)low specific gravity occur 3-6 days after a head injury

will addison's be gaining or losing weight?

losing - not keeping their water

"An adolescent client with type I diabetes mellitus is admitted to the emergency department for treatment of diabetic ketoacidosis. Which assessment findings should the nurse expect to note? "a) sweating and tremors b) hunger and hypertension c) cold, clammy skin and irritability d) fruity breath and decreasing level of consciousness

d) fruity breath and decreasing level of consciousness"Hyperglycemia occurs with diabetic ketoacidosis. Signs of hyperglycemia include fruity breath and a decreasing level of consciousness. Hunger can be a sign of hypoglycemia or hyperglycemia, but hypertension is not a sign of diabetic ketoacidosis. Instead, hypotension occurs because of a decrease in blood volume related to the dehydrated state that occurs during diabetic ketoacidosis. Cold, clammy skin, irritability, sweating, and tremors are all signs of hypoglycemia."

client newly diagnosed with diabetes mellitus has been stabilized with daily insulin injections. A nurse prepares a discharge teaching plan regarding the insulin and plans to reinforce which of the following concepts? a) always keep insulin vials refrigerated b) ketones in the urine signify a need for less insulin c) increase the amount of insulin before unusual exercise d) systematically rotate insulin injections within one anatomic site

d) systematically rotate insulin injections within one anatomic site Insulin doses should not be adjusted nor increased before unusual exercise. If ketones are found in the urine, it possibly may indicate the need for additional insulin. To minimize the discomfort associated with insulin injections, insulin should be administered at room temperature. Injection sites should be rotated systematically within one anatomic site.

A patient with Grave's dz asks the nurse what caused the disorder. The best response by the nurse is a. "The cause of Grave's disease is not known, although it is thought to be genetic." b. "It is usually associated with goiter formation from an iodine deficiency over a long period of time." c. "Antibodies develop against thyroid tissue and destroy it, causing a deficiency of thyroid hormones" d. "In genetically susceptible persons antibodies form that attack thyroid tissue and stimulate overproduction of thyroid hormones."

d. "In genetically susceptible persons antibodies form that attack thyroid tissue and stimulate overproduction of thyroid hormones." (rationale- The antibodies present in Graves' disease that attack thyroid tissue cause hyperplasia of the gland and stimulate TSH receptors on the thyroid and activate the production of thyroid hormones, creating hyperthyroidism. The disease is not directly genetic, but individuals appear to have a genetic susceptibility to become sensitized to develop autoimmune antibodies. Goiter formation from insufficient iodine intake is usually associated with hypothyroidism.)

When working in the community, the nurse will recommend routine screening for diabetes when the person has one or more of seven risk criteria. Which of the following persons that the nurse comes in contact with most needs to be screened for diabetes based on the seven risk criteria? a. A woman who is at 90% of standard body weight after delivering an eight-pound baby b. A middle-aged Caucasian male c. An older client who is hypotensive d. A client with an HDL cholesterol level of 40 mg/dl and a triglyceride level of 300 mg/dl

d. A client with an HDL cholesterol level of 40 mg/dl and a triglyceride level of 300 mg/dl The seven risk criteria include: greater than 120% of standard body weight, Certain races but not including Caucasian, delivery of a baby weighing more than 9 pounds or a diagnosis of gestational diabetes, hypertensive, HDL greater than 35 mg/dl or triglyceride level greater than 250 or a triglyceride level of greater than 250 mg/dl, and, lastly, impaired glucose tolerance or impaired fasting glucose on prior testing.

Also with too many glucocorticoids, you may see increased risk of __________, ______glycemia, __________ to depression, ______-faced, _______-obesity and __________ hump.

infection / hyperglycemia / psychosis to depression / moon faced - fat redistribution or fluid retention / truncal obesity and buffalo hump also all from fat redistribution

Glucocorticoids are also known as anti___________

inflammatories... this is why cortisol cream is for anti itch cream... they stop the fighters in the body... the reaction.

When taking a health history, the nurse screens for manifestations suggestive of diabetes type I. Which of the following manifestations are considered the primary manifestations of diabetes type I and would be most suggestive of diabetes type I and require follow-up investigation? "a. Excessive intake of calories, rapid weight gain, and difficulty losing weight b. Poor circulation, wound healing, and leg ulcers, c. Lack of energy, weight gain, and depression d. An increase in three areas: thirst, intake of fluids, and hunger

d. An increase in three areas: thirst, intake of fluids, and hunger "The primary manifestations of diabetes type I are polyuria (increased urine output), polydipsia (increased thirst), polyphagia (increased hunger). Excessive calorie intake, weight gain, and difficulty losing weight are common risk factors for type 2 diabetes. Poor circulation, wound healing and leg ulcers are signs of chronic diabetes. Lack of energy, weight gain and depression are not necessarily indicative of any type of diabetes."

Which of the following diagnostic tests should be performed annually over age 50 to screen for colon cancer? a. Abdominal CT scan b. Abdominal x-ray c. Colonoscopy d. Fecal occult blood test

d. Fecal occult blood test Surface blood vessels of polyps and cancers are fragile and often bleed with the passage of stools. Abdominal x-ray and CT scan can help establish tumor size and metastasis. A colonoscopy can help locate a tumor as well as polyps, which can be removed before they become malignant.

The nurse working in the physician's office is reviewing lab results on the clients seen that day. One of the clients who has classic diabetic symptoms had an eight-hour fasting plasma glucose test done. The nurse realizes that diagnostic criteria developed by the American Diabetes Association for diabetes include classic diabetic symptoms plus which of the following fasting plasma glucose levels? a. Greater than 106 mg/dl b. Greater than 126 mg/dl c. Higher than 140 mg/dl d. Higher than 160 mg/dl

d. Higher than 160 mg/dl

A frail elderly patient with a diagnosis of type 2 diabetes mellitus has been ill with pneumonia. The client's intake has been very poor, and she is admitted to the hospital for observation and management as needed. What is the most likely problem with this patient? a. Insulin resistance has developed. b. Diabetic ketoacidosis is occurring. c. Hypoglycemia unawareness is developing. d. Hyperglycemic hyperosmolar non-ketotic coma

d. Hyperglycemic hyperosmolar non-ketotic coma Illness, especially with the frail elderly patient whose appetite is poor, can result in dehydration and HHNC. Insulin resistance usually is indicated by a daily insulin requirement of 200 units or more. Diabetic ketoacidosis, an acute metabolic condition, usually is caused by absent or markedly decreased amounts of insulin.

Proliferative retinopathy is often treated using: a. Tonometry b. Fluorescein angiogram c. Antibiotics d. Laser surgery

d. Laser surgery Scatter laser treatment is used to shrink abnormal blood vessels in an effort to preserve vision. When there is significant bleeding in the eye, it is removed in a procedure known as vitrectomy. Tonometry is a diagnostic test that measures pressure inside the eye. A fluorescein angiogram is a diagnostic test that traces the flow of dye through the blood vessels in the retina; it is used to detect macular edema.

Of which of the following symptoms might an older woman with diabetes mellitus complain? a. Anorexia b. Pain intolerance c. Weight loss d. Perineal itching

d. Perineal itching

A patient suspected of having acromegaly has an elevated plasma growth hormone level. In acromegaly, the nurse would also expect the patient's diagnostic results to include a. hyperinsulinemia b. a plasma glucose of less than 70 c. decreased growth hormone levels with an oral glucose challenge test d. a serum sometomedin C (insulin-like growth-factor) of more than 300

d. a serum somatomedin C (Insulin-like-growth-factor) of more than 300 (rationale- a normal response to growth hormone secretion is stimulation of the liver to produce somatomedin C which stimulates growth of bones and soft tissue. The increased levels of somatomedin C normally inhibit growth hormone, but in acromegaly the pituitary gland secretes GH despite elevated somatomedin C levels.)

A college student who has type 1 diabetes normally walks each evening as part of an exercise regimen. The student now plans to take a swimming class every day at 1:00 PM. The clinic nurse teaches the patient to a. delay eating the noon meal until after the swimming class. b. increase the morning dose of neutral protamine Hagedorn (NPH) insulin on days of the swimming class. c. time the morning insulin injection so that the peak occurs while swimming. d. check glucose level before, during, and after swimming.

d. check glucose level before, during, and after swimming. Rationale: The change in exercise will affect blood glucose, and the patient will need to monitor glucose carefully to determine the need for changes in diet and insulin administration. Because exercise tends to decrease blood glucose, patients are advised to eat before exercising. Increasing the morning NPH or timing the insulin to peak during exercise may lead to hypoglycemia, especially with the increased exercise.

Physical changes of hypothyroidism that must be monitored when replacement therapy is started include a. achlorhydria and constipation b. slowed mental processes and lethargy c. anemia and increased capillary fragility d. decreased cardiac contractility and coronary atherosclerosis

d. decreased cardiac contractility and coronary atherosclerosis (rationale- hypothyroidism affects the heart in many ways, causing cardiomyopathy, coronary atherosclerosis, bradycardia, pericardial effusions, and weakened cardiac contractility. when thyroid replacement therapy is started, myocardial oxygen consumption is increased and the resultant oxygen demand may cause angina, cardiac arrhythmias, and heart failures. It is important to monitor patients with compromised cardiac status when starting replacement therapy.)

When lactulose (Cephulac) 30 ml QID is ordered for a patient with advanced cirrhosis, the patient complains that it causes diarrhea. The nurse explains to the patient that it is still important to take the drug because the lactulose will a. promote fluid loss. b. prevent constipation. c. prevent gastrointestinal (GI) bleeding. d. improve nervous system function.

d. improve nervous system function. Rationale: The purpose for lactulose in the patient with cirrhosis is to lower ammonia levels and prevent encephalopathy. Although the medication may promote fluid loss through the stool, prevent constipation, and prevent bearing down during bowel movements (which could lead to esophageal bleeding), the medication is not ordered for these purposes for this patient.

In a patient with central diabetes insipidus, administration of aqueous vasopressin during a water deprivation test will result in a a. decrease in body weight b. increase in urinary output c. decrease in blood pressure d. increase in urine osmolality

d. increase in urine osmolality (rationale- a patient with DI has a deficiency of ADH with excessive loss of water from the kidney, hypovolemia, hypernatreamia, and dilute urine with a low specific gravity. When vasopressin is administered, the symptoms are reversed, with water retention, decreased urinary output that increases urine osmolality, and an increase in blood pressure.)

The nurse is performing discharge teaching for a patient with Addison's disease. It is MOST important for the nurse to instruct the patient about: a. signs and symptoms of infection b. fluid and electrolyte balance c. seizure precautions d. steroid replacement

d. steroid replacement steroid replacement is the most important information the client needs to know.

Colon cancer is most closely associated with which of the following conditions? a. appendicitis b. hemorroids c. hiatal hernia d. ulcerative colitis

d. ulcerative colitis Chronic ulcerative colitis, granulomas, and familial polyps seem to increase a person's chance of developing colon cancer. The other conditions listed have no known effect on the colon cancer risk.

hyperadlosteronism (too much aldosterone) makes serum potassium:

decrease

The dawn phenomenon results from a _______ in tissue sensitivity to __________ that occurs between ___-___am (pre-breakfast hyperglycemia); caused by a release of nocturnal growth hormones. TX: give _______-______ insulin at 10pm.

decrease > insulin > 5-8am > intermediate-acting

"The client is being evaluated for hypothyroidism. The nurse should stay alert for:

decreased body temperature and cold intolerance.

s/s of addisons disease

decreased sodium,& blood volume postural hypotention and syncope,possible hypovolemic shock, tachycardia,arrhythmias,tremors confusion hypoglycemia hyponatremia, diarrhea & hyperkalemia

The nurse is collecting data on a client with possible Cushing's syndrome. In a client with Cushing's syndrome, the nurse would expect to find:

deposits of adipose tissue in the trunk and dorsocervical area.

S/S of Graves disease or hyperthyroidism

elevated metabolic rate, sweating, rapid heartbeat, nervousness, and weight loss. and exophthalmos.

Nursing dx with addson's?

fluid volume deficit

addison's disease means they do not have enough of:

glucocorticiois, mineralocorticoids or sex hormones

Why might sugar be low with addisons?

glucocorticoids are one of the things the adrenal gland and the adrenal gland is not working so glucocorticoids, sex hormones and mineralocorticoids are not in full force... so low glucocorticoids - hypoglycemic. :)

The adrenal cortex secretes which hormones:

glucocorticoids, mineralocorticoids and sex hormones

what could you have in the urine of a cushings pt?

glucose and keytones - remember the keytones are made by the breaking down of protein and fat... we have lots of glucocorticoids.

hyperthyroidism

grave's disease and thyrotoxicosis

what two weird growth problems might you see symptoms of with too many glucocorticoids with cushings?

growth arrest and thin extremities / skin (lipolysis)

hypercalcemia

hallmark of excess parathyroid hormone levels

"A client with type 1 diabetes asks the nurse about taking an oral antidiabetic agent. The nurse explains that these medications are only effective if the client:

has type 2 diabetes.

"Hyperthyroidism is caused by increased levels of thyroxine in blood plasma. A client with this endocrine dysfunction would experience:

heat intolerance and systolic hypertension.

hypothyroidism treatment

hormone replacement

"The physician diagnoses type 1 diabetes in a client who has classic manifestations of the disease and a random blood glucose level of 350 mg/dl. In addition to dietary modifications, the physician prescribes insulin. Initially, most clients receive the least antigenic form of insulin. Therefore, the nurse expects the physician to prescribe:

human insulin.

Hyperparathyroidism=________________=__________

hypercalcemia=hypophosphatemia

Signs and symptoms of addisons disease are innitally those of _______________, which are:

hyperkalemia / beginning with muscle twitching, then proceeds to weakness and then flaccid paralysis. (remember if don't get that bannanna from not enough k, then will have muscle rigidity and cramping)

"A 68-year-old client has been complaining of sleeping more, increased urination, anorexia, weakness, irritability, depression, and bone pain that interferes with her going outdoors. Based on these findings, the nurse would suspect:

hyperparathyroidism.

"Following a transsphenoidal hypophysectomy, the nurse should assess the client carefully for:

hypocortisolism

Cretinism

hyposecretion of thyroid hormone in fetus or neonate causing severe, irreversible mental retardation if not treated - requires lifelong HRT

Myxedema

hyposecretion of thyroxine (T4) and T3 causing weakness and fatigue; increased sensitivity to cold, constipation, dry skin, unexplained weight gain, depression, facial edema, goiter, nonpitting puffy appearance

with an addisonian crisis there are what major symptoms?

hypotension and vascular collapse (NO FLUID)

"Before undergoing a subtotal thyroidectomy, a client receives potassium iodide (Lugol's solution) and propylthiouracil (PTU). The nurse would expect the client's symptoms to subside:

in 1 to 2 weeks.

increased ACTH = __________ cortisol level.

increased

Important nursing interventions to prevent acute renal railure in the critically ill pt include which of the follwing?

maintain fluid voluem and cardiac output

Cushing's syndrome/disease clients have too many __________, _________, and ______ hormones.

mineralcorticoids, glucocorticoids, and sex hormones

Addisons disease care

monitor I&Os, weight, electrolytes levels

adrenal adenoma

most common cause of hyperaldosteronism

Hashimoto's thyroiditis

most common cause of hypothyroidism. an autoimmune disorder, causes wt gain, decreased appetite, constipation, lethargy, dry cool skin, brittle nails, coarse hair, muscle cramps, weakness and sleep apnea

"An incoherent client with a history of hypothyroidism is brought to the emergency department by the rescue squad. Physical and laboratory findings reveal hypothermia, hypoventilation, respiratory acidosis, bradycardia, hypotension, and nonpitting edema of the face and pretibial area. Knowing that these findings suggest severe hypothyroidism, the nurse prepares to take emergency action to prevent the potential complication of:

myxedema coma.

"A client with diabetes mellitus has just been prescribed insulin. When teaching the client about hypoglycemia, the nurse should mention that this reaction may cause:

nervousness, diaphoresis, and confusion.

s/sx of hypothyroid:

no energy (thyroid hormone gives us energy), fatigue, GI system is slow, weight is up, they are cold, speech is slow and no expression

with addison's disease we have ___________ aldosterone which is a mineralocorticoid... so, we have how much fluid? what do they retain?

not enough / losing water and sodium / fluid deficit / pt retains potassium becuase aldosterone usually gets rid of potassium and keeps na and h20.

following a kidney transplant, the nurse notes that the pt's urine is cloudy. how should the nurse respond?

notify the physician

type 2 diabetes

only type of diabetes where oral antidiabetic agent are effective

"Capillary glucose monitoring is being performed every 4 hours for a client diagnosed with diabetic ketoacidosis. Insulin is administered using a scale of regular insulin according to glucose results. At 2 p.m., the client has a capillary glucose level of 250 mg/dl for which he receives 8 units of regular insulin. The nurse should expect the dose's:

onset to be at 2:30 p.m. and its peak to be at 4 p.m.

Pre-op care for t. hypophysectomy

perform capillary glucose testing q4h because excess cortisol may cause insulin resistance, placing the pt at risk for hyperglycemia

"A client is scheduled for a transsphenoidal hypophysectomy to remove a pituitary tumor. Preoperatively, the nurse should assess for potential complications by:

performing capillary glucose testing frequently.

"During a follow-up visit to the physician, a client with hyperparathyroidism asks the nurse to explain the physiology of the parathyroid glands. The nurse states that these glands produce parathyroid hormone (PTH). PTH maintains the balance between calcium and:

phosphorus.

"A client is admitted to an acute care facility with a tentative diagnosis of hypoparathyroidism. The nurse should monitor the client closely for the related problem of:

profound neuromuscular irritability.

aldosterone

promotes Na conservation and K excretion.

thyroid

propylthiouracil blocks _______ hormone production - can cause agranulocytosis

"A client visits the physician's office complaining of agitation, restlessness, and weight loss. The physical examination reveals exophthalmos, a classic sign of Graves' disease. Based on history and physical findings, the nurse suspects hyperthyroidism. Exophthalmos is characterized by:

protruding eyes and a fixed stare.

Glycosylated hemoglobin level

provides information about blood glucose levels during the previous 3 months

Make sure for cushings pt's that they have a __________ environment because they can't handle stress.

quiet - the glucocorticoids cause mood altering from psychosis to depression / insomnia etc. / even with more epi and norepi they could fly off the handle... with none they are flat affect.

glipzide (glucotrol) adverse reaction

rash, pruritus and photosensitivity

10 to 15 g of simple carbohydrates

recommended to reverse hypoglycemia. 3-5 pieces of hard candy, 2-3 packets of sugar or 4oz of fruit juice. if necessary, can be repeated in 15 min

colchicine

reduce the crystal deposits in the joints and ease the inflammation

"In a 28-year-old female client who is being successfully treated for Cushing's syndrome, the nurse would expect a decline in:

serum glucose level.

"A client with type 1 diabetes has been on a regimen of multiple daily injection therapy. He's being converted to continuous subcutaneous insulin therapy. While teaching the client about continuous subcutaneous insulin therapy, the nurse would be accurate in telling him the regimen includes the use of:

short-acting insulin only.

Processed fruit juice has lots of?

sodium

"The nurse is assigned to care for a postoperative client who has diabetes mellitus. During data collection, the client reports that he's impotent and says he's concerned about its effect on his marriage. In planning this client's care, the most appropriate intervention would be to:

suggest referral to a sex counselor or other appropriate professional.

treatment of Cushings syndrome

surgery, radiation, or chemotherapy Monitor weight ,I&O,Encourage turn cough & deep breathing

hypoglycemia

sweating, tremor, tachycardia, palpitations, nervousness, light-headedness, hunger, cool skin, seizures, irritability, confusion, coma

s/s thyroid storm

tachycardia, agitation, coma, hypotension Sweating, Hyperactivity, Tachycardia High temperature, tachypnea, Dehydration, GI disturbances, Delerium, coma

Levothyroxine (Synthroid)

the agent of choice for thyroid hormone replacement because its standard hormone content gives it predictable results

Graves disease

the most common form of hyperthyroidism; caused by an autoimmune defect that creates antibodies that stimulate the overproduction of thyroid hormones; ,

steriods decrease serum calcium how?

they exrete it through the GI tract.

hypercortisolism is just another word for

too many steriods

treatmentof thyroid storm

treated with iodine, PTU, beta blocker for tachycardia, cooling, sedation, glucocorticoids to decrease swelling, and IV fluids

s/s Cushings syndrome

truncal obesity, round moon face, supraclavicular fat pads (buffalo hump, edema, hypertension, muscular weakness and wasting, skin infection.. Decreased potassium increased soduim, , BUN and glucose .

ca gluconate

used to reverse a negative ca balance and relieve muscle cramps

"The nurse is instructing a client with newly diagnosed hypoparathyroidism about the regimen used to treat this disorder. The nurse should state that the physician probably will prescribe daily supplements of calcium and:

vitamin D.

hypocortisolism

vomiting, weakness, dehydration and hypotension. steroids should be given before surgery (hypophysectomy) to prevent hypocortisolism from occuring

"The nurse is developing a teaching plan for a client with diabetes mellitus. A client with diabetes mellitus should:

wash and inspect his feet daily.

S/S SIADH

water retention, hyponatremia, & seurm hypo-osmlality; blood volume expands, but plasma diluted brain cells swell causing headache ;change in mental status or personality lethragy and irritability weight gain occurs without edema

Cushing's syndrome

weight gain, facial hair, amenorrhea, frequent bruising and acne. affects women ages 20 to 40. causes changes in fat distribution, adipose tissue accumulates in the trunk, face (moonfaace) and dorsocervical areas (buffalo hump)

what is vitiligo?

white pachy area of depigmented skin (caused by addisons)

Tx Hyperthyroid:

with PTU and Methimazole and Iodine compounds and Beta Blockers (lols)-decreases anxiety, radioactive iodine

With Addison's, there's a problem with what gland: / you think not enough steriods, shock and high potassium? what thoughts do you have about how shock could be an outcome?

with addisons.. it's a problem with the adrenal cortex and there's not enough steroids (the glucocorticoids, sex hormones and mineralocorticoids - these are aldosterone which normal hold water and sodium and get rid of potassium... but we are now getting rid of water and soduim and keeping the potassium. The lack of fluid - dehydration and with dehydration or fluid deficit, always come worst case senario = shock.

hypothyroidism

wt gain, constipation, lethargy, decreased sweating and cold intolerance

hypethyroidism

wt loss, nervousness, tachycardia, exopthalmos, diaphoresis, fever and diarrhea

diagnosis for pheochromocytoma:

you do a VMA / no vanilla for a week and take it easy. Don't want to release any epi/norepi that you don't have to, so you can see if it comes from a tumor or not.

Why would you need to avoid infection with cushings?

you have too many steriods. glucocorticoids are immunosuppressors... they and antiinflammatory.


Ensembles d'études connexes

Constitution (Separation of Powers)

View Set

Medical law smartbooks 1 through

View Set

Ch 42: Management of Patients with Musculoskeletal Trauma (3)

View Set

Research Methods Chapter 4, Research Methods Exam 1

View Set

MKT 300 Concept Check Exam 2 Review

View Set

Week 8 Clinical Review Questions

View Set